UBP book 3

Lakukan tugas rumah & ujian kamu dengan baik sekarang menggunakan Quizwiz!

A 42-year-old, 84 kg, man with an 8-year history of chronic renal failure is brought to the operating room for nephrectomy and cadaveric kidney transplantation harvested 16 hours ago. He has a functioning AV fistula in the left forearm and received dialysis 24 hours ago. The nurse states he is extremely anxious. Medications include nifedipine, lisinopril, and ranitidine. Vital Signs: P = 95, BP = 195/115 mmHg, RR = 22, T = 37°C. 1) Are you concerned about this patient's blood pressure? 2) What is your cut off blood pressure for cancelling a case? 3) Will you treat this blood pressure prior to proceeding with the case? 4) How would you assess this patient's volume status? 5) Is a potassium level of 5.2 mEq/L significant in this patient with chronic renal failure? 6) What would be your choice of anesthesia for this case? 7) Could you utilize regional anesthesia for this case?

1) Are you concerned about this patient's blood pressure? I am concerned because this patient with stage 3 hypertension (> 180/110 mmHg) with renal failure may be at increased perioperative risk for blood pressure lability, myocardial ischemia, dysrhythmias, congestive heart failure, stroke, and other end-organ ischemia. Therefore, I would: 1) perform a focused history and physical to identify any additional end-organ damage (i.e. left ventricular hypertrophy or a strain pattern on ECG); 2) obtain an ECG, electrolyte panel, blood urea nitrogen, and creatinine to further evaluate end-organ damage and identify metabolic derangements resulting from medications used in the treatment of hypertension; and 3) consult the transplant team to determine how long I could reasonably delay this urgent case to achieve better control of the patient's blood pressure, recognizing that longer graft cold-ischemia periods can lead to delayed, impaired, or even failed graft function. My treatment goal would be to carefully reduce the patient's blood pressure to less than 160/110 mmHg over several hours, while avoiding end-organ hypoperfusion. Additionally, I would administer a beta-blocker to reduce the risk of perioperative hemodynamic lability and myocardial ischemia. 2) What is your cut off blood pressure for cancelling a case? In making a decision to delay or cancel a case, I would weigh the benefits of blood pressure optimization against the risk of delaying surgery. The evidence suggests that patients with stage 1 hypertension (140-159/90-99 mmHg) or stage 2 hypertension (160-179/100-109 mmHg) may be more prone to perioperative ventricular dysrhythmias, myocardial ischemia, and blood pressure lability. However, they do not appear to be at increased risk of worsening perioperative outcomes such as stroke, myocardial infarction, renal failure, and death. Unfortunately, there may by increased perioperative risk of major cardiovascular complications for patients with concomitant end-organ damage (renal insufficiency or left ventricular hypertrophy) and/or stage 3 hypertension (> 180/110 mmHg). Additionally, patients with blood pressures greater than 140/90 mmHg who are undergoing cardiac surgery, carotid surgery, or pheochromocytoma resection, may also be at increased risk for major perioperative complications. Therefore, I would prefer to delay elective surgery for at least 6-8 weeks to optimize the blood pressure of any patient who: 1) has stage 3 baseline hypertension, 2) has stage 1 or stage 2 hypertension with concomitant end-organ damage, and/or 3) is undergoing cardiac surgery, carotid surgery, or pheochromocytoma resection. However, as I mentioned, the decision to delay any case (and the length of that delay) must weigh the risks of blood pressure optimization against the risk of surgical delay. In the case of emergency surgery, I would carefully reduce the patient's blood pressure to less than 160/110 mmHg and ensure adequate beta-blockade (if not contraindicated) to reduce the risk of perioperative hemodynamic lability and myocardial ischemia, while avoiding end-organ hypoperfusion. 3) Will you treat this blood pressure prior to proceeding with the case? Given the potential perioperative risks associated with severe baseline hypertension, I would make the transplant team aware of my concerns, determine how long I could reasonably delay this urgent case, carefully reduce the patient's blood pressure to less than 160/110 mmHg (over several hours, if possible, to avoid inducing end-organ ischemia), ensure adequate beta-blockadeto reduce the risk of perioperative hemodynamic lability and myocardial ischemia, and proceed with the case. Assuming there were no contraindications, I would treat his hypertension with a short acting beta-blocker like esmolol and, if that were not effective, consider a more potent vasodilator such as sodium nitroprusside. 4) How would you assess this patient's volume status? In assessing this patient's volume status, I would look at his vital signs, take into consideration the time of his last dialysis (dialysis-dependent patients should be dialyzed before renal transplant surgery), compare his current weight with previous predialysis and postdialysis weights, and examine the patient for signs of either hypovolemia or hypervolemia. Signs of hypervolemia include: pulmonary edema, hypertension, peripheral edema, and JVD; while signs of hypovolemia include dry mucous membranes, hypotension, tachycardia, and orthostasis. Proper volume assessment is important for these patients at risk for both hypervolemic and hypovolemic complications. While over-aggressive hydration can lead to congestive heart failure and pulmonary edema, inadequate replacement may result in intra-operative hypotension, end-organ ischemia, and/or postoperative acute tubular necrosis (ATN). In fact, maintaining adequate intravascular volume is associated with earlier onset of graft function, reduced incidence of delayed graft function, and improved graft survival. 5) Is a potassium level of 5.2 mEq/L significant in this patient with chronic renal failure? It is significant, because a potassium level above 5.0 mEq/L represents hyperkalemia. Since significant hyperkalemia places the patient at risk of skeletal muscle weakness and delayed cardiac depolarization with potential progression to respiratory failure, ventricular fibrillation, and ventricular asystole, elective surgery should be delayed when potassium levels exceed 5.5 mEq/L. This patient's hyperkalemia is likely chronic in nature, which is better tolerated than an acute increase in plasma potassium. Therefore, I would proceed with this case despite his elevated potassium, recognizing that there is some additional risk due to the added potassium load following the washout of potassium-containing preservative solution from the newly perfused kidney. If his potassium level were > 5.5 mEq/L, I would delay surgery for dialysis and correction of this electrolyte disturbance. Fortunately, current preservation techniques allow for up to 48 hours of cold-ischemia time prior to transplant, providing sufficient time for pre-operative dialysis. 6) What would be your choice of anesthesia for this case? Kidney transplant has been successfully performed under regional anesthesia. However, the subsequent sympathectomy may complicate systemic blood pressure management, and uremic platelet dysfunction and/or residual heparin from preoperative dialysis may increase the risk of neuraxial hematoma. Therefore, assuming he did not have a difficult airway, I would choose to provide a balanced general anesthetic, using a volatile agent (avoid sevoflurane the production of fluoride and compound A have been implicated in renal toxicity), a short acting opioid (consider using opioids without active metabolites that depend on renal excretion), and a muscle relaxant (mivacurium, atracurium, cisatracurium do not depend on renal excretion) in order to provide optimal operating conditions, while at the same time, maintaining hemodynamic stability and adequate end-organ perfusion. 7) Could you utilize regional anesthesia for this case? Yes. In the case of a difficult airway or strong patient preference, and assuming there were no contraindications, I would consider utilizing regional anesthesia. In preparing for regional anesthesia, I would inform the patient of potential complications; ensure adequate hydration; prepare to address any acute hypotension; obtain coagulation studies (specifically a PTT); and look for signs of coagulopathy such as marked or easy bruising, bleeding at the IV sight, and/or mucosal bleeding (CRF patients may have platelet dysfunction that results in abnormal coagulation despite normal coagulation studies). In performing the procedure, I would take precautions to avoid vascular trauma (i.e. single pass, injection of local or saline through epidural needle prior to placing the catheter) and slowly raise the block to avoid rapid sympathectomy. Finally, I would develop a plan to convert to general anesthesia should regional anesthesia prove insufficient.

Complications associated with mediastinoscopy include:

(1) tracheal compression or laceration, (2) cerebrovascular events (secondary to innominate artery compression), (3) right upper limb ischemia (also secondary to innominate artery compression), (4)compression of the aorta leading to reflex bradycardia, (5) pneumothorax, (6) recurrent laryngeal or phrenic nerve injury, (7) venous air embolism, (8) mediastinal hemorrhage, and (9) esophageal tear.

1) After extubation in the operating room, the patient develops rapid and labored breathing. Auscultation of the lungs reveals diffuse rales. Will you reintubate? 2) The chest x-ray suggests bilateral pulmonary edema and his oxygen saturation is 84%. How might mechanical ventilation prove beneficial? 3) If you didn't place a pulmonary artery catheter for the case, would you place one now? 4) One hour postoperatively, the patient is oozing from the surgical incision. What do you think is going on? 5) What is the cause of uremic thrombocytopathia? 6) How do you treat uremic thrombocytopathia? 7) In the first few hours after transplantation the urine output is only 10 cc/hr. Are you concerned? Would urine lab studies help with a diagnosis?

1) After extubation in the operating room, the patient develops rapid and labored breathing. Auscultation of the lungs reveals diffuse rales. Will you reintubate? I would re-intubate at any point that delaying a definitive airway for further evaluation would place the patient at an unacceptable amount of risk. However, if the patient were stable enough to allow further investigation, I would provide supplemental oxygen and assess oxygenation and ventilation by carefully auscultating all lung fields, evaluating the patient's oxygen saturation and inspiratory effort, and obtaining an ABG. Next, I would evaluate his cardiovascular function by checking the CVP, PAC (if present), EKG, and arterial blood pressure; and by ordering a chest x-ray and, possibly, an echocardiogram. Additionally, I would check the patient's electrolytes and review all medications, giving careful consideration to whether renal excretion played a significant role in the elimination of the parent compounds and/or their active metabolites. Finally, depending on my findings, I would consider diuretics, hemodialysis, bronchodilators, inotropes, afterload reducing agents, and re-intubation. 2) The chest x-ray suggests bilateral pulmonary edema and his oxygen saturation is 84%. How might mechanical ventilation prove beneficial? Mechanical ventilation, and more specifically PEEP, may prove beneficial for patients with pulmonary edema, secondary to a positive-pressure- induced redistribution of alveolar fluid into areas that are less involved in gas exchange, which leads to improved oxygenation. However, in the presence of left ventricular dysfunction, positive ventilation and PEEP must be applied with caution to prevent worsening cardiac function secondary to decreased preload (positive intrathoracic pressure decreases blood return to the heart through the inferior vena cava). It is also important to avoid over-inflation of alveoli, which can result in barotrauma, increased capillary permeability, and potentially worsening pulmonary edema. 3) If you didn't place a pulmonary artery catheter for the case, would you place one now? Given the presence of significant pulmonary edema, potentially compromised cardiac function (normal cardiac function is required for CVP to accurately reflect left heart filling pressures), and the importance of maintaining adequate renal blood flow in this post-transplant patient, I would certainly consider placing a pulmonary artery catheter to aid in fluid management, assess cardiac function, and help differentiate cardiac and noncardiac causes of his pulmonary edema. 4) One hour postoperatively, the patient is oozing from the surgical incision. What do you think is going on? "what do you think is going on" means what's ddx and how would you differentiate these? The most common cause of post-operative coagulopathy is platelet dysfunction, which may result secondary to uremia (interferes with vWF formation and release), hypothermia (affects the morphology and function of platelets), drug effect, or severe anemia (with decreased viscosity, the platelets tend to travel in the center of the blood column, instead of near the endothelial surface). However, I would also consider other causes of coagulopathy such as residual heparin effect, inadequate surgical hemostasis, metabolic acidosis (interferes with the generation of thrombin), thrombocytopenia, and DIC. Therefore, I would check a platelet count, PT, PTT, INR, CBC, fibrinogen level, and fibrin degradation products. TEG analysis may also prove helpful, if available. 5) What is the cause of uremic thrombocytopathia? The mechanisms by which uremia contributes to thrombocytopathia include: 1) decreased von Willebrand factor (vWF) formation and release secondary to the accumulation of various metabolites, 2) increased synthesis of nitric oxide and prostacyclin, which have platelet inhibitory effects, and 3) uremia-induced anemia, which contributes to decreased viscosity with a subsequent decrease in platelet interaction with endothelial surfaces. Patients with chronic renal failure may also be more susceptible to coagulopathy because they tend to have reduced levels of factor V and the vitamin K-dependent factors (II, VII, IX, X). 6) How do you treat uremic thrombocytopathia? The treatment options for uremic thrombocytopathia include: 1) desmopressin (DDAVP) to increase the release of vWF; 2) erythropoietin, which may exert its affect by correcting anemia; 3) conjugated estrogens, which may reduce nitric oxide formation; 4) cryoprecipitate, which contains vWF; 5) platelets; and 6) hemodialysis to remove uremic acid. The fastest and most effective treatment is hemodialysis, which eliminates the uremic acid and quickly restores adequate platelet function. If hemodialysis were ineffective in correcting a clinically significant and life-threatening thrombocytopathic coagulopathy, I would consider a platelet transfusion. 7) In the first few hours after transplantation the urine output is only 10 cc/hr. Are you concerned? Would urine lab studies help with a diagnosis? I am concerned, because a urinary output of less than 20ml/hour represents significant oliguria in a post-kidney transplant patient. Potential causes of his oliguria include: hypovolemia, hypotension, acute graft rejection, renal artery thromboembolism, renal vein thrombosis, drug-induced constriction of the afferent arterioles (ACE inhibitors), acute tubular necrosis, mechanical obstruction of the graft vessels or ureter, and obstruction of the Foley catheter. Urine lab studies may help to differentiate the cause of oliguria. The most sensitive indices are fractional excretion of sodium and the renal failure index. The fractional excretion of sodium relates sodium clearance (Urinary Na+/Plasma Na+) to creatinine clearance (Urine Creatinine/Plasma Creatinine), with a fractional excretion of sodium less than 1% suggesting prerenal azotemia, and a value greater than 1% suggesting intrinsic renal failure. The presence or absence of urinary sediment and casts may provide additional information, with renal tubular cells, casts, and pigmented granular casts providing strong evidence of acute tubular necrosis.

1) Assume you are proceeding with general anesthesia. Is invasive monitoring indicated in this case? 2) How will you induce this patient? 3) Would sevoflurane be a good choice for a volatile agent? 4) How will you manage fluids during this case? Which fluids would you use for replacement? 5) Are there any drugs you would give prior to clamping of the iliac vessels? 6) After unclamping, his blood pressure falls to 61/40 mmHg. What do you think is going on? 7) You see peaked T waves on the EKG. What are you going to do? 8) What are the characteristic EKG changes associated with hyperkalemia? 9) How would you treat this hyperkalemic patient with associated electrocardiographic changes?

1) Assume you are proceeding with general anesthesia. Is invasive monitoring indicated in this case? This patient's uncertain volume status and poorly controlled preoperative hypertension place him at increased risk of hemodynamic instability during induction, laryngoscopy, and unclamping of the iliac vessels following graft placement. Therefore, I believe placing a CVP catheter and arterial line would be beneficial in rapidly identifying and treating hemodynamic changes that place the patient at increased risk for complications such as myocardial ischemia/infarction, cerebrovascular events, and delayed or compromised graft function. Additionally, optimization of the patient's intravascular volume may reduce the incidence of postoperative acute tubular necrosis and promote early onset of graft function; CVP monitoring may help to facilitate this goal while, at the same time, avoiding fluid overload. Finally, if additional history or the physical exam identified congestive heart failure, significant coronary artery disease, valvular heart disease, left ventricular dysfunction, or severe chronic obstructive pulmonary disease, I would consider placing a pulmonary artery catheter. 2) How will you induce this patient? My goal during induction is to safely secure the airway while avoiding hypotension or hypertension in this patient with poorly controlled hypertension, who is potentially volume contracted and more likely to exhibit an exaggerated response to laryngoscopy. Therefore, as long as there was not significant risk of aspiration (i.e. uremic gastroparesis), I would perform a slow controlled induction after placing the appropriate monitors, ensuring adequate beta-blockade and hydration, and administering intravenous lidocaine to further attenuate a sympathetic response to laryngoscopy. Following intubation, I would maintain general anesthesia using a volatile agent, a short acting opioid (consider using opioids without active metabolites that depend on renal excretion), and a muscle relaxant (mivacurium, atracurium, cisatracurium do not depend on renal excretion) in order to provide optimal operating conditions. 3) Would sevoflurane be a good choice for a volatile agent? Given the potential risk of nephrotoxicity with the use of sevoflurane, I don't believe it would be a good choice for kidney transplant surgery. The concern of nephrotoxicity stems from the fact that sevoflurane metabolism leads to the formation of potentially nephrotoxic levels of inorganic fluoride (>50 pmol/L), and that degradation by sodium or barium hydroxide (barium hydroxide > soda lime) can lead to the production of compound A, which has been associated with renal toxicity in animals. However, despite these theoretical or potential risks, sevoflurane has been successfully used for kidney transplant surgery. If using sevoflurane became necessary for some reason, I would minimize the length of patient exposure to the agent, ensure gas flow rates of at least 2 L per minute (>1 L per minute is ok for cases lasting less than 1 hour), and make sure my carbon dioxide absorbent is not desiccated (consideration could also be given to using Amsorb, which contains calcium hydroxide & calcium chloride, and results in less degradation of volatile agents). 4) How will you manage fluids during this case? Which fluids would you use for replacement? I would use central venous pressure monitoring (or the PAC, if present) and vigilance to optimize the patient's volume status and maintain renal blood flow, which is important in reducing the incidence of post-operative acute tubular necrosis and promoting early onset of graft function. Assuming the patient was normovolemic at the start of the case, I would replace blood loss with an isotonic solution, such as normal saline or Plasma-Lyte, at a ratio of 3:1, avoiding potassium containing solutions, such as Ringer lactate, that could exacerbate intraoperative hyperkalemia. If the administration of blood became necessary, I would transfuse cytomegalovirus-negative packed red blood cells. 5) Are there any drugs you would give prior to clamping of the iliac vessels? Usually heparin is administered prior to clamping the iliac vessels to prevent clotting. Just prior to revascularization, a calcium channel blocker (verapamil) or papaverine is often injected into the arterial circulation of the graft to prevent arterial vasospasm and/or reperfusion injury. Finally, mannitol and/or furosemide are given following reperfusion to induce diuresis. Mannitol has the added advantage of acting as a free radical scavenger. 6) After unclamping, his blood pressure falls to 61/40 mmHg. What do you think is going on? Hypotension immediately following unclamping is most likely secondary to the washout of vasoactive substances from previously ischemic tissues and the acute increase of up to 300 ml to the intravascular space. However, I would also consider 1) congestive heart failure, secondary to fluid overload or cardiac ischemia; 2) dysrhythmia, secondary to cardiac ischemia or acute hyperkalemia; 3) anaphylaxis, secondary to antibiotics, thymoglobulin, etc.; 4) blood loss; 5) tension pneumothorax (possibly secondary to central line placement); 6) anesthetic overdose; 7) hypoxia; and 8) monitor error. In response, I would ensure adequate oxygenation and ventilation; auscultate the lungs; check the EKG for dysrhythmia; check the CVP and PAC pressures (if available); consider transesophageal echocardiography, if the etiology of the hypotension remains unclear; inspect the surgical field for bleeding; check serum electrolytes (i.e. hyperkalemia, hyponatremia); and prepare to treat with fluids, inotropes, and vasopressors, as indicated. 7) You see peaked T waves on the EKG. What are you going to do? Peaked T waves on the EKG are consistent with significant hyperkalemia, which may occur with perfusion of the newly transplanted kidney and subsequent washout of potassium-containing preservative solution into the systemic circulation. Therefore, I would order a plasma potassium level, notify the surgeon, and initiate treatment, with the goals of: 1) eliminating any factors that may exacerbate hyperkalemic cardiotoxicity, such as acidosis, hyponatremia, and hypocalcemia (all of which occur in the presence of impaired renal function); 2) depressing cardiac membrane hyperexcitability; and 3) reducing extracellular fluid potassium, by either shifting it into cells or removing it from the body. I would also ensure immediate access to a cardiac defibrillator and prepare to treat cardiovascular dysrhythmias and instability as necessary. 8) What are the characteristic EKG changes associated with hyperkalemia? EKG signs associated with hyperkalemia, (usually when plasma potassium levels exceed 6 mEq/L) characteristically progress from peaked T waves, to prolongation of the PR interval, to decreasing amplitude, to loss of the P wave, to widening of the QRS complex (representing the loss of atrial activity), to a sine wave pattern, and finally, to ventricular fibrillation and asystole (usually occurring at levels around 8-9 mEq/L). 9) How would you treat this hyperkalemic patient with associated electrocardiographic changes? Given the presence of EKG changes, I would: 1) give calcium chloride or calcium gluconate to depress cardiac membrane excitability (immediate affect); 2) ensure immediate access to a cardiac defibrillator and prepare to treat cardiovascular dysrhythmias and instability as necessary; 3) attempt to correct any factors that may exacerbate hyperkalemic cardiotoxicity, such as acidosis, hyponatremia, and hypocalcemia; 4) attempt to shift extracellular potassium into cells by administering insulin and glucose (effective in 10-20 minutes), beta-adrenergic drugs (albuterol, salbutamol), and possibly sodium bicarbonate (although the latter is not considered very affective); 5) hyperventilate the patient to promote alkalosis and subsequent intracellular movement of potassium; and 6) consider hemodialysis, if temporizing measures are insufficient.

1) How do you plan to extubate this patient? 2) Would your plan for extubation change if the CSF space had been opened intra-operatively? 3) After surgery, the PACU nurse calls to inform you that the patient is obtunded and requiring 10 liters of oxygen per facemask to maintain a SpO2 above 90%. Are you concerned? What do you think is going on? 4) Assuming this patient had significant OSA, how would you manage him postoperatively? 5) On the second postoperative day, the nurse reports over two liters of urine output in the last hour. What do you think is the cause? 6) How would you treat this patient? 7) On the fourth postoperative day, the patient gradually develops hypotension that is refractory to fluid boluses and vasopressors. What do you suspect is going on?

1) How do you plan to extubate this patient? This patient with acromegaly, obesity, and obstructive sleep apnea, remains at risk for difficult airway management. Given these concerns and the fact that his GERD increases his risk of aspiration during emergence, I would place the patient in the semi-upright position, provide 100% oxygen, and extubate him only after he was awake and demonstrating full neuromuscular block reversal (especially with history of obstructive sleep apnea), adequate ventilation and oxygenation, hemodynamic stability, and intact airway reflexes. If I were concerned about the presence of CAD, I would have administered lidocaine prior to extubation to blunt a potential sympathetic response to extubation. 2) Would your plan for extubation change if the CSF space had been opened intra-operatively? If the CSF space had been entered intraoperatively a smooth emergence (avoiding coughing) is desirable to prevent the reopening of the CSF leak, which would increase the risk of subsequent meningitis. Although coughing during emergence may increase a CSF leak, I would plan to extubate this patient awake to avoid airway obstruction, laryngospasm, aspiration, and hypoventilation. I would attempt to prevent coughing by administering intravenous lidocaine just prior to extubation. 3) After surgery, the PACU nurse calls to inform you that the patient is obtunded and requiring 10 liters of oxygen per facemask to maintain a SpO2 above 90%. Are you concerned? What do you think is going on? This is concerning to me because this obese patient with obstructive sleep apnea is at increased risk of post-operative airway obstruction and apnea, especially with the use of narcotics for post-operative pain management. Other potential causes or contributing factors include atelectasis (hypoventilation of this obese patient), pulmonary edema (secondary to congestive heart failure and/or air entering the pulmonary artery during VAE), aspiration (leading to bronchospasm or atelectasis), inadequate reversal of neuromuscular blockade, hypo/hyperglycemia (diabetic patient), electrolyte abnormalities, arrhythmia, myocardial infarction, or stroke (paradoxical emboli, ischemia). 4) Assuming this patient had significant OSA, how would you manage him postoperatively? Postoperatively, I would maintain the patient in a seated or lateral position (avoiding supine positioning) and provide supplemental oxygen until the patient was able to maintain his baseline oxygen saturation on room air and in a quiet peaceful environment. Additionally, I would apply his home CPAP machine as soon as possible; administer NSAIDs for analgesia with the goal of avoiding sedatives and narcotics, if possible; and utilize continuous pulse-oximetry until his oxygen saturation remained above 90% during sleep. I would continue to monitor this patient for airway obstruction, hypoxemia, dysrhythmias, and hypertension for a median of 3 hours longer than a non-OSA patient and for at least 7 hours after the last episode of airway obstruction or hypoxemia while breathing room air in a non-stimulating environment. Clinical Note: · The 2006 OSA guidelines suggest closely monitoring a patient for at least 7 hours following the last postoperative episode of apnea. The 2014 guidelines do not contradict this, but fail to re-address the issue. 5) On the second postoperative day, the nurse reports over two liters of urine output in the last hour. What do you think is the cause? This patient's postoperative polyuria could be due to central diabetes insipidus (DI), mobilization of third-spaced fluid, diuretic use, or osmotic diuresis from hyperglycemia. Therefore, I would review the fluids and medications administered; rule out hyperglycemia; and further evaluate the patient for DI, a condition that occurs in as many as 40% of patients following hypophysectomy (tests: urine specific gravity, serum osmolality, serum/urine electrolytes). DI is characterized by marked impairment in renal concentrating ability secondary to decreased ADH secretion, and usually occurs 4-12 hours postoperatively (but may develop intraoperatively). This condition is suspected when a patient produces copious amounts of urine despite rising serum sodium levels (increased serum osmolality). The diagnosis is confirmed when hyperglycemia is ruled out, the patient's urine specific gravity is < 1.005 (some sources say < 1.002), and the urine osmolality increases with the administration of exogenous ADH. 6) How would you treat this patient? The form of diabetes insipidus that occurs following transphenoidal hypophysectomy is almost always central diabetes insipidus (decreased ADH production) rather than nephrogenic diabetes insipidus (abnormal response of renal tubules to circulating ADH). Therefore, I would replace the urinary loss of hypo-osmolar, low sodium fluids with 1/2 normal saline (while D5W is sometimes used in this situation, I would avoid it in this diabetic patient) at a rate equal to hourly maintenance requirements plus two thirds of the previous hour's urine output (alternatively, replacement could be based on maintenance requirements plus the previous hour's urine output less 50 mL). If the hourly fluid requirements exceeded 350 mL, I would administer aqueous vasopressin or desmopressin (DDAVP) to replace the patient's ADH. 7) On the fourth postoperative day, the patient gradually develops hypotension that is refractory to fluid boluses and vasopressors. What do you suspect is going on? This patient's refractory hypotension is concerning because it may be associated with adrenal insufficiency secondary to panhypopituitarism or pituitary apoplexy, a known complication of surgical hypophysectomy. This diagnosis could be confirmed by identifying decreased cortisol and ACTH plasma concentrations, and would require specific hormone replacement therapy, including glucocorticoids, mineralocorticoids, and thyroid hormone.

1) Postoperatively, he is extubated in the ICU and immediately becomes dyspneic. What do you think may be going on? 2) You visualize the vocal cords and note that they are not moving and are in the midline position. What will you do?

1) Postoperatively, he is extubated in the ICU and immediately becomes dyspneic. What do you think may be going on? His stridor could be caused by: (1) laryngospasm, occurring secondary to superior vena cava syndrome or recent extubation. It could also be a result of: (2) an expanding hematoma, (3) significant airway edema (secondary to superior vena cava syndrome and/or aggressive fluid resuscitation), (4) recurrent laryngeal nerve injury (resulting in obstruction due to unopposed tension of the vocal cords by the cricothyroid muscle), (5) incomplete muscle relaxant reversal (patients with LES are more sensitive to nondepolarizers and reversal of neuromuscular blockade is often less effective), (6) an impaired ventilatory response to hypoxia and hypercapnia secondary to autonomic neuropathy (these patients are more susceptible to drug- induced respiratory depression), or (7) an allergic reaction. Clinical Note: If worsening SVC obstruction is suspected during a case, steroids and diuretics may help. 2) You visualize the vocal cords and note that they are not moving and are in the midline position. What will you do? This is consistent with bilateral partial recurrent laryngeal nerve injury (injury affecting only the abductor fibers of the nerve), a known complication associated with mediastinoscopy. If the airway obstruction were resulting in inadequate ventilation, I would place an endotracheal tube and provide oxygen, positive pressure ventilation, and sedation as necessary. I would then alert the surgeon and consult an otolaryngologist. Clinical Note: Complications associated with mediastinoscopy include: (1) tracheal compression or laceration, (2) cerebrovascular events (secondary to innominate artery compression), (3) right upper limb ischemia (also secondary to innominate artery compression), (4)compression of the aorta leading to reflex bradycardia, (5) pneumothorax, (6) recurrent laryngeal or phrenic nerve injury, (7) venous air embolism, (8) mediastinal hemorrhage, and (9) esophageal tear. 3) You go by the ICU to see the patient later that evening and are just getting ready to leave when his pressure acutely drops from 140/88 mmHg to 75/38 mmHg. What are you going to do? I would (1) auscultate the chest, (2) check the ventilator settings, (3) ensure adequate oxygenation, (4) place the patient in trendelenburg position, and (5) administer fluids, vasoconstrictors, and inotropes as indicated. Since the patient suffered a tear of the superior vena cava during the procedure, my differential would include massive hemorrhage or cardiac tamponade. While attempting to identify the cause of his hemodynamic instability, I would (6) ensure adequate intravenous access, (7) order an echocardiogram, (8) call for a surgeon, and (9) consider placing a central line. 4) On exam you hear muffled heart sounds and notice jugular venous distension. The blood pressure is now 52/31 mmHg. What are you going to do? While the jugular venous distention was already present due to the obstruction of his superior vena cave, the combination of this finding along with muffled heart sounds is consistent with cardiac tamponade and, if the diagnosis cannot be quickly confirmed by echo, should be treated as such. Since the patient is intubated and sedated, the chest could be reopened in the intensive care unit, waiting to provide additional anesthesia until the pericardial constriction is relieved and the patient is more stable. If the surgeon is not available, a pericardiocentesis should be performed to at least temporarily relieve the increasing pericardial pressure, improve diastolic filling, and improve cardiac output. Once the chest is opened, the tamponade is relieved, and the patient is stable, I would administer anesthetic agents that preserve sympathetic tone (e.g. ketamine) while definitive surgical repair occurs.

1) The child's blood pressure is elevated post-operatively. What do you think may be going on? 2) What does "neutral temperature" mean as it relates to the neonate? 3) How do infants maintain heat? 4) How will you maintain normothermia in this infant? 5) The post-op nurse grabs you as you are leaving the NICU and says she thinks the infant is having a seizure. What are some causes of neonatal seizure? 6) What will you do?

1) The child's blood pressure is elevated post-operatively. What do you think may be going on? While ligation of the ductus arteriosus is often associated with post operative systemic hypertension (the mean arterial pressure and diastolic pressure increase due to the elimination of pulmonary runoff during diastole), I would also consider other potential causes, such as inaccurate measurement (i.e. an undersized blood pressure cuff), inadequate pain control, agitation, hypervolemia, hypercarbia, hypoxemia, bladder distention, and the administration of exogenous medications. Other less likely causes would include undiagnosed coarctation of the aorta and increased intracranial pressure (there is a higher risk of intracranial bleed in this extremely low birth weight, premature neonate). If his hypertension did not resolve after providing sufficient pain control, draining the bladder, and ensuring adequate oxygenation and ventilation, 1 would consider administrating an antihypertensive medication, such as nitroprusside (other choices may include hydralazine, beta-blockers, calcium channel blockers, and a-adrenergic blockers). 2) What does "neutral temperature" mean as it relates to the neonate? The neutral temperature is the ambient temperature at which oxygen consumption is minimized. Avoiding the compensatory responses to hypothermia, such as increased oxygen utilization, increased glucose consumption, and the resultant acidosis, is extremely important in sick infants. The neutral temperature is about 34°C for a preterm neonate, 32°C for a term neonate, and 28°C for adults. 3) How do infants maintain heat? The primary mechanism of heat generation in the infant is nonshivering thermogenesis. Hypothermia-induced release of norepinephrine initiates the metabolism of brown adipose tissue, which results in increased oxygen consumption and heat production. Unfortunately, this already inefficient process is severely limited in premature and sick infants who are deficient in brown fat stores. Therefore, it is especially important to take the necessary steps to maintain normothermia in premature infants during the perioperative period. 4) How will you maintain normothermia in this infant? Assuming the case was going to be performed in the OR, I would maintain normothermia by ensuring the ambient temperature in the OR was 26-30°C and by utilizing infrared heating lamps, forced air warmers, warmed intravenous solutions, and heated and humidified anesthetic gasses. I would also use a heated transport incubator for transport to and from the OR. 5) The post-op nurse grabs you as you are leaving the NICU and says she thinks the infant is having a seizure. What are some causes of neonatal seizure? The differential for neonatal seizure is extensive and includes (1) intracranial hemorrhage (a relatively higher risk in this extremely premature neonate), (2) hypoxic-ischemic encephalopathy, (3) cerebral edema, (4) hypoglycemia, (5) hypocalcemia, (6) hypomagnesemia, (7) benign seizures, (8) obstetric history of TORCH (toxoplasmosis, rubella, cytomegalovirus, herpes), and (9) sepsis. 6) What will you do? I would immediately (1) check endotracheal tube placement, (2) auscultate the chest, and (3) review the monitors and ventilator settings to verify adequate ventilation and oxygenation. I would then (4) check the ECG and blood pressure to assess cardiovascular stability; (5) administer a small amount of barbiturate or benzodiazepine to stop the seizure; (6) order electrolytes; (7) notify the neonatologist; and (8) consider consulting a neurologist.

1) What are your positioning concerns? 2) You just get the patient positioned when the resident, who was filling one of the vaporizers, spills an entire bottle of volatile agent on the floor. What would you do? 3) Postoperatively, an ENT surgeon is consulted to evaluate the patient's facial injuries. He tells you he would like to schedule surgery in the next couple of weeks to address the injuries. What would you say? 1) Six months later this same patient presents to your operating room for repair of an open compound fracture of his right femur. He is quadriplegic, but able to maintain adequate respiratory function without support. His medications include baclofen, plavix, lipitor, advair, and albuterol. What would you consider in your preoperative workup? 2) Due to his spinal cord injury, the man does not experience pain in his lower extremities. Knowing this, the surgeon requests mild sedation for the case. Would you agree to this request? 3) The patient requests spinal anesthesia. Would you agree? 4) You discover that the patient has yet to discontinue his Plavix, making a general anesthetic necessary. Would you perform rapid sequence intubation using succinylcholine? 5) The case has just started, when you are informed that the central oxygen supply is compromised. What would you do? 6) You are using oxygen cylinders for the case. Two hours into the procedure, the patient's blood pressure increases to 205/106 mmHg, and his heart rate decreases to 42 beats per minute. What would you do? 7) You discover that the Foley catheter had been obstructed. What is the pathophysiology of autonomic hyperreflexia (AH)?

1) What are your positioning concerns? I have several concerns about this patient with head and neck injury being in the prone position, including: (1) additional spinal cord injury, secondary to inadequate neck stabilization during transport and initial positioning; (2) pressure-induced injury to the eyes, ears, nose, breasts, genitals, knees, and toes; (3) position- related obstruction of venous drainage leading to increased intracranial and intraocular pressure, placing him at increased risk for cerebral ischemia and post operative visual loss; and (4) brachial plexus injury secondary to compression of the axillary sheath or abduction of the arms by more than 90 degrees. Clinical Note: The prolonged immobility associated with chronic spinal cord injury can lead to osteoporosis, increasing the risk for pathologic fractures when moving or positioning the patient (not yet a concern for this patient with acute spinal cord injury). 2) You just get the patient positioned when the resident, who was filling one of the vaporizers, spills an entire bottle of volatile agent on the floor. What would you do? Given the size of the spill and the risk of significant exposure of the operating room personnel to evaporating volatile agent (the patient is intubated and isolated from the evaporating agent), I would immediately (1) have someone suction the spilled volatile agent into a plastic container, (2) seal and label the container, and (3) transport it to the appropriate waste disposal site (i.e. waste disposal company or medical waste incinerator). At the same time, I would (4) verify the patient was securely positioned on the bed, (5) ensure adequate sedation and neuromuscular blockade, and (6) prepare him for transport on the operating room bed to the nearest anesthetizing location (i.e. 100% oxygen, rescue drugs, appropriate monitoring). 3) Postoperatively, an ENT surgeon is consulted to evaluate the patient's facial injuries. He tells you he would like to schedule surgery in the next couple of weeks to address the injuries. What would you say? I would begin by discussing the urgency of the procedure with the ENT surgeon; assuming this were an elective procedure that could potentially be postponed, there would be several issues that should be considered, such as the type of coronary stent that was placed six months ago, his hemodynamic stability (is he still in spinal shock?), and the risk of autonomic hyperreflexia (assuming spinal shock had resolved). In the case of persistent spinal shock, any elective procedure should be postponed until the resolution of this condition and its' associated hemodynamic instability. If it were determined that he had a drug eluting stent (as suggested by the fact that he is still taking Plavix), then he should not be scheduled for any elective surgery that required the discontinuation of his clopidogrel (Plavix) until at least 365 days following stent placement (However, with newer generation drug-eluting stents, the risk of stent thrombosis stabilizes at 6 months following stent placement. Therefore, elective noncardiac surgery that does not require the discontinuation of dual antiplatelet therapy may be considered after 180 days if the risk of further delay is considered to be greater than the risks of ischemia and/or stent thrombosis. Class lib recommendation.) Undergoing a procedure that required the discontinuation of his clopidogrel would place him at considerable risk because, following the placement of a drug eluting stent, the incidence of late stent thrombosis is significantly increased with the premature discontinuation of thienopyridine therapy within the first year. Moreover, the inherent hypercoagulable state associated with surgery places the patient at additional risk of thrombosis. Clinical Note: Elective procedures should be delayed for 14 days following balloon angioplasty to allow for complete healing of any vessel injury. When a stent is placed, elective procedures for which there is significant risk of bleeding should be delayed for at least 30 days following bare metal stent placement (BMS) and for 365 days following drug eluting stent (DES) placement. (ACC/AHA Guidelines - 2014). Post-operative Management 1) Six months later this same patient presents to your operating room for repair of an open compound fracture of his right femur. He is quadriplegic, but able to maintain adequate respiratory function without support. His medications include baclofen, plavix, lipitor, advair, and albuterol. What would you consider in your preoperative workup? In addition to evaluating and optimizing his coronary artery disease and asthmatic condition, I would ask about recent food ingestion (emergent case), ensure the availability of short acting vasodilators, make sure his baclofen therapy is continued throughout the perioperative period (abrupt cessation can lead to seizures), provide aspiration prophylaxis (high spinal cord lesion, GERD, and possible recent food ingestion), and carefully evaluate the patient for the various complications associated with cervical spinal cord injury, such as: (1) autonomic hyperreflexia, which occurs with the return of spinal cord reflexes below the level of injury (occurs with spinal cord lesions above the level of T7); (2) pulmonary dysfunction, secondary to impaired diaphragmatic function (denervation and/or elevation of the diaphragm secondary to fecal impaction), chronic pulmonary infections (impaired ability to cough and clear secretions), and denervation of the intercostal and abdominal muscles; (3) renal dysfunction, secondary to recurrent calculi, amyloid deposition, and chronic urinary tract infections (inability to completely empty the bladder); (4)altered thermoregulation (increased susceptibility to hypothermia due to chronic vasodilation and absence of normal reflex cutaneous vasoconstriction); (5) anemia; (6) cardiac conduction abnormalities, secondary to autonomic dysfunction and/or electrolyte abnormalities; (7) decreased neck range of motion, which may affect airway management; (8) additional bone fractures, (these patients often develop osteoporosis secondary to prolonged immobility); and (9) decubitus ulcers and deep venous thrombosis, secondary to immobility. To this end, I would review the most current neck films, perform a careful history and physical exam, and order an EKG, CXR, pulmonary function tests, blood urea nitrogen, creatinine, arterial blood gasses, hematocrit, and electrolytes. 2) Due to his spinal cord injury, the man does not experience pain in his lower extremities. Knowing this, the surgeon requests mild sedation for the case. Would you agree to this request? I would not agree to sedation for this case due to the risks of pulmonary aspiration and autonomic hyperreflexia. The gastrointestinal paralysis associated with high spinal cord injury, combined with possible recent food ingestion (emergent case), place this patient at increased risk for pulmonary aspiration. Moreover, assuming the resolution of spinal shock and the return of spinal reflexes, this patient would be at increased risk for autonomic hyperreflexia with visceral distention (i.e. bladder) and/or any noxious stimuli below the level of injury, despite the lack of pain perception. Therefore, assuming there were no contraindications, I would provide neuraxial, regional, or general anesthesia for this case in order to minimize the risk of aspiration and autonomic hyperreflexia (the latter is best prevented by spinal, regional, or general anesthesia). 3) The patient requests spinal anesthesia. Would you agree? I would agree to spinal anesthesia, recognizing that both spinal anesthesia and deep general anesthesia suppress the unmodulated reflex sympathetic stimulation that can lead to autonomic hyperreflexia. Furthermore, neuraxial anesthesia would provide adequate analgesia, while avoiding the need for airway instrumentation of this patient with moderate asthma who is at risk for aspiration. However, recognizing that the placement of a neuraxial block in a patient taking clopidogrel would risk spinal hematoma formation, I would ensure that he had discontinued his Plavix at least 7 days prior to instrumentation (antiplatelet therapy is often continued for a year following the placement of a drug-eluting stent). 4) You discover that the patient has yet to discontinue his Plavix, making a general anesthetic necessary. Would you perform rapid sequence intubation using succinylcholine? While the risk of succinylcholine-induced hyperkalemia is reduced by 6 months following spinal cord injury (the highest risk is between 4 weeks and 5 months), 1 would prefer to avoid using this depolarizing muscle relaxant anytime between 24 hours and one year following the injury. Moreover, given his moderate asthma, I would be concerned that the unreliable depth of anesthesia associated with a rapid sequence induction would place him at increased risk for bronchospasm. However, recognizing this patient is at increased risk for pulmonary aspiration (high spinal cord injury gastrointestinal paralysis -> delayed gastric emptying; emergent case possible recent food ingestion), I would take precautions to reduce this risk as much as possible. Therefore, I would: (1) optimize his asthmatic condition by providing a beta-agonist; (2) administer an H2-receptor antagonist and a non-particulate antacid; (3) place him in reverse-trendelenburg position, to facilitate intubation and reduce the risk of passive regurgitation; (4) apply cricoid pressure; (5) perform a slow controlled induction, with the goal of achieving an adequate depth of anesthesia to prevent bronchospasm; (6) administer Rocuronium to provide optimal intubating conditions; (7) secure his airway with an appropriately sized endotracheal tube; and (8) decompress his stomach with a nasogastric tube following intubation. 5) The case has just started, when you are informed that the central oxygen supply is compromised. What would you do? Given the risk of administering a hypoxic gas mixture, I would immediately switch to the backup oxygen cylinders, disconnect the main pipeline supply, hand ventilate with low gas flow rates (Pneumatically driven mechanical ventilators use significantly more oxygen than hand ventilation at low gas flow rates. Many newer machines use air rather than oxygen for pneumatic power, making this step unnecessary.), calculate the approximate amount of time before the oxygen cylinders are depleted, and have someone locate additional oxygen E-cylinders in case they are required. It is important to disconnect the main pipeline supply to prevent the machine from preferentially using the main pipeline supply (50 psig), rather than the lower-pressure (45 psig) backup oxygen cylinder. Clinical Note: Method of Estimating Oxygen E-Cylinder Supply Time Approximate Remaining Time (hours): Oxygen Cylinder Pressure (psig) 200 X Oxygen Flow Rate (L/min) 6) You are using oxygen cylinders for the case. Two hours into the procedure, the patient's blood pressure increases to 205/106 mmHg, and his heart rate decreases to 42 beats per minute. What would you do? Recognizing that the development of hypertension and bradycardia in this paraplegic patient with a high spinal cord lesion is consistent with autonomic hyperreflexia (AH), and that bladder distention or some other stimulus below the level of the spinal cord transection is the likely cause of his hypertension and bradycardia, I would: (1) ask the surgeon discontinue any unnecessary stimulation, (2) deepen the anesthetic, (3) administer a direct acting vasodilator (i.e. sodium nitroprusside), (4) ensure that the bladder is emptying appropriately (i.e. check the Foley catheter and urine output), (5) place an arterial line for continuous blood pressure monitoring, and (6) monitor the patient closely for subsequent complications, such as cerebral, retinal, or subarachnoid hemorrhage; seizures; myocardial ischemia; dysrhythmias; and pulmonary edema (left ventricular failure due to increased afterload). 7) You discover that the Foley catheter had been obstructed. What is the pathophysiology of autonomic hyperreflexia (AH)? Cutaneous (pain) or visceral (i.e. bladder or rectal distention) stimulus below the level of spinal cord injury results in a reflex sympathetic discharge. Because the area of the body below the transaction is neurologically isolated, the sympathetic activity in this area is not modulated by inhibitory impulses from higher central nervous system centers. This unopposed sympathetic activity leads to vasoconstriction below the level of the lesion, with reflex vasodilation above the level of the lesion. When the lesion is above T7, vasodilation above the lesion is insufficient to prevent systemic hypertension, which then stimulates carotid sinus receptors, leading to reflex bradycardia.

A 34-year-old, 98 Kg, male presents for transphenoidal resection of a pituitary adenoma in the sitting position. He complains of progressive headache, blurred vision, and rhinorrhea over the last 3 months. Past medical history is significant for hypertension, GERD, obstructive sleep apnea (OSA), and diet controlled diabetes mellitus. His medications include; propranolol, hydrochlorothiazide, omeprazole, octreotide, and bromocriptine. Vital Signs: HR = 54, BP = 140/62 mmHg, RR = 18, Temp = 37.1 °C, Hgb =12.1 gm/dL. 1) What general concerns do you have for this case? 2) What do you think of this patient's complaints? 3) What is the physiologic function of the pituitary gland? 4) Why is this patient taking Bromocriptine? Octreotide? What is the mechanism of these drugs? 5) How is the diagnosis of acromegaly made? 6) What general anesthetic concerns would you have in someone with Acromegaly? 7) Would you require an echocardiogram prior to starting the case?

1) What general concerns do you have for this case? In general, my concerns for this case include the following: 1) hemodynamic lability and risk of end organ ischemia associated with chronic hypertension; 2) the risk of cerebral ischemia secondary to a reset autoregulation curve, hemodynamic lability, air embolism, and/or increased ICP with suprasellar extension of the tumor; 3) the increased risk of aspiration associated with GERD and bromocriptine (may cause gastroparesis); 4) the potential for difficult airway management associated with acromegaly (the patient is taking bromocriptine and octreotide), obesity, and OSA; 5) the potential for endocrinologic conditions, such as Cushing's disease, panhypopituitarism, acromegaly, hyperthyroidism, and diabetes insipidus; and 6) the risk of procedure-related complications, such as massive hemorrhage, air embolism, arrhythmias, high spinal, and cranial nerve damage. 2) What do you think of this patient's complaints? This patient's complaints of headache, blurred vision, and rhinorrhea are consistent with parasellar extension of the pituitary tumor, with expansion of the sella (headache), compressing the optic chiasm (blurred vision), and inferior extension of the adenoma (rhinorrhea). Compression of adjacent structures is usually associated with larger, nonfunctional macro-adenomas that are greater than 1 cm in size. However, since this patient is taking bromocriptine and octreotide, I would suspect that his pituitary tumor is functional, with the excessive secretion of growth hormone leading to acromegaly. Because hormone-secreting lesions cause noticeable systemic affects, they are often diagnosed earlier, prior to the onset of the compression-related signs and symptoms associated with larger, non functioning tumors (i.e. nausea, vomiting, headache, visual disturbances, rhinorrhea, apoplexy (Apoplexy is rupture of an internal organ and the accompanying symptoms. The term formerly referred to what is now called a stroke), panhypopituitarism, and hydrocephalus). The most common functional pituitary adenoma is a prolactinoma, which is associated with amenorrhea, galactorrhea, and infertility. Other pituitary adenomas secrete excessive amounts of ACTH, leading to Cushing's disease (truncal obesity, abdominal striae, hypertension, and hyperglycemia); growth hormone, leading to acromegaly (HTN, insulin resistance, visceromegaly, osteoporosis, skeletal overgrowth, and soft-tissue overgrowth); and TSH, leading to hyperthyroidism (this is rare). 3) What is the physiologic function of the pituitary gland? The anterior pituitary is responsible for the synthesis, storage, and secretion of the six following tropic hormones: 1) adrenocorticotrophic hormone (ACTH), which stimulates the adrenal cortex secretion; 2) prolactin, which stimulates the secretion of breast milk and inhibits ovulation; 3) human growth hormone, responsible for body growth; 4) thyroid-stimulating hormone (TSH), which stimulates thyroid secretion and growth; 5) follicle-stimulating hormone (FSH), responsible for ovarian follicle growth in females and spermatogenesis in males; and 6) luteinizing hormone, which stimulates ovulation in females and testosterone secretion in males. The anterior pituitary also secretes beta-lipotropin, which contains the amino acid sequences of several endorphins that bind to opioid receptors. The posterior pituitary stores and secretes two hormones that are initially synthesized in the hypothalamus and transported to the posterior pituitary. These two hormones are (1) antidiuretic hormone (ADH), which promotes water retention and regulates plasma osmolarity, and (2) oxytocin, which causes uterine contraction and the ejection of breast milk. 4) Why is this patient taking Bromocriptine? Octreotide? What is the mechanism of these drugs? Bromocriptine is used to treat the excessive excretion of both prolactin and growth hormone from functional pituitary tumors. It is a synthetic dopamine-2 receptor agonist that inhibits the secretion of both growth hormone and prolactin, bringing their levels down sufficiently to improve symptoms in many patients. Since this patient is also being treated with Octreotide, a drug not often utilized in the treatment of a functional prolactinoma, I suspect that he is receiving these medications to treat acromegaly. Octreotide is a somatostatin analogue that inhibits the release of growth hormone and may actually shrink the size of pituitary tumors. 5) How is the diagnosis of acromegaly made? The diagnosis of acromegaly is based on an initial clinical suspicion, due to the presence of several characteristic manifestations, and confirmed by biochemical testing. The characteristic manifestations of acromegaly include skeletal overgrowth (large body, hands, and feet; prognathism), soft tissue overgrowth (large lips, tongue, epiglottis, and vocal cords), recurrent laryngeal nerve paralysis (secondary to stretching caused by overgrowth of surrounding structures), peripheral neuropathy (secondary to trapping caused by the overgrowth of surrounding tissue), visceromegaly, glucose intolerance, osteoarthritis, osteoporosis, hyperhidrosis, and skeletal muscle weakness. Biochemical tests used to confirm the diagnosis include measurement of serum IGF-1 (the most reliable test since it is less variable throughout the day), measurement of serum growth hormone (varies from hour-to-hour with exercise, sleep, and food ingestion), and the performance of an oral glucose tolerance test (in acromegalic patients, the serum GH levels remain above 2 ng/mL within two hours of ingesting 75 g of glucose). 6) What general anesthetic concerns would you have in someone with Acromegaly? My primary concern in patients with acromegaly is the potential for difficult airway management. Distorted facial anatomy (difficult mask fit), enlargement of the tongue and epiglottis (predisposition to upper airway obstruction & impaired visualization during laryngoscopy), overgrowth of the mandible (prognathism), a narrowed glottis opening (may require smaller endotracheal tube), enlarged nasal turbinates (may inhibit nasal passage of an airway), and recurrent laryngeal nerve paralysis may all compromise airway management. Other than the airway, my concerns would include an increased incidence of coronary artery disease and the risks associated with hypertension, cardiomegaly, congestive heart failure, obstructive sleep apnea (mask ventilation, postoperative apnea, and inadequate pain control), diabetes mellitus, and peripheral neuropathy (positioning). 7) Would you require an echocardiogram prior to starting the case? Given the increased risk of venous air embolism during intracranial procedures performed with the head above the level of the heart, I would prefer to perform a bubble study with precordial echocardiography or transesophageal echocardiography to rule out a patent foramen ovale, which would place him at risk for paradoxical embolism to the coronary or cerebral circulations (bubble studies could also be performed with a transcranial Doppler). If he had a patent foramen ovale, the sitting position is relatively contraindicated and I would discuss alternative patient positioning with the surgeon. In addition to identifying this risk, an echocardiogram may prove helpful in identifying any cardiac pathology resulting from his hypertension or obstructive sleep apnea.

1) What monitoring would you utilize for this case? 2) Your resident asks where you would like to monitor blood pressure and oxygen saturation. Does it matter? 3) An ABG shows a pH of 7.30, PaCO2 of 51, Bicarbonate of 25, and PaO2 of 110 with a FiO2 of 40%. What do you think about this information?

1) What monitoring would you utilize for this case? In addition to standard monitoring, such as an ECG, end-tidal CO2, inspiratory oxygen, inspiratory pressure, and oxygen saturation, I would utilize a precordial and/or esophageal stethoscope to aid in monitoring of cardiopulmonary status and an esophageal or axillary probe to monitor temperature. Blood pressure should be monitored in the right arm (preductal) since clamping of the left subclavian artery may become necessary should the ductus be torn causing massive bleeding. Oxygen saturation should be monitored on the right hand and a lower limb to provide pre-ductal and post-ductal readings respectively. While invasive blood pressure monitoring and a CVP line would be helpful if already present, I would not routinely require them for this procedure. 2) Your resident asks where you would like to monitor blood pressure and oxygen saturation. Does it matter? As I mentioned in my previous answer, blood pressure should be monitored in the right arm since clamping of the left subclavian artery may become necessary should the ductus be torn causing massive bleeding. Oxygen saturation should be monitored on the right hand and a lower limb to provide pre-ductal and post-ductal readings respectively. Pre-ductal and post-ductal saturation monitoring can provide information about shunting (right-to-left shunting through a PDA results in a relatively lower post-ductal oxygen saturation) and help the surgeon to avoid accidental ligation of the aorta (would result in post-ductal loss of waveform), or pulmonary artery (would result in both decreased pre-ductal and post-ductal oxygen saturation along with decreased end-tidal CO2) 3) An ABG shows a pH of 7.30, PaCO2 of 51, Bicarbonate of 25, and PaO2 of 110 with a FiO2 of 40%. What do you think about this information? This ABG indicates a respiratory acidosis and a PaO2 that is higher than optimal given the risk of retinopathy of prematurity (ROP) in a premature infant such as this one (very premature infants weighing < 1000 g are at the highest risk of developing ROP). Consideration should be given to increasing ventilation to correct the acidosis, and reducing the FiO2 to maintain a PaO2 of 50-80 mmHg or an oxygen saturation of 87-94% to minimize the risk of ROP. 4) Are you concerned about retinopathy of prematurity (ROP) in this patient? I am concerned given this neonate's high number of risk factors for developing ROP, such as prematurity (especially < 32 weeks gestation), low birth weight (under 1,500 grams), cyanotic congenital heart disease, mechanical ventilation, respiratory distress, hypoxia, and acidosis. Therefore, I would be careful to maintain an oxygen saturation of 87-94% (or a PaO2 of about 50-80 mmHg) and avoid anemia, acidosis, hypotension, and major fluctuations in oxygen or carbon dioxide levels. 5) What are the risk factors for ROP? The etiology of ROP is multifactorial with risk factors including: (1) hyperoxia, (2) prematurity (probably the most important risk factor; especially < 32 weeks and a birth weight of < 1000 g), (3) carbon dioxide fluctuations, (4) hypotension, (5) sepsis, (6) red blood cell transfusions, (7) cyanotic congenital heart disease, (8) respiratory distress syndrome, (9) intraventricular hemorrhage, (10) corticosteroid therapy, (11) mechanical ventilation, (12) hyperglycemia, (13) maternal diabetes, (14) hypoxemia, (15) fluctuations in oxygen levels, (16) exposure to bright light, and (17) maternal antihistamine use within 2 weeks of delivery. While hyperoxia has long been considered a major component of this neonatal disorder, ROP has been documented to occur in the absence of oxygen supplementation. Moreover, major fluctuations in oxygen saturation may be a greater risk factor for ROP than hyperoxia. 6) Are you concerned about minimizing FiO2 in this sick infant with a heart that is volume overloaded? I am concerned about compromising oxygen delivery to the heart of this infant, and the benefits and risks of reducing his oxygen saturation would have to be considered before taking any action. However, reducing the PaO2 to 50-80 mmHg is unlikely to compromise oxygen delivery to the heart if anemia is avoided and oxygen consumption is minimized. Moreover, lowering oxygen saturation may serve to reduce the work of the heart by increasing hypoxic pulmonary vasoconstriction in the lungs and subsequently leading to a reduction of the left to right shunt that is causing pulmonary vascular congestion and volume overload of the heart. However, I would not allow concerns about causing retinopathy of prematurity to prevent me from increasing the inspired oxygen concentration if I believed it necessary due to the neonate's cardiopulmonary status. 7) How would you maintain anesthesia for this case? I would prefer to maintain anesthesia with fentanyl, supplementing with nitrous oxide or ketamine, as necessary. Since the infant is not likely to be extubated at the end of the case, I would select pancuronium for muscle relaxation as the mild tachycardia it induces may be beneficial in maintaining the infant's blood pressure. The decrease in systemic vascular resistance that accompanies the use of volatile agents may prove beneficial by reducing left-to-right shunting through the PDA. However, these often sick and hypovolemic infants may not tolerate their use since they are particularly sensitive to the cardiovascular depression associated with volatile agents. 8) How would you monitor blood loss? Blood loss must be monitored very carefully because this neonate's entire blood volume is only about 98 mL (100 mL/kg). Therefore, I would weigh all sponges, laps, etc. to estimate blood loss as precisely as possible and replace with crystalloid (3:1 ratio), colloid, or packed red blood cells as indicated. Due to the risks associated with transfusion, I would normally avoid the administration of blood products in a healthy full term infant until the hematocrit was below 20-25%. However, for this sick premature neonate, with decreased cardiac reserve (i.e. PDA, left atrial enlargement, and abnormal blood flow in the pulmonary artery) and leftward shifting of the oxyhemoglobin dissociation curve (high concentration of fetal hemoglobin), I would prefer to maintain a hematocrit above 40%. Clinical Note: Some authors recommend replacing blood loss at 125-150% of measured loss due to the difficulty of accurately measuring blood loss in a situation where the margin of error is so small. 9) How would you establish your maximal allowable blood loss? While the optimal transfusion point for extremely low birth weight infants (< 1000 grams) remains controversial, I would take into consideration the presence of residual fetal hemoglobin (increased affinity for oxygen - shifting the oxyhemoglobin dissociation curve to the left), increased oxygen consumption in the neonate, the presence of decreased cardiopulmonary reserve (i.e. severe pulmonary disease, cyanotic congenital heart disease, and heart failure), and the acuity and persistence of blood loss. Therefore, I would consider transfusion of this sick, extremely low birth weight, premature infant, if his hematocrit dropped below 39%, or even earlier if the neonate became unstable and I felt that an increased hematocrit would be beneficial. With a starting hematocrit of 49%, this would place my transfusion point at about 20 mL of blood loss. Clinical Notes: Estimated Allowable Blood Loss = EBV X (Hi - Hf) / Hi 10) During dissection of the ductus arteriosus, the oxygen saturation drops from 90% to 78% and the heart rate drops from 150 to 86 beats/minute. What are you going to do? I would manually ventilate the patient with 100% oxygen, verify the accuracy of the monitors, and evaluate the neonate's ECG, airway pressures, and tidal volumes. Since surgical retraction of the lung during dissection of the ductus arteriosus can lead to increased right-to-left intra-pulmonary shunting (compression- induced increases in pulmonary vascular resistance could also potentially lead to right-to-left extra-pulmonary shunting through the PDA), hypoxia (secondary to right-to-left shunting), and bradycardia (probably due to hypoxemia), I would ask the surgeon to relax any traction on the lung until the patient is stabilized. At the same time, I would be assessing the patient's blood loss and volume status to determine if any correction was required. If the bradycardia persisted, I would consider administering atropine (0.01-0.02 mg/kg). Finally, I would adjust the inspiratory pressures and inspired oxygen concentration to ensure optimal ventilation and oxygenation.

1) What monitors would you place for this case? Are invasive monitors warranted? 2) How would you induce anesthesia? 3) How would you anesthetize the airway? 4) The patient refuses an awake intubation. What are you going to do?

1) What monitors would you place for this case? Are invasive monitors warranted? In addition to the standard ASA monitors, I would place a 5-lead EKG to monitor for myocardial ischemia as well as place an esophageal or bladder probe to monitor core temperature. Recognizing the potential for lithium-induced conduction problems or dysrhythmias, and given the increased perioperative risks associated with inadequately treated hyperthyroidism, such as a hyperdynamic circulation, cardiac arrhythmias, and thyroid storm, I would place an arterial line to aid in the monitoring and timely treatment of hemodynamic instability (these patients often exhibit an exaggerated sympathetic response to surgery). Considering her treatment with lithium (long term use may result in a form of vasopressin resistant diabetes insipidus), her questionable volume status (hyperthyroid state may result in hypovolemia), and the length of the case, I would place a Foley catheter. Finally, since the duration of action of muscle relaxants may be prolonged in patients receiving lithium, I would carefully monitor neuromuscular blockade with a peripheral nerve stimulator. 2) How would you induce anesthesia? Given her hyperthyroidism, airway obstruction, dysphagia, and GERD, my goals during induction would be to safely secure the airway while maintaining spontaneous respirations, maintaining stable hemodynamics, and avoiding aspiration. Therefore, I would administer an H2-receptor antagonist and metoclopramide; and ensure the presence of difficult airway equipment, a rigid bronchoscope, reinforced endotracheal tubes in various sizes, a tracheostomy kit, and a surgeon capable of performing an emergency tracheostomy. I would then verify sufficient beta-blockade, ensure adequate airway analgesia, and provide minimal sedation to prevent an exaggerated sympathetic response during intubation, recognizing that the latter two interventions could theoretically increase the risk of aspiration and exacerbate airway obstruction. Finally, I would place the patient in slight reverse trendelenburg position and perform an awake fiberoptic intubation, passing a reinforced endotracheal tube beyond the point of extrinsic compression. 3) How would you anesthetize the airway? While maintaining spontaneous ventilation, I would provide minimal sedation and supplemental oxygen; administer nebulized lidocaine to anesthetize the oropharynx (above the epiglottis); topicalize the nose in case a nasal airway becomes necessary; and block the superior laryngeal nerves to anesthetize the hypopharynx (below the epiglottis to the vocal cords). While a trans-tracheal block may aid in anesthetizing the larynx (below the vocal cords), it may not be an appropriate airway block for a patient with goiter. Moreover, when anesthetizing the airway, I would avoid epinephrine-containing local anesthetics to reduce the risk of an exaggerated hemodynamic response with systemic absorption. 4) The patient refuses an awake intubation. What are you going to do? I would make the patient, her family, and the surgeon aware of my concerns of possible difficult intubation, mask ventilation, and complete airway obstruction. At the same time, I would attempt to identify and address the patient's concerns with an awake intubation, reassuring her that all steps would be taken to make the procedure as comfortable as possible. If she still refused this procedure, I would administer an H2-receptor antagonist and metoclopramide; and ensure the presence of difficult airway equipment, a rigid bronchoscope, reinforced endotracheal tubes in various sizes, a tracheostomy kit, and a surgeon capable of performing an emergency tracheostomy. I would then have the neck prepped and draped, place the patient in slight reverse trendelenburg position, apply cricoid pressure, provide minimal sedation, and perform an inhalational induction with sevoflurane with the goal of maintaining spontaneous respiration until the airway is secured. Finally, I would pass a reinforced endotracheal tube, ensuring that the distal tip of the tube extends beyond the point of extrinsic compression. 5) How would you maintain anesthesia? I would maintain anesthesia with a balanced technique consisting of IV narcotics and a volatile agent, being careful to maintain an adequate depth of anesthesia to prevent exaggerated sympathetic responses to surgical stimulation. Additionally, I would avoid the administration of any agents that stimulate the central nervous system, such as atropine, ketamine, desflurane, ephedrine, epinephrine, and pancuronium. If intraoperative hypotension occurred, I would treat with fluids and a direct-acting vasopressor such as phenylephrine. Given the increased incidence of myasthenia gravis in hyperthyroid patients, any administration of muscle relaxants would begin with a reduced initial dose, with subsequent dosing carefully titrated based on responses from a peripheral nerve stimulator (also keep in mind the potential for prolonged blockade secondary to lithium treatment). Finally, I would take extra precautions to protect the patient's eyes (especially if there was evidence of proptosis) and closely monitor the patient for any signs of thyroid storm, such as hyperthermia, dysrhythmias, tachycardia, myocardial ischemia, congestive heart failure, and cardiovascular instability. 6) How does thyrotoxicosis effect minimum alveolar concentration (MAC)? There is no objective data to support the impression that hyperthyroidism has any direct effect on MAC. However, the increased cardiac output associated with thyrotoxicosis would theoretically increase the uptake of inhaled anesthetics. This increased uptake would slow the rate of rise of FA/FI (fraction alveolar/fraction inspired), which could potentially be perceived as an increased anesthetic requirement. In addition, increases in core body temperature often associated with thyrotoxicosis could potentially increase MAC. 7) How would you extubate this patient at the end of the case? While any preoperative mass-induced airway compression should be resolved following total thyroidectomy, the patient remains at risk for post-extubation airway obstruction secondary to tracheomalacia, hematoma, edema, or recurrent laryngeal nerve injury. Moreover, this hyperthyroid patient with dysphagia and GERD is at increased risk of aspiration and an exaggerated sympathetic response during emergence. Given these concerns, I would ensure the presence of difficult airway equipment; plan to extubate this patient when she was awake and demonstrating intact airway reflexes; administer beta-blockers, narcotics, and lidocaine as necessary to blunt a sympathetic response; pass a fiberoptic bronchoscope through the endotracheal tube, and slowly pull the tube back while, at the same time, visualizing the patency of the airway and vocal cord movement. If at any time the airway appeared to be compromised, I would immediately re-advance the ETT. 8) What would you expect to find with a recurrent laryngeal nerve injury? Acute injury to the abductor fibers of the recurrent laryngeal nerve (posterior cricoaryetnoid is the only ABductor) is one of the most concerning complications associated with thyroidectomy. While a rare occurrence, bilateral injury results in unopposed adduction of the true vocal cords, resulting in stridor, aphonia, and laryngeal obstruction. This type of injury requires intubation to maintain airway patency. Unilateral injury is more common, characterized by hoarseness, and does not result in laryngeal obstruction due to compensatory abduction of the unaffected vocal cord. Post-operative Management 1) Assume extubation was uneventful. Three hours after surgery the patient develops inspiratory stridor, restlessness, and tingling around her mouth. What is your differential diagnosis? Stridor associated with restlessness and circumoral tingling or numbness is consistent with hypocalcemia, a known complication of total thyroidectomy secondary to inadvertent surgical removal of the parathyroid glands. While this complication typically develops 24-96 hours after surgery, it may manifest in as early as 1 to 3 hours. However, I would also consider other possible causes or contributing factors such as cervical hematoma formation, aspiration (resulting in dyspnea and hypercarbia), post-intubation croup, residual neuromuscular blockade (higher incidence of myasthenia gravis in hyperthyroid patients), recurrent laryngeal nerve injury, hypoglycemia, hypoxia, or hypercapnia. 2) What would you do? I would evaluate the patient; provide supplemental oxygen; auscultate the chest; ensure stable hemodynamics, adequate ventilation, and a normal cardiac rhythm; examine the patient's neck for signs of hematoma formation; attempt to illicit Chvostek's sign (twitching of the facial muscles with tapping of the facial nerve at the angle of the jaw) or Trousseau sign (spasm of the hand muscles with occlusion of the brachial artery for three minutes using an inflated sphygmomanometer); examine the patient for hyperactive tendon reflexes; check an arterial blood gas, electrolytes, and serum phosphate concentration; and consider ordering a chest X-ray. If I believed her respiratory distress was secondary to hypocalcemia, I would administer intravenous calcium (rule of 10s: 10 mL of 10% calcium gluconate over 10 minutes), correct any hyperkalemia and/or hypomagnesemia (these potentiate hypocalcemia-induced cardiac and neuromuscular irritability), and continue to monitor the EKG for signs of cardiotoxicity (i.e. ventricular fibrillation, heart block). If, however, I believed her respiratory distress was due to hematoma-induced compression of the trachea, I would evacuate the hematoma, intubate the patient (airway edema secondary to hematoma-induced inhibition of venous and lymphatic drainage may persist even after hematoma evacuation), administer steroids and nebulized racemic epinephrine to decrease laryngeal edema, and sit the patient upright to facilitate venous drainage. Finally, if I believed that bilateral recurrent laryngeal nerve injury was the causative factor, I would immediately secure the airway with an endotracheal tube and notify the surgeon (who may consider tracheotomy). 3) Six hours after surgery the nurse calls you and reports the patient is tachycardic and febrile with a temp of 38.9 °C. She wants to administer aspirin as an antipyretic. Do you agree? I am concerned that her symptoms could potentially represent thyroid storm. Therefore, I would not administer aspirin to this patient because this drug may displace thyroid hormones from binding proteins in the blood, subsequently increasing free thyroid hormone levels and potentially exacerbating her condition. Instead, I would assess the patient, administer acetaminophen, ensure stable hemodynamics, and attempt to identify the underlying cause of her symptoms. Given this patient's hyperthyroid state, lack of anesthetic history, and recent olanzapine dosage adjustment, her symptoms could potentially represent thyroid storm, malignant hyperthermia, and neuroleptic malignant syndrome, respectively. Other potential causes that I would consider, include light anesthesia and an undiagnosed pheochromocytoma. 4) How would you differentiate between thyroid storm, malignant hyperthermia (MH), and neuroleptic malignant syndrome (NMS)? Differentiating between these various conditions would be very difficult in this patient with risk factors for all three conditions, especially since they all manifest similarly with tachycardia, hyperthermia, and mental status changes. However, both malignant hyperthermia and neuroleptic malignant syndrome result in a metabolic acidosis, profound hypercarbia, and muscle rigidity, symptoms not present during thyroid storm. Distinguishing between NMS and MH would be even more difficult in this patient who has received a triggering agent and is receiving medications that can lead to dopamine depletion. Two distinguishing features are: 1) NMS usually exhibits a slower progression to a critical temperature and multisystem organ failure as compared to MH; 2) non-depolarizing muscle relaxants will produce flaccid paralysis in patients experiencing NMS, but not in those experiencing MH. Unfortunately, treatment should be initiated immediately, making an observation of the speed of progression less helpful. Therefore, in a situation where I was unable to readily determine whether MH or NMS was the cause of a patient's hypermetabolic state, I would initiate treatment with dantrolene, which may prove helpful with both conditions (occasionally used as adjunctive treatment of NMS); intubate the patient following the administration of a non-triggering induction agent and a nondepolarizing muscle relaxant; hyperventilate the patient with 100% oxygen; consider administering bromocriptine (a dopamine agonist often used in the treatment of NMS), if the nondepolarizing agent results in the resolution of muscle rigidity; initiate cooling measures; and continue with the appropriate monitoring and supportive care. My notes: NMS occurs 2/2 Dopamine D2 receptor antagonism. In this model, central D2 receptor blockade in the hypothalamus, nigrostriatal pathways, and spinal cord leads to increased muscle rigidity and tremor via extrapyramidal pathways. 5) Assuming this was a case of thyroid storm how would you treat this patient? I would treat thyroid storm by administering acetaminophen and initiating active cooling measures (i.e. cold lavage, cooling blankets, ice packs, etc.) to control hyperthermia; titrating on additional beta-blocker to control tachycardia (esmolol or propranolol); provide intravenous fluids to ensure adequate intravascular fluid volume; correct any electrolyte imbalance; and administer propylthiouracil, sodium iodide, and hydrocortisone to reduce circulating levels of active thyroid hormone. If necessary, I would consider administering a catecholamine-depleting agent, such as reserpine or quanethidine, to aid in treating hyperadrenergic effects of thyroid storm.

1) What special considerations would you have concerning monitoring? 2) Do you think this patient is experiencing superior vena cava syndrome? 3) The surgeon confirms that the patient is suffering from superior vena cava syndrome. Does this concern you? 4) Would general or local anesthesia be preferable for this case? 5) Assume that you have decided to perform general anesthesia. How will you induce and intubate the patient? 6) Would you administer a muscle relaxant as part of your anesthetic maintenance? 7) The surgeon insists on muscle relaxation. Shortly after induction, he is advancing the mediastinoscope when the right radial arterial line indicates a blood pressure drop from 145/90 mmHg to 90/65 mmHg. What do you think is going on? 8) What would you do? 9) In managing his hypotension, would you consider utilizing nitrous oxide to allow for a reduction in volatile agent? 10) The surgeon says there is bleeding. Now, what will you do? 11) The patient requires sternotomy and repair of a tear in his superior vena cava. Would you extubate him immediately following the case?

1) What special considerations would you have concerning monitoring? Given this patient's increased risk for hemodynamic instability (poorly controlled hypertension, possible autonomic neuropathy, and the intraoperative hypotension associated with ACE inhibitors) and cerebral ischemia (carotid artery disease, poorly controlled hypertension, possible superior vena cava syndrome, potential for innominate artery compression during mediastinoscopy), I would place a 5-lead ECG and a left radial arterial line to facilitate the rapid identification and treatment of any cardiac ischemia and/or significant hypotension. Moreover, I would monitor the tracing of a pulse oximeter placed on his right upper extremity to quickly identify any compression of the innominate artery, recognizing that this is a known complication of mediastinoscopy. I would also monitor the patient's peak inspiratory pressure for signs of tracheal compression. Since patients with autonomic neuropathy are at increased risk for intraoperative hypothermia (due to impaired peripheral vasoconstriction), I would closely monitor his temperature. Finally, if muscle relaxation were required for the case, I would utilize a nerve stimulator, recognizing that patients with Lambert-Eaton myasthenic syndrome are very sensitive to both depolarizing and nondepolarizing muscle relaxants. Clinical Note: Some form of monitoring of the right radial pulse is mandatory during mediastinoscopy due to the risk of innominate artery compression with subsequent cerebral ischemia. Methods that may be employed include: (1) placement of a right radial arterial line, (2) monitoring the plethysmographic tracing of a pulse oximeter on the right hand, or (3) continuous palpation of the right carotid or right radial pulse. 2) Do you think this patient is experiencing superior vena cava syndrome? This patient's worsening cough, shortness of breath, jugular venous distention, and opacification of upper extremity collateral veins on CT are all consistent with superior vena cava syndrome. A definitive diagnosis could be established by venography. Other signs and symptoms that may be present in a patient with superior vena cava obstruction would include: (1) headache; (2) facial, neck, and upper limb edema; (3) chest pain, (4) dysphagia, (7) lightheadedness, (8) orthopnea, (9) hoarseness, (10) nasal stuffiness, (11) nausea, (12) pleural effusions, (13) papilledema, (14) visual disturbances, (15) mental confusion, and (16) facial cyanosis. 3) The surgeon confirms that the patient is suffering from superior vena cava syndrome. Does this concern you? I have several concerns related to this patient's superior vena cava syndrome including: (1) complicated airway management secondary to airway edema, (2) unreliable drug delivery through intravenous lines in the upper extremities (primarily in the setting of surgical damage to the vessel), (3) the potential for massive hemorrhage secondary to damage to any one of several major blood vessels in the surgical field (i.e. azygos vein, innominate artery, aorta, superior vena cava, and pulmonary artery), (4) compromised cerebral perfusion in this patient with carotid artery disease (impaired drainage of cerebral veins -> increased cerebral venous pressure -> increased ICP and impaired cerebral perfusion), and (5) an increased risk of postoperative respiratory complications, such as acute laryngospasm, bronchospasm, and airway obstruction (secondary to airway edema and/or mass compression).Given these concerns, I would (1) prepare for difficult airway management and minimize manipulation of the airway as much as possible to prevent exacerbating any airway edema (especially if there were concomitant tracheal compression, a.k.a. superior mediastinal syndrome); (2) place two large-bore intravenous catheters, with at least one of the lines inserted in his lower extremity (providing reliable delivery of drugs, fluids, and blood products even in the setting of a surgically damaged superior vena cava; (3) type and cross match blood to prepare for possible massive hemorrhage; (4) maintain the patient in the head up position to facilitate venous drainage and avoid increased airway edema and intracranial pressures; (5) employ cautious fluid management, recognizing that too much fluid would lead to venous engorgement and edema, while too little fluid would lead to decreased preload; (6) avoid coughing and bucking during emergence, recognizing that either could lead to exacerbated venous congestion and acute airway obstruction; and (7) maintain endotracheal tube placement and mechanical ventilation during the immediate postoperative period (unless the obstruction of the superior vena cava had been relieved). 4) Would general or local anesthesia be preferable for this case? Given the risk of increased airway edema and acute airway obstruction associated with superior vena cava syndrome, I would prefer to avoid instrumentation of the airway and perform the procedure under local anesthesia (this is possible for anterior mediastinoscopy, but not for cervical mediastinoscopy, (which is the most common approach). Moreover, this technique would avoid exposing this patient with cerebrovascular disease and increased risk for hypotension (secondary to poorly controlled hypertension, possible autonomic neuropathy, SVC syndrome, and ACE inhibitor administration) to the hemodynamic instability associated with laryngoscopy, induction, and general anesthesia; allow more reliable monitoring of neurologic function (i.e. an awake patient); and avoid the increased risk of aspiration associated with general anesthesia in this patient with possible autonomic neuropathy (LES and malignancy are associated with autonomic neuropathy; this patient's symptoms of constipation and orthostatic hypotension are consistent with this condition). However, there are several advantages to performing the procedure under general anesthesia with controlled positive-pressure ventilation, including: (1) increased flexibility for surgical manipulations, (2) decreased risk of surgical trauma resulting from sudden patient movement, (3) reduced risk of air embolism (the tip of the mediastinoscope is intrathoracic, placing the patient at risk for venous air embolism in the presence of venous bleeding and the negative intrathoracic pressure created during spontaneous ventilation), (4) improved conditions for the management of a significant complication like massive hemorrhage, and (5) reduced risk of coughing during the procedure (coughing during the procure can lead to venous engorgement with subsequent increased risk of surgical damage to local vessels). 5) Assume that you have decided to perform general anesthesia. How will you induce and intubate the patient? Given this patient's multiple comorbidities, I would: (1) ensure the presence of difficult airway equipment (airway edema associated with superior vena cava syndrome), a radial arterial line (increased risk for hemodynamic instability, poorly controlled hypertension, carotid artery disease), and adequate lower extremity intravenous access (increased risk for massive hemorrhage, superior vena cava syndrome); (2) provide aspiration prophylaxis (his constipation and orthostatic hypotension are consistent with autonomic neuropathy, a condition associated with both malignancy and LES); (3) position the patient in reverse trendelenburg to optimize cerebral drainage (SVC syndrome and carotid artery disease) and reduce airway edema (head up positioning helps to reduce the airway edema associated with SVC syndrome); (4) provide adequate airway analgesia to prevent a sympathetic response that may exacerbate his hypertension; (5) perform an awake intubation to secure the airway of this patient at increased risk for difficult airway management and aspiration prior to induction (the presence of SVC syndrome increases the risk of difficult airway management); and (6) induce him with narcotics and etomidate, with the goal of minimizing the risk of hypotension. 6) Would you administer a muscle relaxant as part of your anesthetic maintenance? While the administration of a depolarizer or nondepolarizer for muscle relaxation during the procedure would reduce the risk of surgical trauma secondary to coughing (coughing leads to venous engorgement, thereby increasing the risk of vascular trauma by the mediastinoscope) or sudden patient movement, I would prefer to avoid administering either of these medications to this patient suffering from LES. Not only are patients with LES extremely sensitive to both depolarizers and nondepolarizers, but also the reversal of neuromuscular blockade is less effective, particularly in patients being treated with 3,4-diaminopyridine or an anticholinesterase. Since this patient is already at increased risk for postoperative respiratory complications due to his superior vena cava syndrome and LES, I would prefer to avoid introducing any additional drugs that may further compromise his respiratory function. Moreover, the muscle relaxation associated with volatile agents is often exaggerated in patients with LES, making the administration of nondepolarizing agents unnecessary. If the administration of a nondepolarizer were deemed necessary, I would administer small doses, monitor his neuromuscular function carefully with a nerve stimulator, and evaluate his ventilatory function thoroughly prior to extubation. 7) The surgeon insists on muscle relaxation. Shortly after induction, he is advancing the mediastinoscope when the right radial arterial line indicates a blood pressure drop from 145/90 mmHg to 90/65 mmHg. What do you think is going on? There are a number of things that could be causing or contributing to his hypotension, including: (1) surgical compression of the innominate artery; (2) a vagal reflex resulting from manipulation of the trachea, great vessels, or vagus nerve; (3) anesthetic-induced cardiovascular depression; (4) autonomic neuropathy associated with the malignancy and/or LES (patients with autonomic neuropathy may be unable to compensate for the cardiovascular depression associated with anesthesia due to impaired peripheral vasoconstriction and baroreceptor function); (5) reduced preload secondary to superior vena cava syndrome; (6) tension pneumothorax (the pleural space may be entered unintentionally); (7) air embolism (a known complication of mediastinoscopy, occurring when surgical-induced vascular trauma makes possible the venous entrainment of air); (8) perioperative ACE inhibitor use (there is an increased risk of significant perioperative hypotension associated with perioperative ACE inhibitor administration), (9) infarction (poorly controlled hypertension, decreased preload, vascular disease, autonomic neuropathy); and (10) arrhythmia (his poorly controlled blood pressure predisposes him to perioperative arrhythmias). 8) What would you do? In managing his hypotension, I would: (1) check the arterial line for accuracy; (2) check the blood pressure in the left arm, recognizing that a difference in pressures between the arms may indicate innominate artery compression; (3) ask the surgeon to rule out massive hemorrhage, innominate artery compression, and compression of other vascular structures; (4) ensure adequate ventilation; (5) look at my ECG tracing to rule out arrhythmia or cardiac ischemia; (6) auscultate the chest for bilateral breath sounds; (7) reduce my volatile agent; (8) quickly review all administered drugs; (9) administer fluids and vasopressors as indicated; and (10) temporarily place the patient in trendelenburg position to improve preload, recognizing that this could exacerbate the venous congestion associated with SVC syndrome. If these interventions failed to resolve the problem, I would consider (11) placing a central line in the femoral vein, (12) ordering a bedside ultrasound to better evaluate the patient for pneumothorax, (13) placing a precordial Doppler (sporadic roaring sounds would be consistent with air embolism), and (14) employing echocardiography to better evaluate cardiac function. 9) In managing his hypotension, would you consider utilizing nitrous oxide to allow for a reduction in volatile agent? While nitrous oxide would potentially allow for a reduction in the amount of volatile agent employed to maintain adequate depth of anesthesia (and thus reduce the degree of cardiovascular depression), I would not use it for this case due to the risk of expanding an unrecognized pneumothorax or venous air embolism (spontaneous ventilation increases the risk of venous air embolism during mediastinoscopy). Pneumothorax may occur when the pleural space is unintentionally entered during the procedure. It is because of this known complication that a chest x-ray is required prior to discharge for patients who have undergone mediastinoscopy. 10) The surgeon says there is bleeding. Now, what will you do? In the case of hemorrhage, I would: (1) ask the surgeon discontinue the surgery and pack the wound with surgical sponges, (2) call for help, (3) ensure adequate large-bore access in the lower extremities (if the hemorrhage is resulting from a tear in the superior vena cava, medications and fluids utilized in resuscitation would be lost in the mediastinum), (4) initiate resuscitation with fluids and vasopressors, (5) have cross-matched blood brought to the operating room, (6) prepare for massive blood transfusion by notifying the blood bank and ensuring the presence of blood warmers, cell saver, and rapid infusers, (7) place a double-lumen tube or bronchial blocker if the surgeon believes a thoracotomy may be required, (8) place a precordial Doppler, recognizing that there is an increased risk for venous air embolism in the setting of vascular injury, and (9) convert to sternotomy or thoracotomy as necessary. While deliberate hypotension and reverse-trendelenburg positioning are sometimes used to reduce bleeding and improve surgical visualization in this setting, I would not employ these techniques in this case due to the patient's symptomatic cerebrovascular disease, potentially reduced preload (SVC syndrome), poorly controlled hypertension (rightward-shifting of the cerebral autoregulation curve), and possible autonomic neuropathy (may be unable to compensate for a change from supine to a head-up position). 11) The patient requires sternotomy and repair of a tear in his superior vena cava. Would you extubate him immediately following the case? Given the increased risk of postoperative respiratory complications associated with LES (i.e. respiratory muscle weakness and residual neuromuscular blockade), superior vena caval obstruction (airway edema resulting from caval obstruction and possibly exacerbated by aggressive fluid resuscitation), and aggressive fluid resuscitation (in response to his caval tear), I would leave the patient intubated and maintain mechanical ventilation during the immediate postoperative period. I would delay extubation until any major fluid shifts had resolved and the patient demonstrated adequate ventilatory function. Clinical Note: Patients can usually be extubated following mediastinoscopy and discharged on the same day after a chest x-ray is performed to rule out pneumothorax. However, this patient is at increased risk for post-operative respiratory complications.

1) Which monitors would you place for this case? 2) Do you need an arterial line? Where would you place it? 3) The surgeon suggests monitoring visual evoked potentials. What do you think? 4) Prior to induction, the surgeon injects the nasal mucosa with cocaine and the patient suddenly loses consciousness. What do you think is going on? What would you do? 5) Assume there were no complications during topicalization of the nose. How would you induce this patient? 6) Thirty minutes into the surgery the blood pressure suddenly drops to 62/28 mmHg. What are the potential causes? 7) His end-tidal CO2 has decreased. Does this move venous air embolism to the top of your list? 8) The Doppler is emitting sporadic roaring sounds. What would you do?

1) Which monitors would you place for this case? In addition to the ASA standard monitors I would place a 5-lead EKG to monitor for myocardial ischemia. Given the patient's hypertension, signs of increased ICP (headache), risk of massive hemorrhage (close proximity of the cavernous sinus and internal carotid artery), and increased risk of end-organ ischemia, I would place an arterial line to more accurately monitor hemodynamic changes. Since this case is going to be performed with the patient in the sitting position, I would place a precordial Doppler to aid in the detection of venous air embolism (place to the right of the sternum, between the 2nd and 4th ribs), and consider placing a central line to provide a means of aspirating embolized air from the heart (a multiorifice catheter placed with the tip 2 cm below the superior vena caval-atrial junction allows for optimal aspiration of air). Since the patient is experiencing signs consistent with compression of optic nerves (blurred vision), I would discuss with the surgeon the benefits of monitoring visual evoked potentials. Due to the length of the surgery, the potential need for drug-induced diuresis, and the risk of diabetes insipidus, I would place a Foley catheter. And, finally, given the inaccessibility of the arms after draping, I would consider placing a lower extremity nerve stimulator for monitoring of neuromuscular blockade. 2) Do you need an arterial line? Where would you place it? I believe an arterial line would be indicated in this case for several reasons: 1) the patient's hypertension places him at increased risk of hemodynamic instability and, if not well controlled, may have caused a right-ward shift of end organ autoregulation curves; 2) his blurred vision suggests the possibility of increased ICP, which would make the rapid identification of hypotension vital to preventing cerebral ischemia; and 3) the transphenoidal approach places the surgeon in close proximity of the cavernous sinus and internal carotid artery, which increases the risk of massive hemorrhage and hemodynamic instability. In placing the arterial line, I would utilize the dorsalis pedis or femoral artery, since patients with acromegaly have poor collateral blood flow to the hand, increasing the risk of ischemia with partial or complete obstruction of the radial artery. 3) The surgeon suggests monitoring visual evoked potentials. What do you think? Visual evoked potentials (VEPs) are sometimes used to monitor the integrity of the optic nerves when the surgeon believes they are at significant risk of surgical trauma during tumor resection. Unfortunately, VEPs are extremely sensitive to inhalational and intravenous anesthetics, making consistent monitoring very difficult during general anesthesia. If the surgeon believed that this type of monitoring would be beneficial, I would plan to use a balanced technique with narcotics and a low concentration of volatile agent to minimize anesthetic interference with VEP monitoring. 4) Prior to induction, the surgeon injects the nasal mucosa with cocaine and the patient suddenly loses consciousness. What do you think is going on? What would you do? The timing of the event suggests that the local anesthetic injected during topicalization of the nose has spread to the CNS causing a total spinal, systemic toxicity, or a cocaine associated dysrhythmia. Therefore, I would: (1) apply cricoid pressure (risk of aspiration with GERD), (2) intubate the patient (to reduce the risk of aspiration and control ventilation avoiding hypoxia, hypercapnia, and acidosis is very important in the setting of local anesthetic toxicity), (3) ventilate with 100% oxygen, (4) check the EKG and blood pressure, (5) call for a lipid emulsion kit, and (6) administer vasoactive, inotropic and antiarrhythmics agents as indicated. If I suspected local anesthetic toxicity, I would (7) administer a benzodiazepine (to avoid seizure-induced patient injury and acidosis), (8) administer small doses of succinylcholine to treat ongoing tonic-clonic movement (to minimize the hypoxia and acidosis associated with muscle activity,) and (9) provide hemodynamic support as required. If the signs and symptoms of local anesthetic toxicity appeared to be rapidly progressing, the patient experienced prolonged seizure activity, or he developed signs of cardiac toxicity (i.e. bradycardia, heart block, hypotension, asystole, or ventricular arrhythmia), I would (10) initiate lipid emulsion therapy and (11) alert the nearest facility with cardiopulmonary bypass capability. 5) Assume there were no complications during topicalization of the nose. How would you induce this patient? This patient's obesity, his history of sleep apnea, and the physiologic changes of acromegaly, including the large protruding mandible, large tongue, and hoarse voice (suggestive of acromegalic involvement of the larynx) noted on examination, place him at increased risk of difficult airway management. Therefore, I would administer metoclopramide and an H2 receptor antagonist (history of GERD); apply the appropriate monitors; ensure the presence of emergency airway equipment, including smaller ETTs in case there is subglottic stenosis; place the patient in the sniff position or 30° reverse trendelenburg position; anesthetize the airway; provide adequate preoxygenation; and attempt an awake fiberoptic intubation. My goals in developing this plan are to safely secure the airway, while avoiding apnea, hypoxia, aspiration, and a sympathetic surge during laryngoscopy (his hypertension places him at risk for exaggerated hemodynamic responses which could prove harmful in the presence of CAD or elevated intracranial pressure). 6) Thirty minutes into the surgery the blood pressure suddenly drops to 62/28 mmHg. What are the potential causes? There are many possible causes for his drop in blood pressure, including massive hemorrhage secondary to dissection into the cavernous sinus or internal carotid artery, venous air embolism, myocardial ischemia, anesthetic overdose, delayed anaphylactic reaction, cardiac dysrhythmia, and tension pneumothorax. Therefore, I would deliver 100% oxygen, verify proper endotracheal tube placement, auscultate the chest, verify proper function of the arterial line, evaluate the EKG, reduce my anesthetic agent, ask the surgeon about bleeding, look at the surgical field, listen to the precordial Doppler, check the end-tidal CO2, and look for hemoglobin in the urine. 7) His end-tidal CO2 has decreased. Does this move venous air embolism to the top of your list? Since the patient is in the sitting position, venous air embolism (VAE) is already at the top of my list. However, a decrease in end-tidal CO2 is not diagnostic for VAE. While the increased alveolar dead space and/or reduced cardiac output that results from significant VAE does lead to a decrease in end-tidal CO2, this same decrease would be seen with any event that significantly reduced cardiac output. Unfortunately, any of the items in my differential could lead to decreased cardiac output with subsequently decreased end-tidal CO2. Similarly, the various complications in my differential would likely produce other clinical signs of VAE, such as tachycardia, cardiac arrhythmias, hypoxia, and cyanosis. Therefore, to determine if this clinical picture were secondary to VAE, I would listen for sporadic roaring sounds from the precordial Doppler, auscultate the heart for the characteristic "mill wheel" murmur (best heard through an esophageal stethoscope), attempt to aspirate air through the central venous catheter, and/or consider transesophageal echocardiography. 8) The Doppler is emitting sporadic roaring sounds. What would you do? Since this finding is consistent with a venous air embolism, I would immediately ask the surgeon to flood the field with saline; discontinue nitrous oxide (probably should be avoided in the first place) and deliver 100% oxygen; attempt to aspirate entrained air through the central venous catheter (to remove air that may be interfering with right-sided cardiac output); consider the application of direct jugular venous compression to increase venous pressure at the surgical site, provide cardiovascular support with fluid, vasoconstrictors, inotropes, and chest compressions, as necessary; and treat bronchospasm with beta-adrenergic agonists (reflex bronchospasm may occur with the entry of air into the pulmonary artery) . Clinical Notes: · PEEP has been recommended to increase venous pressure at the surgical site. However, the application of PEEP may not be optimal for several reasons: 1) the potential for impaired systemic venous return in a patient with significant cardiovascular dysfunction; 2) it is less effective than jugular venous compression in increasing cerebral venous pressure when in the sitting position (where the head is significantly higher than the heart); and 3) PEEP may reverse the normal trans-atrial (left > right) pressure gradient, increasing the risk of paradoxical embolism with an unrecognized patent foramen ovale. · Placing the patient in the head down, right-chest-up position (or lateral position with the right side up) has been advocated to move air into the right atrium where it is more easily aspirated through the central venous catheter, and less likely to cause an air lock in the right ventricle. However, this type of positioning is unproven, difficult, or even dangerous during most intracranial operations, and wastes valuable treatment time to achieve.

5) What is the cause of uremic thrombocytopathia? 6) How do you treat uremic thrombocytopathia?

5) What is the cause of uremic thrombocytopathia? The mechanisms by which uremia contributes to thrombocytopathia include: 1) decreased von Willebrand factor (vWF) formation and release secondary to the accumulation of various metabolites, 2) increased synthesis of nitric oxide and prostacyclin, which have platelet inhibitory effects, and 3) uremia-induced anemia, which contributes to decreased viscosity with a subsequent decrease in platelet interaction with endothelial surfaces. Patients with chronic renal failure may also be more susceptible to coagulopathy because they tend to have reduced levels of factor V and the vitamin K-dependent factors (II, VII, IX, X). 6) How do you treat uremic thrombocytopathia? The treatment options for uremic thrombocytopathia include: 1) desmopressin (DDAVP) to increase the release of vWF; 2) erythropoietin, which may exert its affect by correcting anemia; 3) conjugated estrogens, which may reduce nitric oxide formation; 4) cryoprecipitate, which contains vWF; 5) platelets; and 6) hemodialysis to remove uremic acid. The fastest and most effective treatment is hemodialysis, which eliminates the uremic acid and quickly restores adequate platelet function. If hemodialysis were ineffective in correcting a clinically significant and life-threatening thrombocytopathic coagulopathy, I would consider a platelet transfusion.

6) You are using oxygen cylinders for the case. Two hours into the procedure, the patient's blood pressure increases to 205/106 mmHg, and his heart rate decreases to 42 beats per minute. What would you do? 7) You discover that the Foley catheter had been obstructed. What is the pathophysiology of autonomic hyperreflexia (AH)?

6) You are using oxygen cylinders for the case. Two hours into the procedure, the patient's blood pressure increases to 205/106 mmHg, and his heart rate decreases to 42 beats per minute. What would you do? Recognizing that the development of hypertension and bradycardia in this paraplegic patient with a high spinal cord lesion is consistent with autonomic hyperreflexia (AH), and that bladder distention or some other stimulus below the level of the spinal cord transection is the likely cause of his hypertension and bradycardia, I would: (1) ask the surgeon discontinue any unnecessary stimulation, (2) deepen the anesthetic, (3) administer a direct acting vasodilator (i.e. sodium nitroprusside), (4) ensure that the bladder is emptying appropriately (i.e. check the Foley catheter and urine output), (5) place an arterial line for continuous blood pressure monitoring, and (6) monitor the patient closely for subsequent complications, such as cerebral, retinal, or subarachnoid hemorrhage; seizures; myocardial ischemia; dysrhythmias; and pulmonary edema (left ventricular failure due to increased afterload). 7) You discover that the Foley catheter had been obstructed. What is the pathophysiology of autonomic hyperreflexia (AH)? Cutaneous (pain) or visceral (i.e. bladder or rectal distention) stimulus below the level of spinal cord injury results in a reflex sympathetic discharge. Because the area of the body below the transaction is neurologically isolated, the sympathetic activity in this area is not modulated by inhibitory impulses from higher central nervous system centers. This unopposed sympathetic activity leads to vasoconstriction below the level of the lesion, with reflex vasodilation above the level of the lesion. When the lesion is above T7, vasodilation above the lesion is insufficient to prevent systemic hypertension, which then stimulates carotid sinus receptors, leading to reflex bradycardia.

1) If the patient required post-op ventilation, which mode of ventilation would you recommend? 2) Postoperatively, the patient continues to have excessive bleeding and the surgeon suggests giving more protamine. Would you agree? 3) How does hypothermia result in a coagulopathy? 4) How does thromboelastography measure coagulation? 5) Following emergence the patient is unable to move his legs. What do you think is going on? 6) How does surgical dissection and placement of an aortic cross-clamp increase the risk of spinal cord ischemia?

Assuming the patient's lung compliance was normal, I would place the patient on synchronized intermittent mandatory ventilation. This would provide adequate ventilatory support while allowing the patient to assist in the work of breathing and exercise respiratory muscles until weaned from the ventilator. Given the history of aortic regurgitation, the recent administration of large amounts of fluids and blood products, the use of one-lung ventilation, and possible surgical trauma (i.e. surgical manipulation of the diaphragm and the lungs; phrenic nerve and recurrent laryngeal nerve injury, surgically-induced tracheobronchial bleeding), he may have reduced pulmonary compliance secondary to pulmonary edema and/or atelectasis. In this case, I would consider a pressure control mode of ventilation to maximize oxygenation and avoid ventilator associated lung injury. 2) Postoperatively, the patient continues to have excessive bleeding and the surgeon suggests giving more protamine. Would you agree? While additional protamine may be the appropriate treatment, I would first assess the patient for other potential causes of post-operative bleeding, such as hypothermia, thrombocytopenia, thrombocytopathia, and depletion of coagulation factors. My assessment would include ordering a PTT, PT, INR, platelets, fibrinogen level, a thromboelastogram (TEG), and an ACT or heparin-protamine titration assay. If either the ACT or heparin-protamine titration assay indicated residual heparin effects, I would consider administering additional protamine, recognizing that excessive protamine could lead to paradoxical bleeding. If bleeding continued despite these interventions, surgical exploration may be required to check anastomotic sites. 3) How does hypothermia result in a coagulopathy? Mild hypothermia (33-37 °C) can induce a coagulopathy secondary to cold-induced defects in platelet aggregation and adhesion. Coagulation enzyme activity, however, is only minimally affected within this range of temperatures. Therefore, thromboelastography may prove more helpful than an aPTT in identifying the coagulation defect in a mildly hypothermic patient (an aPTT does not measure platelet function). On the other hand, at temperatures below 33 °C, both platelet function and coagulation enzyme activity are abnormal, which would be detected by either test. 4) How does thromboelastography measure coagulation? The thromboelastogram (TEG) measures the viscoelastic properties of blood during induced clot formation. The pattern of changing shear-elasticity enables the evaluation of clot formation kinetics and growth, as well as the strength and stability of the formed clot. Clinically, the TEG allows the identification of platelet dysfunction, primary fibrinolysis, stage I and II DIC, as well as residual Anticoagulants. The most common parameters derived from the study include: 1) R - the time to initial clot formation, representing intrinsic pathway factor function; 2) K & the alpha angle - the speed of clot formation, representing deficient thrombin and fibrin formation; and 3) MA - the strength of the clot, a reflection of platelet number and function. 5) Following emergence the patient is unable to move his legs. What do you think is going on? This leg weakness is very concerning, since it could represent either an epidural/spinal hematoma with cord compression (secondary to lumbar drain and/or epidural catheter placement), neurologic injury secondary to intra-operative spinal cord ischemia, or the infusion of local anesthetics through an epidural catheter inadvertently placed in the intrathecal space. Any of these complications should be considered extremely serious, requiring immediate intervention to prevent further harm. Therefore, I would: (1) perform a neurologic examination, (2) increase the mean arterial pressure to > 80 mmHg to optimize spinal cord perfusion, (3) discontinue the epidural infusion (the motor weakness associated with neuraxial local anesthetics may confuse the clinical picture), (4) aspirate the epidural catheter for CSF to identify intrathecal placement, (5) decrease the CSF pressure to 5-10 mmHg to maximize spinal cord perfusion; and (6) give consideration to a CT/MRI and neurological consult. 6) How does surgical dissection and placement of an aortic cross-clamp increase the risk of spinal cord ischemia? The risk of spinal cord ischemia is increased because surgical ligation of significant radicular arteries and the application of the aortic cross-clamp can lead to hypoperfusion of the spinal cord. The anterior spinal cord is particularly vulnerable due to its reliance on a single anterior spinal artery for blood supply, the largest and most important of these being the artery of Adamkiewicz. Moreover, spinal cord perfusion may be further compromised by the increase in CSF pressure associated with aortic cross-clamp application (application of the cross-clamp leads to cerebral hyperemia with subsequent shifting of CSF into the spinal compartment). This increase in CSF pressure in combination with decreased distal aortic pressures can lead to decreased spinal cord perfusion and ischemia. The risk of spinal cord ischemia can be reduced by: (1) avoiding hypotension (the maintenance of proximal hypertension should be considered - especially when planning a "clamp and sew" technique without distal bypass), (2) minimizing aortic cross-clamp time, (3) reattaching critical segmental arteries, (4) achieving an appropriate level of hypothermia, (5) lowering CSF pressure via passive drainage and maintaining lower pressures postoperatively, (6) monitoring spinal cord function using SSEPs and MEPs, (7) utilizing a shunt or bypass to increase distal perfusion, (8) ensuring a normal hematocrit and PaO2, and (9) considering the administration of one or more of the various pharmacologic agents utilized for spinal cord protections, such as corticosteroids (i.e. dexamethasone or methylprednisolone), magnesium, barbiturates, calcium channel blockers, dextrorphan, papaverine, mannitol, and naloxone.

Causes of hypertension include: Signs of end-organ damage include:

Causes of hypertension include: (1) chronic kidney disease, (2) renovascular disease, (3) chronic steroid therapy (Cushing's syndrome), (4) sleep apnea, (5) drugs (i.e. cocaine, amphetamines, certain dietary supplements, oral contraceptives), (6) alcohol abuse, (7) obesity/metabolic syndrome, (8) thyroid or parathyroid disease, (9) pheochromocytoma, and (10) coarctation of the aorta. • Signs of end-organ damage include (5 cardiac, 2 neuro, 1 kidney): (1) left ventricular hypertrophy, (2) angina, (3) myocardial infarction, (4) congestive heart failure, (5) coronary artery disease, (6) stroke, (7) transient ischemic attack, (8) chronic kidney disease, (9) retinopathy, and (10) peripheral artery disease.

Common Causes of Atrial Fibrillation:

Heart Failure, Cardiomyopathy, Acute MI, Longstanding Hypertension, Valvular Heart Disease, Hyper/Hypothyroidism, Drugs (cocaine, sympathomimetics), PE, Hypoxemia, Sick Sinus Syndrome

A 28-year-old, 104 kg, G2P1 female, with premature rupture of membranes at 37 weeks gestation and preeclampsia, is being induced secondary to maternal fever, suspected chorioamnionitis, and fetal tachycardia. She is requesting an epidural for vaginal delivery. Her history includes multiple sclerosis, migraine headaches, asthma, and aortic stenosis with a mean transvalvular gradient of 50 mmHg. She is receiving albuterol for asthma, digoxin for aortic stenosis, methylprednisolone for multiple sclerosis, antibiotics for suspected chorioamnionitis, and 80 mg of enoxaparin BID for deep vein thrombosis that developed during this pregnancy. BP = 146/88 mmHg; Temp. = 38.8 °C; Platelets = 115,000. 1) What do you think of her aortic transvalvular gradient? 2) Does this patient require bacterial endocarditis prophylaxis? 3) Would you agree to a labor block? 4) Wouldn't the drop in SVR often associated with neuraxial anesthetics improve her cardiac output? 5) Assuming you were willing to place an epidural for labor, how long would you have to wait following her last dose of enoxaparin?

Her transvalvular gradient of 50 mmHg is very concerning since it may represent severe, or even critical, aortic stenosis. Recognizing that the hyperdynamic circulation associated with pregnancy can lead to an increased aortic transvalvular gradient and overestimate of the severity of her aortic stenosis, I would also review the echocardiographic estimation of the aortic valve area (a superior method of assessing the severity of aortic stenosis in pregnant patients). Likewise, I would keep in mind that a failing or dysfunctional left ventricle can lead to a decreased aortic transvalvular gradient as compared to a normally functioning left ventricle, potentially leading to an underestimate of the severity of her aortic stenosis. Clinical Note: aortic stenosis severity: 2) Does this patient require bacterial endocarditis prophylaxis? According to the revised American Heart Association guidelines from 2007, aortic stenosis is no longer an independent indication for endocarditis antibiotic prophylaxis. The new guidelines now emphasize prophylaxis for those conditions associated with the highest risk for adverse outcomes from infectious endocarditis (IE) versus those associated with the highest lifetime risk of acquisition of IE. Therefore, prophylaxis is more reasonably reserved for patients with the following conditions: (1) a prosthetic cardiac valve or prosthetic material used for valve repair; (2) a previous occurrence of IE; (3) unrepaired cyanotic congenital heart disease; (4) the 6 month postoperative period following a repaired congenital heart defect using prosthetic material or a device; (5) repaired congenital heart disease with residual defects at the site or adjacent to the site of a prosthetic patch or device (which inhibits endothelialization); and (6) cardiac transplantation recipients who develop cardiac valvulopathy. Although aortic stenosis is not an independent indication for antibiotic prophylaxis, I would continue any current antibiotics she is receiving to treat chorioamnionitis. 3) Would you agree to a labor block? While I recognize there are some concerns about providing neuraxial anesthesia to a patient with multiple sclerosis, maternal fever, severe aortic stenosis, and preeclampsia, I would agree to a labor block in this case as long as her enoxaparin had been discontinued for at least 24 hours (therapeutic dosing requires a 24 delay prior to neuraxial block placement). Neuraxial blockade would be desirable because it would more reliably prevent pain-induced tachycardia in this patient with severe aortic stenosis. Moreover, should a cesarean section be required, it would preclude the necessity for providing general anesthesia to this patient who is at increased risk for difficult airway management (the physiologic changes of pregnancy along with her obesity, preeclampsia, and asthma increase the risk of difficult airway management). In regards to the previously mentioned concerns, the evidence suggests that neuraxial anesthesia may be safely provided for patients at increased risk for bacteremia (fever and chorioamnionitis), despite concerns that dural puncture and vascular disruption could lead to increased incidence of epidural abscess and meningitis in these patients. Also, while neuraxial anesthesia has been associated with an exacerbation of multiple sclerosis, this has been most significant with spinal anesthesia and epidural anesthesia utilizing higher concentrations of local anesthetic. Therefore, most practitioners consider it safe to provide epidural anesthesia using dilute solutions of local anesthetic for patients in labor. Furthermore, while preeclampsia often affects both platelet number and function, the risk of bleeding in the epidural or spinal space should be minimal with her platelet level of 115,000. And, finally, even with severe aortic stenosis, an epidural anesthetic can be safely provided by slowly administering small doses of local anesthetic without epinephrine (epinephrine could lead to tachycardia), allowing for adequate fluid replacement and compensatory vasoconstriction above the level of neuraxial blockade. 4) Wouldn't the drop in SVR often associated with neuraxial anesthetics improve her cardiac output? While a decrease in afterload often promotes increased cardiac output, this is not the case in severe aortic stenosis where the stenotic valve, rather than systemic vascular resistance, creates most of the afterload on the left ventricle. Therefore, a significant drop in systemic vascular resistance is poorly tolerated by these patients with a relatively fixed stroke volume and an inability to adequately increase cardiac output. The subsequent diastolic hypotension and compensatory tachycardia places this patient with a likely hypertrophied myocardium at increased risk for subendocardial ischemia. 5) Assuming you were willing to place an epidural for labor, how long would you have to wait following her last dose of enoxaparin? Based on recommendations of the second ASRA consensus conference on neuraxial anesthesia and anticoagulation, I would wait at least 24 hours following her last therapeutic dose of enoxaparin before initiating neuraxial blockade. While a delay of only 10-12 hours is recommended with patients receiving low-dose LMWH for thromboprophylaxis, a longer delay of 24 hours is recommended for those receiving high-dose LMWH for DVT treatment. Of course, in addition to considering the guidelines, I would weigh all risks and benefits before making a definitive decision. My considerations in this regard would include airway management (i.e. the anticipated difficulty of airway management; and the likelihood that she will require a cesarean section where airway management would be necessary in the absence of neuraxial blockade), the importance and urgency of providing neuraxial anesthesia (i.e. cesarean section rather than just labor; the importance of more effective pain control for a patient with aortic stenosis), and the risks associated with alternative anesthetic options (i.e. PCA for labor; or general anesthesia for cesarean section).

A 62-year-old male with suspected lung cancer presents for mediastinoscopy for staging and diagnosis after sputum cytology and needle biopsy failed to provide a definitive diagnosis. Notes from his primary care physician indicate that in the last 6 months the patient has developed a persistent cough, worsening shortness of breath, orthostatic hypotension, and constipation. His medical history is also significant for tobacco use, carotid artery disease, and hypertension. VS: P = 87; BP = 175/103; R = 10; T = 36.6 °C 1) What do you think of his shortness of breath? 2) With additional investigation you find that the patient was recently diagnosed with Lambert-Eaton Myasthenic Syndrome (LES). Tell me about this syndrome? 3) His sodium is 129 mEq/L. What do you think may be the cause?4) Basing your decision on this low sodium level, would you proceed with surgery? 5) Does his carotid artery disease concern you? 6) Are you concerned about his blood pressure? 7) What would you do? 8) You discover the patient is taking an ACE inhibitor. Would you continue this medication throughout the perioperative period? 9) Considering this patient's hypertension and carotid artery disease, do you think that a reasonable physician would proceed with this case?

I am very concerned about his shortness of breath, recognizing that there are a number of clinical conditions associated with his medical history that could be causing or contributing to his symptomatology, such as: (1) pulmonary disease resulting from his tobacco abuse (i.e. COPD and/or pneumonia); (2) mass- induced ventilation perfusion mismatching; (3) mass-induced post-obstructive pneumonia; (4) superior vena cava syndrome secondary to neoplastic invasion of the vessel wall (usually in association with intravascular thrombosis), with the resultant obstruction of venous drainage leading to mucosal edema and venous engorgement of the airways and subsequent dyspnea, orthopnea, and coughing; (5) mass compression of the heart or other great vessels (i.e. direct cardiac compression, pericardial effusion, pulmonary artery compression); (6) cardiac disease in this older male smoker with hypertension and known carotid disease; or (7) a late finding of Lambert-Eaton Myasthenic syndrome (LES), a known paraneoplastic syndrome that may also present with signs of autonomic dysfunction, such as constipation and orthostatic hypotension. Clinical Notes: There are a number of paraneoplastic and paraendocrine syndromes associated with cancer. Common Paraneoplastic Syndromes: (1) Humoral Hypercalcemia - Tumor release of parathyroid hormone-related peptides can lead to muscle weakness, cardiac arrhythmias, nausea, vomiting, and renal failure. (2) Syndrome of Inappropriate secretion of Antidiuretic Hormone (SIADH) - tumor production of arginine vasopressin can lead to hyponatremia, decreased serum osmolarity, and inappropriately increased urine osmolarity in the setting of euvolemia and normal thyroid and adrenal function. (3) Cushing's Syndrome - increased secretion of ACTH or CRH can lead to hypokalemia, alkalosis, hypertension, and psychosis. (4) Lambert-Eaton Myasthenic Syndrome - most commonly associated with small cell lung cancer. 40-60% of patients presenting with LES are subsequently diagnosed with a malignancy (3% of cases of small cell lung cancer are associated with LES). Paraendocrine Syndrome: Carcinoid syndrome - the most common extra-intestinal location of carcinoid tumors is the lung. 2) With additional investigation you find that the patient was recently diagnosed with Lambert-Eaton Myasthenic Syndrome (LES). Tell me about this syndrome? Lambert-Eaton myasthenic syndrome (LES) is a syndrome in which the formation of antibodies to prejunctional voltage-gated calcium channels results in a reduced release of acetylcholine at the motor end-plate. Patients characteristically present with proximal weakness of the lower extremities (sometimes progressing to involve the upper extremities) and signs of autonomic dysfunction, such as dry mouth, impotence, constipation, and orthostatic hypotension. In contrast to Myasthenia Gravis where strength improves with rest, patients with LES show improved strength with muscle activity. While bulbar involvement and respiratory compromise are less common than with Myasthenia Gravis, they do sometimes occur (respiratory weakness is a late finding). In the case of malignancy, treatment begins with (1) cancer therapy. Other interventions include: (2) plasma exchange, (3) intravenous immune globulin therapy,(4) the administration of prednisone or azathioprine for immunosuppression, (5) increasing the release of acetylcholine with 3,4-diaminopyridine (should be continued up to the time of surgery), or (6) decreasing the degradation of acetylcholine with pyridostigmine (a cholinesterase inhibitor). 3) His sodium is 129 mEq/L. What do you think may be the cause? Given the patient's medical history, his hyponatremia could be the result of SIADH, a paraneoplastic syndrome associated with malignancy, or it could possibly be the result of diuretic administration if this type of medication were being used to treat his hypertension (while loop diuretics can lead to hyponatremia, they are less likely to do so than thiazide diuretics). Further evaluation would be helpful in determining the etiology. Hyponatremia secondary to SIADH is associated with normovolemia (no signs of hypovolemia, such as orthostatic hypotension, tachycardia, or dry mucous membranes; no signs of hypervolemia, such as peripheral edema, ascites, or pulmonary edema), a normal total body sodium level, and an elevated urine osmolality and urinary sodium concentration (increased urine osmolality and urinary sodium concentration result from reduced urine volume). On the other hand, hyponatremia secondary to diuretic use is associated with hypovolemia (however, this reduction in blood volume may be attenuated by the stimulation of ADH secretion, with subsequent water retention), a low total body sodium level, and an elevated urinary sodium concentration. 4) Basing your decision on this low sodium level, would you proceed with surgery? Since proceeding with surgery in the presence of a sodium concentration under 130 mEq/L places the patient at risk of developing significant cerebral edema, I would prefer to delay the surgery until the cause of the hyponatremia could be determined, and the sodium level corrected. However, I would discuss the pros and cons of delaying the case with the surgeon since even a short delay may be unacceptable if an urgent diagnosis is needed. 5) Does his carotid artery disease concern you? His carotid artery disease does concern me because of the risk of inadequate cerebral perfusion secondary to a hypertension-induced rightward shifting of the cerebral autoregulation curve and/or a surgically-induced decrease in cerebral blood flow secondary to compression of the innominate artery. A patient with carotid artery disease may be unable to compensate adequately for these insults resulting in neurologic damage. In fact, because of the risk of innominate artery compression, mediastinoscopy is relatively contraindicated in a patient with cerebrovascular disease. In addition to shifting the cerebral autoregulation curve to the right, his poorly controlled hypertension places him at increased risk for cerebral ischemia due to blood pressure lability. Finally, I would be even more concerned if I believed that his cough and shortness of breath were secondary to superior vena cava syndrome, recognizing that partial or complete obstruction of the superior vena cava could further compromise cerebral perfusion (impaired drainage of cerebral veins -> increased cerebral venous pressure increased ICP and impaired cerebral perfusion). Therefore, if avoiding mediastinoscopy were not an option, I would monitor the patient for signs of innominate artery compression (i.e. right radial arterial line or right upper extremity pulse oximeter tracing), attempt to optimize his blood pressure, and avoid any additional factors that could compromise cerebral perfusion such as hypotension, additional obstruction of cerebral venous drainage, hypercapnia, and acidosis. Clinical Notes: o Previous mediastinoscopy is a strong contraindication to mediastinoscopy, o Relative contraindications to mediastinoscopy include: (1) severe tracheal deviation, (2) cerebrovascular disease, (3) severe cervical spine disease with limited neck extension, (4) previous chest radiotherapy, and (5) thoracic aortic aneurysm. 6) Are you concerned about his blood pressure? I am concerned about his blood pressure because poorly controlled hypertensive patients are more prone to intraoperative-end-organ ischemia (i.e. myocardial infarction and stroke), arrhythmias, congestive heart failure, hypotension, and hypertension. The potential for blood pressure lability and a rightward-shifted cerebral autoregulation curve is even more concerning in this patient with carotid artery disease, who is already at increased risk of compromised cerebral perfusion (i.e. cerebral vascular disease, mediastinoscopy, possible superior vena cava syndrome). Clinical Notes: · For patients > 50 years of age: a systolic blood pressure in excess of 140 mmHg is a more important risk factor for cardiovascular disease than is an elevated diastolic pressure (this does not hold true for patients younger than 50 years of age). · For patients 40-70 years of age: each incremental increase in SBP of 20 mmHg or DBP of 10 mmHg above 115/75 mmHg doubles the risk of cardiovascular disease. · Hypertension in children is diagnosed when repeated blood pressure measurements are > the 95th percentile after making adjustments for age, height, and gender. 7) What would you do? I would perform a careful history and physical focused on blood pressure medications, the adequacy of blood pressure control, any identifiable cause of his hypertension (i.e. sleep apnea, kidney disease, or hormonal imbalance), and the presence of associated end-organ damage (cardiac, cerebral, renal, and vascular disease). I would also obtain an ECG and measure electrolytes, blood urea nitrogen, and creatinine to further evaluate end-organ damage and identify metabolic derangements secondary to blood pressure medications. Finally, if the urgency of the case did not allow for blood pressure optimization over several weeks, I would administer pharmacologic agents to lower his blood pressure to around 140/90 mmHg while carefully monitoring for any signs of cerebral ischemia (I would not attempt to normalize his blood pressure since his cerebral autoregulation curve is shifted to the right). Clinical Notes: • A reasonable approach to the management of preoperative hypertension for an elective procedure is to delay elective surgery for at least 6-8 weeks to optimize the blood pressure of any patient who: 1) has SBP readings > 180 mmHg or DBP readings >110 mmHg, 2) has stage 1 or stage 2 hypertension with concomitant end-organ damage, and/or 3) is undergoing cardiac surgery, carotid surgery, or pheochromocytoma resection. However, the decision to delay any case (and the length of that delay) must weigh the risks of blood pressure optimization against the risk of surgical delay. • Classification of Blood Pressure (adults >18 years of age)*: Normal: SBP < 120 and DBP < 80 mmHg Prehypertensive: SBP of 120-139 or DBP of 80-89 mmHg Stage 1 Hypertension: SBP of 140-159 or DBP of 90-99 mmHg Stage 2 Hypertension: SBP > 160 or DBP > 100 mmHg *Based on the average of 2 or more readings taken in the seated position on two separate occasions. • Causes of hypertension include: (1) chronic kidney disease, (2) renovascular disease, (3) chronic steroid therapy (Cushing's syndrome), (4) sleep apnea, (5) drugs (i.e. cocaine, amphetamines, certain dietary supplements, oral contraceptives), (6) alcohol abuse, (7) obesity/metabolic syndrome, (8) thyroid or parathyroid disease, (9) pheochromocytoma, and (10) coarctation of the aorta. • Signs of end-organ damage include (5 cardiac, 2 neuro, 1 kidney): (1) left ventricular hypertrophy, (2) angina, (3) myocardial infarction, (4) congestive heart failure, (5) coronary artery disease, (6) stroke, (7) transient ischemic attack, (8) chronic kidney disease, (9) retinopathy, and (10) peripheral artery disease. 8) You discover the patient is taking an ACE inhibitor. Would you continue this medication throughout the perioperative period? Given the fact that his blood pressure is already poorly controlled, I would continue his ACE inhibitor recognizing that this does increase the risk of intraoperative hypotension. Therefore, I would be prepared to quickly identify and treat any hypotension in order to maintain adequate cerebral perfusion pressure in this patient with carotid artery disease. 9) Considering this patient's hypertension and carotid artery disease, do you think that a reasonable physician would proceed with this case? Given this patient's significant hypertension and carotid artery disease, I would prefer to delay the case to control his blood pressure and obtain a vascular surgeon consult. However, if after discussing the risks and benefits with the surgeon, it was determined that the benefits of timely diagnosis and/or staging of the carcinoma outweighed the risk of cerebrovascular compromise, I would proceed taking the precautions previously discussed.

A 21-year-old female is scheduled for total thyroidectomy to treat a multinodular goiter. She states she has had progressive dyspnea and dysphagia over the last 2 weeks, and stridor is noted during an initial physical exam. Her past medical history is significant for gastroesophageal reflux (GERD), irritable bowel syndrome, and bipolar disorder. Her current medications include lithium, olanzapine, propylthiouracil (PTU), omeprazole, and oral birth control pills. Vital Signs: HR = 122, BP = 158/88 mmHg, RR = 22, Hgb = 13 mg/dL. 1) What are your concerns with this patient? 2) How would your anesthetic management be different for a patient taking lithium? 3) How would you evaluate her for possible airway obstruction? Would you order flow-volume loops? 4) What type of flow-volume loop pattern would you expect to find with this patient? 5) Is this patient euthyroid? How could you tell? 6) Her total T4 is elevated. What do you think about this finding? 7) Assuming her lab work also showed an elevated free T4, would you delay surgery in order to medically treat her hyperthyroidism? 8) Assuming this is an urgent case, how would you optimize this patient's thyroid status for surgery?

I have several concerns in providing care for this patient. First, her thyroid mass combined with progressive dyspnea, dysphagia, and inspiratory stridor is consistent with tracheal compression, which significantly increases the risk and difficulty of airway management. Second, I am concerned her GERD and dysphagia place her at increased risk for aspiration. Third, the presence of tachycardia and hypertension in this patient being treated for hyperthyroidism (propylthiouracil) is concerning, since it may represent inadequate treatment, placing her at increased risk of experiencing a hyperdynamic circulation, cardiac arrhythmias, and/or thyroid storm in the perioperative period (on the other hand, her tachycardia and hypertension may simply represent the patient's reaction to respiratory distress and/or impending surgery). Finally, given this patient's bipolar disorder, I am concerned about patient cooperation and the potential complications associated with lithium treatment and lithium toxicity, such as polyuria, skeletal muscle weakness, ataxia, cognitive changes, widening of the QRS, atrioventricular block, hypotension, and seizures. 2) How would your anesthetic management be different for a patient taking lithium? Given the detrimental side effects of lithium toxicity, I would evaluate the patient for signs of toxicity, such as skeletal muscle weakness, cognitive changes (sedation), ataxia, widening QRS, atrioventricular heart block, hypotension, and seizures. In addition to this examination, I would check the patient's most recent lithium level and/or order lab work to check her current level. Finally, I would avoid any drugs that may lead to toxicity (i.e. thiazide diuretics, NSAIDs, ACE inhibitors), administer sodium containing fluids to prevent excessive renal reabsorption of lithium (reabsorption occurs in the proximal tubule in exchange for sodium), watch the ECG for lithium-induced atrioventricular blockade or dysrhythmias, and closely monitor both anesthetic depth and neuromuscular blockade throughout the case (lithium has the potential to reduce anesthetic requirements and prolong the effects of both depolarizing and nondepolarizing muscle relaxants). 3) How would you evaluate her for possible airway obstruction? Would you order flow-volume loops? Assuming she was adequately oxygenating, I would perform a focused history and physical to determine the severity and onset of her respiratory symptoms, identify any aggravating factors, and note the effects of positioning, if any, on her respiratory function (supine vs. prone vs. upright). I would also review or order airway films and/or CT scan of the neck to identify the location of the mass and assess the extent of airway compression. Finally, I would consider pulmonary function testing to more accurately evaluate her pulmonary function. Flow volume loops may be beneficial in determining the effects of positioning on the airway, whether the obstruction is fixed or variable, and whether the mass is intrathoracic or extrathoracic. However, I would not necessarily order flow-volume loops since a CT scan combined with a careful history and physical in which the patient is observed in both the sitting, supine, and prone positions may provide the needed information, such as tumor location (substernal extension increases the risk of worsening tracheal compression with the loss of respiratory muscle tone), the degree of tracheal compression, and the effects of positioning on mass compression of the trachea. 4) What type of flow-volume loop pattern would you expect to find with this patient? Patients with large thyroid masses usually have a fixed obstruction that may be intrathoracic or extrathoracic in location. Airway flow with this type of obstruction is limited during both inspiration and expiration, with subsequent flattening of both limbs of the flow-volume loop (regardless of whether the obstruction is intrathoracic or extrathoracic). Clinical Note: The flow-volume loop plots the inspiratory and expiratory airflow (Y-axis) against the flow volume (X-axis) during the performance of maximally forced inspiratory (total lung capacity) and expiratory (residual volume) maneuvers. 5) Is this patient euthyroid? How could you tell? While her tachycardia and hypertension may be due to anxiety associated with her respiratory distress and/or the upcoming surgery, these symptoms are concerning since they may represent inadequate treatment and hyperthyroidism. Therefore, I would perform a history and physical exam to identify additional signs of inadequately treated hyperthyroidism, such as diarrhea, warm moist skin, heat intolerance, cardiac arrhythmias, fatigue, skeletal muscle weakness, fine tremor of the hands, and hyperactive tendon reflexes. Additionally, I would order a TSH, free T3, and free T4, recognizing that a hyperthyroid patient would likely present with a low TSH (due to negative feedback) and elevated levels of both free T3 (more active than T4) and free T4. While I would not necessarily order them, other tests to evaluate thyroid function include total T4, total T3, T4 resin uptake, T3 resin uptake, and radioactive iodine uptake. 6) Her total T4 is elevated. What do you think about this finding? While an elevated T4 may indicate hyperthyroidism, this test (or a total T3) should never be used alone to evaluate thyroid function, especially for this patient who is taking an oral contraceptive. While total T4 and T3 levels are often utilized in the evaluation of thyroid function, thyroid hormone protein binding influences these concentrations. Unfortunately, the principal binding protein, thyroxine-binding globulin (TBG), does not remain at a reliably constant level, increasing with acute liver disease, pregnancy, and estrogen containing drugs (such as oral contraceptives), and decreasing with chronic liver disease, nephrotic syndrome, and conditions associated with elevated glucocorticoids. Therefore, total T4 and T3 levels should always be used in conjunction with a test used to assess thyroid hormone binding, such as a T4 or T3 resin uptake test. The information from these tests can then be used to calculate the free T4 index and/or free T3 index, which are proportional to free levels of T4 and T3 respectively. 7) Assuming her lab work also showed an elevated free T4, would you delay surgery in order to medically treat her hyperthyroidism? While I would delay any elective case to achieve a euthyroid state prior to surgery, her progressive dyspnea and dysphagia may make a significant delay in surgical intervention unacceptable. Unfortunately, rendering this patient euthyroid may require an extended amount of time due to potentially large stores of hormones within the thyroid gland (7-14 days with propranolol and iodides; 6-8 weeks with PTU). Therefore, I would discuss my concerns with the surgeon and, if the case was deemed urgent or emergent, consult an endocrinologist, attempt to medically optimize this patient's condition, and proceed to surgery; recognizing that proceeding with surgery when the patient remains hyperthyroid places her at increased risk for perioperative complications such as hemodynamic instability (hyperdynamic circulation), cardiac arrhythmias, and thyroid storm. 8) Assuming this is an urgent case, how would you optimize this patient's thyroid status for surgery? In optimizing this patient's thyroid status for urgent surgery, I would consult an endocrinologist; continue her PTU, which inhibits the organification of iodide, the synthesis of thyroid hormone, and the peripheral conversion of T4 to T3; and administer a beta-blocker (to achieve a normal heart rate), glucocorticoids (to reduce thyroid hormone secretion and the peripheral conversion of T4 to T3), and iopanoic acid (can reduce T3 levels by 50-75% in 6 - 12 hours). Additionally, I would provide adequate hydration and ensure a normal electrolyte balance. I would also consider a small dose of benzodiazepine to relieve anxiety, taking care to avoid respiratory depression in this patient with apparent airway obstruction. My goal would be to minimize the risk of hemodynamic instability, cardiac arrhythmias, and thyroid storm.

6) If it had not been enough time yet, could you check an anti-Xa level or administer protamine to reverse the affects of enoxaparin? 7) Would you require any special monitoring at this point? 8) Would you place a pulmonary artery catheter? 9) You decide to place a central line for fluid management. During placement she develops atrial fibrillation with a pulse of 130. Does this concern you? 10) What will you do? 11) Assume it has been an appropriate amount of time since her last dose of anticoagulant. How would you provide neuraxial anesthesia to control her labor pain? 12) How does aortic stenosis affect the heart?

I would not check an anti-Xa level as a means of weighing the risk of epidural or spinal hematoma because, while it may be used to guide therapeutic dosing, the anti-Xa level is not predictive of hemorrhagic complications. Likewise, I would not administer protamine, since it does not reliably reverse the anti-factor Xa activity of LMWH and is not recommended for the reversal of LMWH. 7) Would you require any special monitoring at this point? Considering this patient's aortic stenosis, the increased risk for bleeding (due to preeclampsia and the acquired von Willebrand syndrome associated with moderate to severe aortic stenosis), and the potential for hemodynamic changes during vaginal delivery (pain-induced sympathetic stimulation, bleeding, and auto-transfusion during uterine involution), I would employ (1) fetal heart rate monitoring (the mother's aortic stenosis and preeclampsia increase the risk of inadequate uterine perfusion), (2) a 5-lead EKG (increased risk for arrhythmia and/or myocardial ischemia), and (3) an arterial line for labor and delivery (peripartum invasive arterial monitoring is recommended in the setting of moderate to severe aortic stenosis). If her enoxaparin had been discontinued at least 24 hours prior, I would consider placing a central venous pressure line to facilitate fluid management. Finally, I would ensure the availability of transthoracic echocardiography to help with determining the cause of any hypotension (e.g. heart failure, hypovolemia, or myocardial ischemia) 8) Would you place a pulmonary artery catheter? Although a pulmonary artery catheter could prove useful for fluid management, identifying the cause of any hypotension (hypovolemia vs. heart failure), and providing a means for pacing should it become necessary, I would not employ it in this situation. First, in the setting of aortic stenosis, the monitor may overestimate the left ventricular end-diastolic volume due to the decreased compliance of the often hypertrophied left ventricle. Second, there is no evidence that utilizing a pulmonary artery catheter improves outcome. Rather, I would utilize a central venous pressure line and transthoracic echocardiography to facilitate fluid management and more accurately diagnose the cause of any cardiovascular instability. Moreover, I would ensure the presence of a defibrillator, adenosine, Diltiazem, a beta-blocker, and Amiodarone in order to treat the development of any supraventricular tachycardia (e.g. atrial fibrillation). 9) You decide to place a central line for fluid management. During placement she develops atrial fibrillation with a pulse of 130. Does this concern you? This does concern me because atrial fibrillation significantly increases the risk myocardial ischemia in patients with aortic stenosis, who already have increased myocardial oxygen demand secondary to concentric hypertrophy (increased muscle mass) and increased afterload. First, the rapid ventricular rate often associated with atrial fibrillation leads to increased myocardial oxygen demand while, at the same time, reducing the time for ejection of stroke volume, coronary perfusion, and left ventricular filling (the latter two occur during diastole). Second, atrial fibrillation eliminates the atrial contribution to ventricular filling (which normally accounts for up to 30-40% of left ventricular filling in the setting of aortic stenosis due to the decreased compliance of the left ventricle), leading to a significant reduction in left ventricular filling and cardiac output. 10) What will you do? I would immediately treat her dysrhythmia, recognizing that atrial fibrillation places this patient with aortic stenosis at increased risk for myocardial ischemia. To this end, I would: (1) pull back the central line, recognizing that the line may have induced the dysrhythmia, and (2) get a 12-lead EKG to confirm a narrow QRS (a QRS <0.12 seconds confirms a supraventricular tachycardia), verify the absence of P-waves (multifocal atrial tachycardia is often mistaken for atrial fibrillation), and identify any myocardial ischemia. Assuming this represented atrial fibrillation and she were stable, I would: (3) administer Diltiazem to slow her ventricular rate (while beta-blockers are another option to slow the ventricular rate, I would avoid them if possible in this asthmatic patient). If at any point she developed hypotension, pulmonary edema, or signs of myocardial ischemia (ST-segment elevation or depression), I would: (4) proceed with cardioversion (biphasic defibrillator- 100-200 joules; monophasic defibrillator-200 joules) Clinical Notes: · Treatment Priorities in the Stable Patient (In order) o Find the cause o Fix the cause o Slow the rate o Convert the rhythm 11) Assume it has been an appropriate amount of time since her last dose of anticoagulant. How would you provide neuraxial anesthesia to control her labor pain? Given this patient's maternal fever and suspected chorioamnionitis, severe aortic stenosis, and multiple sclerosis, I would: (1) ensure this patient with possible bacteremia was receiving the appropriate antibiotics to reduce the risk of epidural abscess and meningitis; (2) ensure adequate hydration to maintain adequate preload; (3) utilize epidural anesthesia for pain control (as opposed to a spinal anesthetic), slowly raising the level of blockade to T10 with small doses of local anesthetic without epinephrine, to avoid the rapid drop in SVR and/or tachycardia that is so poorly tolerated by patients with severe aortic stenosis (epinephrine could lead to tachycardia, which can lead to cardiac ischemia in patients with severe aortic stenosis); and (4) administer a low concentration local anesthetic (0.125% or 0.0625% Bupivacaine), recognizing that the use of higher concentration local anesthetics for neuraxial anesthesia may be associated with an increased risk of exacerbating her multiple sclerosis. 12) How does aortic stenosis affect the heart? Progressive stenosis of the aortic valve leads to concentric ventricular hypertrophy (increased left ventricular wall thickening without dilation), which occurs in response to increased intraventricular systolic pressure. Concentric ventricular hypertrophy results in diastolic dysfunction ("stiff' ventricle), increased left ventricular end-diastolic pressures (the atrial contribution to end-diastolic volume increases from the normal 20% to 30-40%), and increased myocardial oxygen requirements (placing the patient at increased risk for subendocardial ischemia). Eventually, eccentric left ventricular hypertrophy may develop, leading to decreased cardiac output. Clinical Note: · Concentric Ventricular Hypertrophy: o Usually results from pressure overload, o Sarcomeres are added in parallel (thickening rather than lengthening), o Ventricular volume is not increased. · Eccentric Ventricular Hypertrophy: o Usually results from volume overload. o Sarcomeres are added in series (lengthening rather than thickening), o Ventricular volume is increased.

1) What monitors would you place for this case? 2) Does it matter where you place the arterial line? 3) Is a pulmonary artery catheter (PAC) necessary? How would it change your management? 4) How would you induce and secure the airway of this patient? 5) How would you maintain anesthesia following induction? 6) Immediately after placement of the aortic cross clamp, the SSEP signal shows decreased amplitude and increased latency. What would you do? 7) You have the perfusionist increase flows to the lower extremities and the SSEP signal improves. Describe the blood supply to the spinal cord. 8) The surgeon asks you to start rewarming the patient prior to releasing the aortic cross-clamp. Would you utilize forced-air warming? 9) Later in the surgery, the surgeon releases the cross-clamp and the patient's blood pressure drops to 68/44 mmHg. Why do you think this occurred? 10) What would you do? 11) Would you extubate this patient in the operating room?

In addition to the standard ASA monitors, I would require: (1) a 5- lead EKG with ST-segment analysis to monitor for cardiac ischemia (aortic repair is associated with a high incidence of intraoperative myocardial ischemia and arrhythmias); (2) both upper and lower extremity arterial lines to facilitate arterial blood gas sampling and rapid treatment of hemodynamic instability (secondary to hypertension, blood loss, and aortic cross-clamping); (3) a large bore central line to provide access for the safe central administration of vasoactive drugs, the rapid transfusion of fluid and blood products, and the insertion of a pulmonary artery catheter and/or a transvenous pacemaker; (4) a pulmonary artery catheter to assess fluid status and cardiac function; (5) TEE to help confirm the extent and severity of his aortic disease, monitor real-time cardiac function, and identify ischemia (TEE is the most sensitive modality for detecting myocardial ischemia -induced wall motion abnormalities on TEE appear earlier than either ST-segment or PAC waveform changes); (6) core and peripheral temperature monitors, (7) somatosensory and motor evoked potential monitoring to detect spinal cord ischemia, and (8) a Foley catheter to assess urine output and, indirectly, fluid status and renal perfusion. Clinical Notes: o Central Venous Pressure Monitoring: o In the setting of normal right ventricular function, the central venous pressure may give an indirect estimation of right ventricular end-diastolic volume and right ventricular preload. This information may then be used to guide fluid replacement and, where ventricular compliance is normal, to allow estimation of left heart pressures. o Isolated CVP measurements are difficult to interpret and should be assessed rather as a trend analysis, keeping in mind the patient's hemodynamic variables and overall condition. o The CVP is informative only when the pressure is very low or very high because of the relatively increased compliance of the right side of the heart as compared to the left. o Temperature Monitoring o Core Temperature - can be monitored via the bladder, nasopharynx, tympanic membrane, pulmonary artery, or distal esophagus. o Peripheral Temperature - can be monitored via the axilla or the rectum. o MEPs / SSEPs o Neurologic injury is associated with a loss of signal lasting 15-30 minutes, o MEPs may be used to identify critical intercostal arteries supplying the cord, and then confirm successful reimplantation of those same arteries (SSEPs are not adequate for this purpose because the response to ischemia is too slow). o SSEPs are ablated with conduction blockade (i.e. epidural or spinal anesthesia). 2) Does it matter where you place the arterial line? Given the potential for hypertension above the aortic clamp and inadequate perfusion below the aortic clamp, I would place proximal and distal arterial lines to allow for rapid identification and treatment of any hemodynamic instability. Since clamping of the left subclavian artery may become necessary during the procedure (the clamp is often placed proximal to the left subclavian artery for surgery involving the proximal descending aorta), I would place the upper extremity arterial line in the right arm to avoid surgical interference. During aortic cross-clamping, the proximal arterial line would provide more accurate information concerning cerebral and cardiac perfusion pressures as well as cardiac afterload, while the lower extremity arterial line would allow close monitoring of distal perfusion pressure to the kidneys, spinal cord, and mesenteric circulation. 3) Is a pulmonary artery catheter (PAC) necessary? How would it change your management? I would place a PAC in this patient. It would help in fluid management, assessment of cardiac function, and timely identification of cardiac ischemia during the case and throughout the postoperative period. Intra-operatively, I would use the PAC in conjunction with TEE to better identify and manage cardiac ischemia, fluid status, and valvular disease. However, TEE is not usually continued in the postoperative period, and the PAC could be left in place to guide management during this period of considerable cardiac risk (i.e. myocardial ischemia and cardiac failure). Clinical Note: PAC signs of myocardial ischemia Include: o Prominent A-waves - result when the atrium contracts into a stiff left ventricle o Prominent V-waves - result when ischemic affects on the papillary muscles, chordae tendineae, and/or myocardium cause functional mitral regurgitation o Increased pulmonary artery occlusion pressure and increased pulmonary artery diastolic pressure - secondary to ischemia-induced increases in LVEDP 4) How would you induce and secure the airway of this patient? When inducing this patient I would: (1) ensure appropriate intravenous access and monitoring (including invasive monitoring); (2) make sure difficult airway equipment is available (GERD, double-lumen tube placement, potential aneurysmal airway compression, potential diabetic stiff-joint syndrome); (3) administer a bronchodilator to optimize his COPD; (4) ensure adequate beta-blockade to reduce the risk of myocardial ischemia and/or aortic rupture/propagation; (5) provide aspiration prophylaxis (GERD and diabetes); (6) obtain a baseline set of vital signs, PA catheter data, and evoked potential signals; (7) preoxygenate the patient to reduce the risk of hypoxia (coronary artery disease); (8) apply cricoid pressure; and (9) perform a carefully titrated, high-narcotic intravenous induction with the goals of avoiding hemodynamic instability, tachycardia, and bradycardia (the latter would exacerbate his aortic valve insufficiency). 5) How would you maintain anesthesia following induction? Assuming the surgeon desired to employ SSEP and MEP monitoring, I would provide a total intravenous anesthetic in order to avoid anesthetic-induced signal depression. My primary goal during maintenance of anesthesia is to maintain adequate end-organ perfusion, particularly during aortic cross-clamp placement. To this end, I would: (1) maintain his heart rate between 60-80 beats per minute, (2) maintain his cardiac index between 2-2.5 L/minute/m2; (3) provide a stable anesthetic to facilitate the accurate interpretation of evoked potential monitoring; (4) carefully titrate a short-acting neuromuscular blocker to maintain electrical muscle amplitude at approximately 50% of baseline; (5) maintain a systolic blood pressure between 105- 115 mmHg, a mean arterial pressure around 100 mmHg above the cross-clamp, and a mean arterial pressure above 50 mmHg distal to the cross-clamp; and (6) have various short-acting vasoactive agents available to maintain hemodynamic stability, especially during cross-clamp application and removal (i.e. nitroglycerine, nicardipine, esmolol, and nitroprusside). 6) Immediately after placement of the aortic cross clamp, the SSEP signal shows decreased amplitude and increased latency. What would you do? Assuming a stable anesthetic, the institution of partial bypass, and a significant change in the SSEP signal (50% decrease in amplitude and/or 10% increase in latency), I would: (1) optimize the patient's hemodynamics (i.e. adequate MAP), (2) check an ABG, and (3) ask the perfusionist to both increase pump flows distal to the aortic clamp and correct any metabolic disturbances (i.e. hypo/hypercarbia and acidosis). If this were unsuccessful, I would: (4) ask the surgeon to release or reposition the aortic clamp (repositioning may be effective if it resulted in renewed flow through a critical intercostal artery), (5) ensure adequate hypothermia at 30-34 °C (Class Ila recommendation), withdraw 10-20 cc of CSF from the lumbar drain to achieve a target ICP of 8-10 mmHg (Class I recommendation for patients at significant risk for neurologic injury during thoracic aneurysm repair), and consider pharmacological intervention. Various drugs that potentially reduce the incidence of spinal cord ischemia include corticosteroids (i.e. methylprednisolone or dexamethasone), naloxone, dextrorphan, magnesium, intrathecal papaverine, naloxone, and calcium channel blockers. note: papaverine mechanism: inhibition of the enzyme phosphodiesterase, causing an elevation of cyclic AMP levels. Side effects of papaverine treatment include arterial hypotension, ventricular tachycardia, flushing of the face, tachycardia, and paradoxical aggravation of cerebral vasospasm Clinical Notes: o A 50% decrease in amplitude of the MEP signal is considered clinically significant (some sources suggest that a "significant" decrease in MEP amplitude requires a 75-80% change). o Hypothermia: o Reduces oxygen requirements by 5-7% for each °C decrease in temperature o In the setting of aortic arch or ascending aorta surgery, deep hypothermic cardiac arrest is required (15°C) o Both systemic and regional cooling are beneficial o Systemic hypothermia - achieved with foil CPB or partial bypass (active cooling) or by allowing the patient to cool passively to 30-34°C o Regional cooling - achieved with the epidural infusion of 4°C saline o CSF Drainage o Spinal cord perfusion = distal mean aortic pressure - CSF pressure or the central venous pressure (whichever is greatest) o CSF drainage is important because CSF pressure may increase by 10-15 mmHg with cross-clamping of the descending aorta. o Low CSF pressures should be maintained postoperatively, raising the pressure back to 10-12 after 48 hours, and then to 12-15 when there is confirmation of preserved motor function. o Complications associated with CSF drainage include: Headache, Spinal/epidural hematoma, Intracranial bleeding secondary to tearing of cerebral bridging vessels, Meningitis, Persistent CSF leak 7) You have the perfusionist increase flows to the lower extremities and the SSEP signal improves. Describe the blood supply to the spinal cord. Two posterior spinal arteries supply the posterior 1/3rd of the spinal cord (sensory). A single anterior spinal artery arising from the basilar and vertebral arteries supplies the anterior 2/3rd of the spinal cord (motor function), receiving contributions from 6-8 transverse radicular arteries (originating from the aorta at variable locations), the most important being the artery of Adamkiewicz. Autoregulation of spinal cord blood flow is relatively constant between 50-125 mmHg, but may be ablated in the setting of hypoxia or hypercarbia. The artery of Adamkiewicz serves as the major supply to the anterior, lower 2/3rd of the spinal cord. It is usually located on the left side, originating anywhere from L5-T5 (T9-T12 60% of the time). This variation in the origin of the Artery of Adamkiewicz may explain why paraplegia may even occur with infrarenal aortic aneurysm repair. When the aortic cross-clamp is applied distal to this radicular artery, the risk of spinal cord ischemia is extremely low. While distal aortic perfusion (i.e. left-heart-bypass) helps maintain spinal cord perfusion below the artery of Adamkiewicz during aortic cross-clamping, it does little to maintain perfusion above this radicular artery (above the artery, but still below the aortic clamp), because resistance to flow moving up the anterior spinal artery (proximal to the artery of Adamkiewicz) is 51 times greater than that going down the anterior spinal artery. 8) The surgeon asks you to start rewarming the patient prior to releasing the aortic cross-clamp. Would you utilize forced-air warming? I would suggest that actively warming the patient via the left heart bypass circuit would be more effective. However, if forced air warming were utilized in conjunction with the bypass circuit (which contains a heat exchanger), I would ensure it was only applied to the patient's upper body. Forced air warming of the lower body is contraindicated because warming ischemic tissue increases metabolic requirements, acidosis, and ischemic injury. 9) Later in the surgery, the surgeon releases the cross-clamp and the patient's blood pressure drops to 68/44 mmHg. Why do you think this occurred? The timing of his hypotension suggests that his hypotension is most likely the result of cross-clamp release. The primary cause of hypotension following cross-clamp release is central hypovolemia and the resulting decrease in cardiac preload. The application of the cross-clamp to the aorta leads to distal tissue ischemia and vasoactive mediator release, with a subsequent drop in systemic vascular resistance, increased venous capacitance distal to the clamp, and increased capillary permeability (the latter develops by the end of the cross-clamp period). The increased venous capacitance leads to a distal shift of blood volume, while the increased capillary permeability results in a loss of intravascular fluid, both of which contribute to central hypovolemia. Moreover, central hypovolemia is further exacerbated by blood loss during the procedure. With removal of the cross-clamp, acid metabolites and vasoactive mediators are released from tissues distal to the clamp, leading to decreased myocardial contractility (which is further compromised secondary to the reduced preload associated with central hypovolemia), increased pulmonary vascular resistance (secondary to the released acid metabolites and vasoactive mediators), and increased capillary permeability (which further contributes to central hypovolemia). 10) What would you do? I would: (1) start a fluid bolus, (2) place the patient in the trendelenburg position, (3) administer a vasopressor, and (4) attempt to identify any other potential causes or contributing factors to hypotension, such as hemorrhage (low filling pressures, surgical and/or anesthesiologist observation), arrhythmia, anesthetic-induced cardiovascular depression, myocardial ischemia (wall-motion abnormalities, EKG changes, elevated filling pressures), tension pneumothorax, metabolic and electrolyte abnormalities, citrate-induced hypocalcemia, and hypothermia. If the hypotension persisted, I would: (5) ask the surgeon to reapply the aortic cross-clamp, (6) correct any abnormalities (i.e. anemia, arrhythmia, myocardial ischemia, tension pneumothorax, and metabolic abnormalities), (7) decrease the depth of anesthesia (particularly volatile agents), and (8) volume load the patient (vasodilators such as nitroprusside and/or nitroglycerin may facilitate volume loading, but these should be discontinued before unclamping). I would then (9) ask the surgeon to gradually release the cross-clamp to prevent further hypotension by allowing time for physiologic compensation. 11) Would you extubate this patient in the operating room? I would not attempt to extubate the patient in the operating room because of his significant pulmonary disease (FEV1 = 78% predicted; FEV1/FVC = 65%), the deleterious effects of this type of surgery on pulmonary function (one-lung ventilation, surgical manipulation of the diaphragm and the lungs, and phrenic nerve and recurrent laryngeal nerve injury), and the potential for airway and pulmonary edema following thoracic aortic repair (secondary to significant fluid administration and the increased capillary permeability associated with aortic cross-clamping). Rather, I would: (1) transport the patient to the ICU; (2) allow sufficient time for hemodynamic stabilization; (3) ensure complete reversal of neuromuscular blockade; (4) confirm adequate respiratory function (i.e. vital capacity > 10 mL/kg, tidal volume > 6 mL/kg, negative inspiratory force > 20 cm H2O, PaO2 > 60 with FiO2 < 50%, PaCO2 < 50 mmHg, and respiratory rate < 30 breaths/minute); (5) verify normothermia and adequate pain control; (6) perform a "cuff leak test" (auscultate for breath sounds with the cuff deflated and the patient breathing spontaneously); and (7) extubate the patient when he was awake and cooperative with an intact gag reflex.

A 62-year-old, 87 Kg, male presents for open repair of a dissecting aneurysm in the descending thoracic aorta. A 3/6 early diastolic, high-pitched murmur is heard over the left sternal border. His history includes 45 years of tobacco use, hypertension, diabetes, severe GERD, and the placement of a drug-eluting coronary stent three months ago. His medications include atenolol, hydrochlorothiazide, simvastatin, aspirin, and sublingual nitroglycerine. Vital Signs: HR = 106, BP = 152/85 mmHg, RR = 24, T = 37 °C, Hct = 32. 1) What are your perioperative concerns in providing anesthesia for this case? 2) What is the DeBakey classification of aortic dissection? Is it important? 3) What is the significance of his diastolic murmur? 4) A family member tells you that the patient had rheumatic fever as a child, and has lived with a leaky aortic valve for years. Would you continue his beta-blocker? 5) Would you initiate beta-blocker therapy if he were not already taking a beta-blocker? 6) How would you evaluate his pulmonary status? 7) Would you place a lumbar drain? 8) You are planning to place the lumbar drain and an epidural catheter for pain management. Upon further inquiry, you learn that the patient did discontinue his clopidogrel seven days ago, but was started three days ago on eptifibatide (integrilin) and intravenous heparin. Would you still proceed with the placement of these neuraxial catheters? 9) The surgeon tells you he is planning to utilize left-heart bypass with a pump, oxygenator, and heat exchanger during the procedure. Would you still place the lumbar drain? 10) The surgeon says he wishes to use MEP and SSEP monitoring for the case. Given this information, would you place an epidural for the procedure? 11) You are appropriately concerned about the potential for massive blood loss. Recognizing this risk, what would you do? 12) The surgeon wants to employ acute normovolemic hemodilution (ANH). Would you agree? 13) Describe how you would perform acute normovolemic hemodilution.

My primary concerns in managing this patient include the risk for: (1) aneurysmal rupture; (2) propagation of the aneurysm (potentially leading to aortic insufficiency and/or cardiac tamponade), (3) myocardial ischemia/infarction (secondary to coronary artery disease, hypertension, recent stent placement, likely aortic regurgitation, possible aneurysmal involvement with the coronary arteries, perioperative hemodynamic instability, cross-clamp application, and potential massive blood loss); (4) post-operative respiratory complications (secondary to long term tobacco abuse, one-lung ventilation, and surgical manipulation of the diaphragm and lungs); (5) paraplegia (secondary to aneurysmal/surgical disruption of radicular arteries supplying the anterior spinal cord, cross-clamp application, hemodynamic instability, potential massive blood loss, and hyperglycemia secondary to diabetes mellitus); (6) post-operative acute kidney injury (secondary to cross-clamp application, diabetes mellitus, aneurysmal/surgical disruption of the renal arteries, hemodynamic instability, and potential massive blood loss); (7) visceral/mesenteric injury (secondary to cross-clamp application, hemodynamic instability, anemia, and aneurysmal/surgical disruption of the superior/inferior mesenteric arteries and/or the celiac trunk) (8) stroke (secondary to aneurysmal/surgical disruption of the left common carotid or innominate arteries, embolization of air or thrombotic material, hypertension, hemodynamic instability, and cross-clamp application); (9) difficult airway management (secondary to possible aneurysmal compression of the trachea, lung parenchyma, or superior vena cava -> tracheal deviation, rapid desaturation, and airway edema; diabetes mellitus possible diabetic stiff joint syndrome; double lumen tube placement; GERD; and large fluid shifts possible post-operative airway edema); (10) hemorrhage (secondary to aneurysm rupture, surgical trauma, coagulopathy, heparinization, hypothermia, and acidosis); and (11) congestive heart failure (secondary to myocardial ischemia; likely aortic regurgitation; and overaggressive fluid administration). 2) What is the DeBakey classification of aortic dissection? Is it important? Due to the strong correlation between clinical outcome and the extent of aortic dissection, several classification systems, such as the DeBakey, Stanford, and Crawford systems, have been developed. The DeBakey classification system defines type I dissections as originating in the ascending aorta and extending distally to involve the descending aorta (thoracic and/or abdominal aorta). Type II dissections originate in the ascending aorta and do not extend beyond the innominate artery. Type III dissections originate beyond the left subclavian artery and extend distally to the diaphragm (type IIIA), or to the aorto-iliac bifurcation (type IIIB). Acute aortic dissection involving the ascending aorta (DeBakey type I and II) is a surgical emergency, requiring immediate surgical repair. Whereas, acute dissection involving the descending aorta (DeBakey type III), is most often treated medically with blood pressure and pain control. However, type III dissections may require surgical intervention if they develop significant aneurysmal dilatation, are at risk of impending rupture, or result in end-organ ischemia. Clinical Notes: · Anatomical Division of the Aorta o Ascending Aorta - between the aortic valve and the innominate artery o Aortic Arch - between the innominate and the left subclavian artery o Descending thoracic aorta - between the left subclavian and the diaphragm o Abdominal aorta - below the diaphragm · Stanford Classification o Type A § Involvement of the ascending aorta with or without involvement of the arch or descending aorta § Includes DeBakey Types I and II § More common than Type B o Type B § All cases in which the ascending aorta is not involved § Includes DeBakey Types Illa and I lib · Crawford Classification System of Thoracoabdominal Aortic Aneurysms o Type 1: Originates below the left subclavian artery and extends into the abdominal aorta, including the celiac axis and mesenteric arteries o Type II: Involves the same area as Type I, with extension caudally to include the infrarenal abdominal aorta o Type III: Originates in the lower descending thoracic aorta (below the 6th rib)and involves the remainder of the aorta (the origin is lower in the descending thoracic aorta than with Type I or Type II) o Type IV: Originates at the diaphragm and involves the abdominal aorta only o Type V: Originates below the 6th rib (lower thoracic aorta) and extends to the renal arteries 3) What is the significance of his diastolic murmur? This diastolic murmur could represent a previous cardiac condition or propagation of the aortic dissection into the aortic valve with resultant aortic regurgitation. Acute aortic regurgitation produces volume overload of the left ventricle due to added regurgitant volume; reduces the effective forward stroke volume; may result in pulmonary edema due to rapidly increasing left ventricular end-diastolic pressures; increases myocardial oxygen demand; and results in reduced myocardial blood supply secondary to reduced diastolic pressures in the aorta and/or increased ventricular end-diastolic pressures. As a result of these factors, myocardial ischemia may occur even in the absence of coronary artery disease. In the case of aortic valve insufficiency, bradycardia should be avoided, since increased diastolic time leads to increased regurgitant volume and worsening cardiac function. 4) A family member tells you that the patient had rheumatic fever as a child, and has lived with a leaky aortic valve for years. Would you continue his beta-blocker? I would continue his beta-blocker therapy for two reasons: first, the preoperative discontinuation of beta-blocker therapy is associated with an increased risk of myocardia ischemia and chest pain; second, it is important to control his heart rate and blood pressure in order to avoid propagation and/or rupture of his aortic dissection (decreasing blood pressure and the force of ventricular contractions reduces aortic wall stress by reducing shear pressure or dP/dT*). Before administering additional beta-blocker (beyond his normal dose), however, I would correct any factors that may be contributing to his tachycardia and hypertension, such as pain, hypovolemia, infection, or anemia. If this failed to resolve the problem, I would consider starting an infusion of a short acting, titratable, beta-blocker such as esmolol, followed by nitroprusside. My treatment goals would be to reduce intramural pressures and aortic shear forces that could lead to propagation or rupture of the dissection; reduce regurgitant volumes by reducing left ventricular afterload; and, at the same time, avoid bradycardia, which could increase aortic regurgitation by increasing diastolic time. Additionally, perioperative beta-blockers have been shown to reduce the perioperative and long-term cardiac morbidity/mortality associated with high-risk vascular surgery. Moreover, I would start the esmolol infusion prior to giving nitroprusside, since the administration of a vasodilator prior to adequate beta-blockade could lead to increased aortic wall stress. The advantage of a short acting agent, like esmolol, is that it would allow me to quickly adjust dosing to fit a rapidly changing clinical situation (i.e. reduce or discontinue infusion if bradycardia developed). Clinical Note: *dP/dT = the change in blood pressure / change in time 5) Would you initiate beta-blocker therapy if he were not already taking a beta-blocker? I would not initiate high-dose, untitrated, beta-blocker therapy to a patient not currently taking beta-blockers unless it was absolutely necessary. While this intervention may reduce the risk of aneurysm rupture, aneurysm propagation, and the patient's cardiovascular morbidity and mortality, it would also increase the incidence of hypotension, bradycardia, and stroke, along with an overall increase in mortality. Rather, if given the opportunity, I would initiate beta-blocker therapy at least 7-10 days prior to surgery, and carefully titrate dosing to achieve a heart rate 60-80 beats/minute while avoiding significant bradycardia and/or hypotension. Clinical Note: In patients undergoing noncardiac surgery who are not currently taking betablockers, routine administration of high-dose, untitrated perioperative betablockers is not recommended (ACCF/AHA Class III Recommendation). 6) How would you evaluate his pulmonary status? Recognizing that aortic repair is associated with a high incidence of postoperative respiratory failure (25-45%), I would: (1) order a chest x-ray to aid in identifying any pulmonary disease or underlying infection; (2) perform a physical exam to identify any signs and symptoms consistent with COPD, such as cough and sputum production; and (3) obtain any relevant history, such as frequency of pulmonary infections, frequency and severity of exacerbations, exercise tolerance, number and course of hospitalizations, and efficacy of past treatments. If the patient were minimally symptomatic, exhibited good functional status, and did not show any signs of an acute pulmonary process on chest x-ray, I would not pursue further pulmonary testing. However, if the patient exhibited signs and symptoms consistent with significant pulmonary disease or showed an acute pulmonary process on chest x- ray, I would obtain: (4) an ABG to better assess ventilation and oxygenation; (5) an ECG to identify any signs of pulmonary hypertension (i.e. narrowly split heart sound, a loud second heart sound secondary to pulmonic valve closure, and right ventricular and/or atrial hypertrophy); and (6) pulmonary function tests to better evaluate the severity of his COPD, determine his response to bronchodilator therapy, predict his tolerance of one-lung ventilation, and assess his risk for postoperative pulmonary complications. Clinical Note: o PFT results consistent with an increased risk for postoperative mechanical ventilation include: o FEV1<2L total o Maximum breathing capacity < 50% of predicted o MMEFR < 50 % of predicted 7) Would you place a lumbar drain? Assuming that this patient with a drug-eluting stent had discontinued any thienopyridine therapy for the appropriate interval (clopidogrel = 7 days; ticlopidine = 14 days) and exhibited no signs of coagulopathy or other contraindication, I would consider placing a lumbar drain. Placement would allow for passive drainage of CSF to a pressure of 8-10 mmHg during the procedure and postoperatively (for about 48 hours) to better preserve adequate spinal cord perfusion. Perfusion pressure to the spinal cord is defined as the mean distal aortic pressure minus the CSF pressure or central venous pressure (whichever is highest). While spinal cord blood flow is controlled by autoregulation (between pressures of 50-125 mmHg), the potential for impaired autoregulation, combined with clamp-induced increases in CSF pressure (hyperemia above the proximal clamp increased ICP -> redistribution of CSF into the intrathecal space -> increase in CSF pressure by 10-15 mmHg), make the careful drainage of CSF a reasonable strategy to improve spinal cord perfusion (Class I recommendation for spinal cord protection for patients at high risk for neurologic injury). However, there are potential complications associated with CSF drainage, including spinal/epidural hematoma, headache, intracranial bleeding (secondary to tearing of cerebral bridging vessels), meningitis, and chronic CSF leakage. Clinical Note: When draining CSF to a specific target, be sure that the zero point of the transducer is set at the patient's mid-axillary line or external auditory meatus while the patient is in the supine position. 8) You are planning to place the lumbar drain and an epidural catheter for pain management. Upon further inquiry, you learn that the patient did discontinue his clopidogrel seven days ago, but was started three days ago on eptifibatide (integrilin) and intravenous heparin. Would you still proceed with the placement of these neuraxial catheters? I would still proceed with lumbar drain placement as long as he had discontinued his eptifibatide and intravenous heparin for at least 4 hours, he had a normal PTT (heparin activity would prolong the PTT), and there were no signs of coagulopathy (the ASRA recommendation is to delay neuraxial placement for 4-8 hours following the discontinuation of eptifibatide and 2-4 hours following the discontinuation of intravenous heparin). Platelet GP IIb/IIIa inhibitors are sometimes used for "bridging therapy" when a Patient who requires thienopyridine therapy to prevent stent thrombosis must undergo a surgical procedure where the risk of bleeding makes the continuation of a long- acting platelet inhibitor unacceptable. When the decision is made to employ bridging therapy, a typical strategy would be to: (1) discontinued the thienopyridine 5-7 days prior to surgery (7 days would be required if planning to place an epidural catheter or lumbar drain), (2) continue aspirin throughout the perioperative period, (3) start a short-acting platelet inhibitor, such as eptifibatide or tirofiban, 2-3 days before surgery, (4) consider starting a concomitant heparin infusion, and (5) discontinue any "bridging" drugs 6 hours prior to surgery (the aspirin should be continued, if possible). Clinical Note: Heparin alone is insufficient to prevent stent thrombosis because the heparin- antithrombin complex's ability to inactivate fibrin-bound thrombin and factor Xa is limited. 9) The surgeon tells you he is planning to utilize left-heart bypass with a pump, oxygenator, and heat exchanger during the procedure. Would you still place the lumbar drain? Even though left-heart bypass requires heparinization, I would plan to proceed with lumbar drain placement for this patient at high risk for neurologic injury. After discussing the risks and benefits of drain placement with the surgeon, patient, and family, I would ensure that preoperative anticoagulants had been discontinued for a sufficient period of time, rule out any current coagulopathy, delay systemic heparinization for 60 minutes following placement of the lumbar drain, utilize the smallest amount of heparin necessary to achieve therapeutic objectives (partial bypass usually requires about 100 U/kg), monitor the patient carefully for any signs of spinal or epidural hematoma, and ensure adequate coagulation at the time of catheter removal. In the case of traumatic placement, a delay of surgery for 24 hours would be desirable to reduce the risk of spinal/epidural hematoma (there is no evidence to support this practice; ASRA recommends a delay when utilizing full heparinization, but does not make a recommendation when planning low dose systemic heparinization - i.e. 100 U/kg). However, the urgency of this case may preclude such a delay, making it essential that post-operative precautions be taken to avoid an undetected epidural or subdural hematoma. To this end, I would ensure normal coagulation prior to removal of the lumbar drain and order neurological examinations every hour to detect the first signs of spinal cord compression secondary to hematoma formation. If an epidural catheter were in place for post-operative pain control, I would utilize narcotics alone or in combination with low concentration local anesthetics in order to preserve the patient's lower extremity motor function and allow for adequate neurologic monitoring. 10) The surgeon says he wishes to use MEP and SSEP monitoring for the case. Given this information, would you place an epidural for the procedure? Assuming there were no contraindications, I would consider placing an epidural for this case. However, since the surgeon wishes to utilize evoked potential monitoring during the procedure, I would use only epidural narcotics during the case, reserving the epidural administration of local anesthetics for postoperative care. In this way, I would avoid the confounding affects of local anesthetic- induced sensory and motor impairment on evoked potential signals. Since this patient will require systemic heparinization, I would rule out any current coagulopathy, delay systemic heparinization for 60 minutes following placement, minimize heparin dosing, and plan to remove the epidural only after the complete restoration of motor function (to avoid diagnostic confusion) and normal coagulation. While the use of epidural analgesia has not been definitively proven to reduce the incidence of cardiovascular, pulmonary, or renal complications, it does provide excellent post-operative pain control and may result in improved respiratory function (decreased incidence of atelectasis, pulmonary infections, respiratory failure, and prolonged mechanical ventilation), improved gastrointestinal motility, improved graft patency (reduced coagulation response), and reduced incidence of postoperative myocardial ischemia (secondary to an attenuation of the stress response and, when a thoracic epidural is used, coronary artery dilation). However, there are risks associated with placement such as epidural hematoma, sympatholysis-induced hypotension, epidural abscess, headache, meningitis, and interference with spinal cord monitoring and/or postoperative neurological exam. 11) You are appropriately concerned about the potential for massive blood loss. Recognizing this risk, what would you do? Recognizing the potential for massive blood loss during the procedure, I would: (1) place at least two large-bore intravenous lines (in addition to a large-bore central introducer); (2) ensure that rapid transfusing and blood warming devices were in the room; (3) have 10-15 units of PRBCs and 5 units of FFP in the operating room (1 unit of FFP should be available for every 1-2 units of PRBCs) with additional units available in the blood bank; and (4) plan to utilize intraoperative cell salvage. Moreover, I would have a discussion with the surgeon concerning the risks and benefits of additional blood conservation strategies, such as acute normovolemic hemodilution and/or the administration of an antifibrinolytic, such as e-aminocaproic acid or tranexamic acid. 12) The surgeon wants to employ acute normovolemic hemodilution (ANH). Would you agree? Assuming the patient's hematocrit was > 33%, his hemoglobin was > 11 g/dL, his smoking has not led to severe pulmonary disease, and there is no significant renal impairment (possibly secondary to diabetes and/or hypertension), I would agree to utilize acute normovolemic hemodilution as a blood conservation strategy recognizing that avoiding exposure to allogeneic blood transfusion reduces the risk of infection, transfusion reactions, and red cell alloimmunization. The rationale for ANH is that blood loss is minimized by reducing the hematocrit of blood likely to be shed during the procedure (i.e. normovolemic hemodilution). Autologous blood collected at the beginning of the case prior to hemodilution (higher hematocrit) is then re-infused when significant bleeding has been controlled. Unfortunately, this strategy has only been shown to be of moderate benefit, saving the equivalent of 1 -2 units of PRBCs even when the patient's initial hematocrit is very high and intraoperative blood loss is substantial (> 70% of the patient's blood volume). In this case, his cardiac disease may limit the reduction in initial hematocrit, further limiting the benefits of this approach. Clinical Note: Ischemic cardiac disease is not necessarily a contraindication to ANH, but the initial reduction in hematocrit should be limited in order to avoid end-organ ischemia. Contraindications to ANH o Anemia (initial hematocrit is < 33% or the hemoglobin is < 11 g/dL) o Impaired renal function - the patient may be unable to excrete the fluid load associated with normovolemic hemodilution o Conditions that would make an increase in cardiac output undesirable, such as aortic stenosis o Significant pulmonary disease - oxygen delivery to tissues may be inadequate in the setting of significant pulmonary disease and the decreased oxygen content associated with ANH o Pre-existing coagulopathy 13) Describe how you would perform acute normovolemic hemodilution. After ensuring the patient was a good candidate for ANH, I would collect 1-2 units of his blood, while simultaneously administering warmed crystalloids or colloids to maintain normovolemia. Given this patient's coronary artery disease, I would only allow his hematocrit to be reduced to around 27% with the initial withdrawal (may use the "estimated allowable blood loss" calculation to approximate the amount of blood to withdraw). Then, following the cessation of significant blood loss (or earlier if clinically indicated), I would re-infuse the autologous blood in reverse order of collection, recognizing that the first unit collected contains the highest concentration of coagulation factors and platelets (re infusion in reverse order of collection is only advantageous if all the blood collected is to be returned to the patient). Clinical Note: o The reduction in RBC concentration associated with ANH decreases blood viscosity, leading to decreased peripheral vascular resistance and increased cardiac output. This compensatory increase in cardiac output helps to maintain adequate oxygen delivery to the tissues despite a decreased hematocrit.

difference between: abciximab vs apixaban

Note: I often get abciximab (gp2b3a inhibitor) mixed up with apixaban (xa inhibitor aka eliquis)

There does not appear to be any R-on-T phenomenon, but the PVCs persist. What will you do?

PVCs are relatively common during anesthesia, especially in the setting of preexisting cardiac disease, and don't always require treatment. However, I would initiate treatment if his PVCs were frequent (more than 3 PVCs per minute), polymorphic, occurred in runs of three or more, or demonstrated an R-on-T phenomenon, recognizing that these conditions are associated with an increased incidence of ventricular tachycardia and/or ventricular fibrillation. If treatment were necessary, I would begin with identifying and eliminating any causative factors such as arterial hypoxemia, myocardial ischemia, hypokalemia, hypomagnesemia, sympathetic activation, and/or mechanical irritation (such as may occur from a central venous line or a pulmonary artery catheter). I would also discontinue any drugs that are pro-dysrhythmic or prolong the QT interval. Additionally, I would ensure the presence of a defibrillator, in case a life-threatening dysrhythmia should develop. If the patient became symptomatic or hemodynamically unstable, I would consider overdrive pacing and/or an antidysrhythmic, such as amiodarone, procainamide, a beta-blocker, or sotalol (the latter should not be used if the QT is prolonged because it can induce Torsades). If the patient developed ventricular tachycardia, I would immediately cardiovert (unstable) or administer amiodarone (stable).Clinical Note: More than 5-6 PVCs per minute preoperatively is associated with increased perioperative morbidity.

A 51-year-old, 81 Kg, man presents to the trauma suite following a motor vehicle accident, during which he was hit from behind and pushed into another car in front of him, causing significant damage to the front and rear of his car. A quick examination reveals an alert patient with a broken nose, possible left femur fracture, and likely several rib fractures. His medical history includes HTN, moderate asthma, GERD, and CAD. Although he has never had a heart attack, he did have a coronary stent placed 6 months ago. His medications include lisinopril, Plavix, Lipitor, Advair, and albuterol. Vital Signs: P = 66, BP = 117/64 mm Hg, RR = 24, T = 37.1 °C 1) During the initial examination, it is determined that the patient is unable to move his extremities and he complains of difficulty breathing. What do you think may be going on? 2) Assuming this was acute cervical spinal cord injury, what are your concerns in relation to this type of injury? 3) Would you administer steroids? 4) What monitoring would you employ if this patient requires cervical spine surgery? 5) Given the patient's apparent head injury, would you initiate hyperventilation prior to intubation? 6) The patient's dyspnea is significant and you decide to intubate him. On airway exam you note a Mallampati score of II. How will you proceed with intubation? 7) You utilize airway blocks to provide analgesia for the airway, but are unable to successfully intubate the patient with fiberoptic assistance due to the impairment of adequate visualization by blood in the oropharynx. You are considering a different plan when you notice premature ventricular contractions (PVCs) on the EKG. What is the "R-on-T phenomenon"? 8) There does not appear to be any R-on-T phenomenon, but the PVCs persist. What will you do? 9) You pull back the central line and the PVCs resolve. A few minutes later, you notice his blood pressure has dropped to 84/36 mmHg. What is your differential for his hypotension? 10) What is spinal shock? 11) The patient remains dyspneic and is yet to be intubated. Assuming the airway exam is reassuring despite the facial trauma and blood in the oropharynx, would you perform a rapid sequence induction with succinylcholine? 12) Is the administration of succinylcholine acceptable with acute spinal cord injury? 13) You attempt to intubate the patient and are unable to visualize the cords under direct laryngoscopy. Would you remove the c-collar in order to improve the view?

Tips: break apart sxs and list causes of each and combined. Considering his recent motor vehicle accident and facial injuries, I would be very concerned that his extremity weakness and dyspnea were related to cervical spine injury above the level of C6. However, I would also consider other factors that could be causing or contributing to his dyspnea such as: (1) pneumothorax, possibly secondary to his rib fractures; (2) pulmonary edema, secondary to cardiac tamponade, myocardial ischemia (CAD, recent coronary stent placement, chronic hypertension), pulmonary embolism, and/or neurogenic pulmonary edema (head injury or cervical spine injury -> sympathetic activation systemic vasoconstriction -> decreased left ventricular compliance and increased left atrial pressure neurogenic pulmonary edema); (3) pulmonary embolism, secondary to his long bone fracture (fat embolism) or venous thrombosis (loss of lower extremity vasomotor tone can lead to arterial and venous pooling, with subsequent thrombosis); and/or his (4) asthma. Clinical Note: · Injuries at the level of C3-5 affect diaphragmatic function; injuries at C6-7 affect chest wall innervation, which can lead to paradoxical respiratory motion and an inability to cough affectively or clear secretions (these affects ultimately result in atelectasis and infection). 2) Assuming this was acute cervical spinal cord injury, what are your concerns in relation to this type of injury? I have several concerns, including: (1) respiratory dysfunction, secondary to loss of diaphragmatic function (C3-C5) (which would make mechanical ventilation necessary) and/or intercostal and abdominal muscle function (C5- T7) (which may also make ventilatory support necessary, depending on the extent of compromise); (2) hypotension, secondary to the loss of sympathetic vascular tone below the level of injury and the loss of cardioaccelerator fibers (T1-T4); (3) pulmonary aspiration, since GERD, paralytic ileus (secondary to trauma and/or cervical spinal cord injury), impaired protective airway reflexes (secondary to cervical spinal cord injury), and possible recent food ingestion increase his risk for aspiration pneumonitis; (4) worsening spinal cord injury, which can result secondary to inadequate spinal stabilization, hypotension, acidosis, hypoxia, and/or anemia; (5) difficult airway management, secondary to respiratory dysfunction (i.e. decreased functional residual capacity), the need for in-line stabilization, and increased risk of aspiration; (6) thermal regulation, since the loss of vasoconstriction and temperature sensation below the level of the injury causes these patients to become poikilothermic; (7) arrhythmias, which are more likely secondary to the autonomic dysfunction often associated with spinal cord injury; (8) end-organ ischemia (i.e. cerebral or myocardial), which may result secondary to elevated intracranial pressure, coronary artery disease, hypotension, acidosis, hypoxia, and/or anemia; and (9) patient positioning injury, which is more likely due to his unstable cervical spine and a lack of sensation below the level of injury. As time progressed, I would be concerned about (10) autonomic hyperreflexia (a condition that develops in 85% of patients following the resolution of spinal shock); (11) a susceptibility to hyperkalemia following the administration of succinylcholine; (12) an increased risk for developing pneumonia, secondary to an impaired ability to cough and clear secretions; and (13) an increased risk for DVT and pulmonary embolism. 3) Would you administer steroids? I would not administer high dose steroids (i.e. methylprednisolone) for acute spinal cord injury, recognizing that this practice is no longer recommended due to the lack of evidence demonstrating any clinical benefit. There is, on the other hand, substantial evidence (Class I, II, and III) indicating that high dose methylprednisolone administration in this population of patients is associated with a number of adverse side effects including infection, gastrointestinal bleeding, respiratory compromise, and even death. Other complications associated with steroid administration in general include fluid retention, hypertension, electrolyte imbalances, hyperglycemia, impaired wound healing, and immunosuppression (i.e. increased rates of sepsis and pneumonia). 4) What monitoring would you employ if this patient requires cervical spine surgery? Given this patient's increased risk for myocardial (CAD, HTN), cerebral (possibly elevated ICP), and spinal cord (neck motion in the setting of an unstable cervical spine can lead to narrowing of longitudinal blood vessels supplying the cord) ischemia, combined with the potential for hemodynamic instability (HTN, loss of sympathetic vascular tone and cardioaccelerator fibers below the level of spinal cord injury), I would require the standard ASA monitors along with: (1) a 5- lead EKG, to monitor for signs of cardiac ischemia; (2) an arterial line, to provide continuous hemodynamic monitoring and facilitate intermittent arterial blood gas and hematocrit analysis; (3) a central venous pressure catheter, to provide for volume resuscitation and the measurement of right atrial filling pressures; (4) an esophageal temperature probe, to ensure normothermia in this poikilothermic patient; and a (5) Foley catheter, to facilitate bladder emptying and fluid management. Moreover, I would have a discussion with the surgeon and electrophysiologist about (6) neuromonitoring (SSEPs and/or MEPs) and (7) pulmonary artery catheter placement. 5) Given the patient's apparent head injury, would you initiate hyperventilation prior to intubation? While hyperventilation would likely reduce his intracranial pressure by inducing cerebral vasoconstriction, I would prefer not to initiate it due to the risk of inducing cerebral and/or spinal cord ischemia (hypocarbia decreases spinal cord blood flow as well as cerebral blood flow; patients with head trauma often experience a reduction in cerebral blood flow during the first 24 hours following the injury). However, if his intracranial pressure became life threatening (i.e. brainstem herniation) and other methods of intracranial pressure reduction were unsuccessful, I would hyperventilate him to a CO2 of 25-30 mmHg. 6) The patient's dyspnea is significant and you decide to intubate him. On airway exam you note a Mallampati score of II. How will you proceed with intubation? Given the importance of avoiding aspiration and additional injury to the spinal cord, and recognizing that fiberoptic assisted intubation requires the least distraction of the cervical spine, I would: (1) provide aspiration prophylaxis (proton pump inhibitors should be avoided in this patient since they may inhibit the antiplatelet effects of clopidogrel and aspirin, placing this patient with a stent at increased risk for stent thrombosis); (2) administer a beta-agonist to optimize his asthma (this may be causing or contributing to his dyspnea); (3) establish inline stabilization to minimize cervical neck motion during topicalization of the airway (coughing can result in cervical spine movement) and intubation; (4) verify the presence of difficult airway equipment and a surgeon capable of obtaining a surgical airway; (5) ensure adequate airway analgesia so as to prevent a sympathetic response (undesirable in a patient with hypertension, CAD, and potentially elevated ICP)and/or coughing during intubation; (6) place the patient in 30° reverse-trendelenburg to improve respiratory mechanics (make sure that this positioning is hemodynamically tolerated in this trauma patient who may develop spinal shock); (7) pre-oxygenate with 100% oxygen; and (8) perform a careful awake fiberoptic assisted intubation. Clinical Note: this patient could certainly be induced prior to fiberoptic intubation. However, the risk for difficult airway management, aspiration, and hemodynamic instability should be considered 7) You utilize airway blocks to provide analgesia for the airway, but are unable to successfully intubate the patient with fiberoptic assistance due to the impairment of adequate visualization by blood in the oropharynx. You are considering a different plan when you notice premature ventricular contractions (PVCs) on the EKG. What is the "R-on-T phenomenon"? When premature ventricular contractions (which originate from foci below the atrial ventricular node) occur during the relative refractory period of the cardiac action potential (the middle of the T-wave), it is referred to as the "R-on-T phenomenon". The development of this phenomenon is concerning because it can lead to ventricular tachycardia or ventricular fibrillation. 8) There does not appear to be any R-on-T phenomenon, but the PVCs persist. What will you do? PVCs are relatively common during anesthesia, especially in the setting of preexisting cardiac disease, and don't always require treatment. However, I would initiate treatment if his PVCs were frequent (more than 3 PVCs per minute), polymorphic, occurred in runs of three or more, or demonstrated an R-on-T phenomenon, recognizing that these conditions are associated with an increased incidence of ventricular tachycardia and/or ventricular fibrillation. If treatment were necessary, I would begin with identifying and eliminating any causative factors such as arterial hypoxemia, myocardial ischemia, hypokalemia, hypomagnesemia, sympathetic activation, and/or mechanical irritation (such as may occur from a central venous line or a pulmonary artery catheter). I would also discontinue any drugs that are pro-dysrhythmic or prolong the QT interval. Additionally, I would ensure the presence of a defibrillator, in case a life-threatening dysrhythmia should develop. If the patient became symptomatic or hemodynamically unstable, I would consider overdrive pacing and/or an antidysrhythmic, such as amiodarone, procainamide, a beta-blocker, or sotalol (the latter should not be used if the QT is prolonged because it can induce Torsades). If the patient developed ventricular tachycardia, I would immediately cardiovert (unstable) or administer amiodarone (stable).Clinical Note: More than 5-6 PVCs per minute preoperatively is associated with increased perioperative morbidity. 9) You pull back the central line and the PVCs resolve. A few minutes later, you notice his blood pressure has dropped to 84/36 mmHg. What is your differential for his hypotension? Given this patient's recent car accident, cervical spine injury, coronary artery disease, long bone fracture, and ACE inhibitor therapy, there could be a number of factors contributing to his hypotension, such as: (1) the loss of sympathetic vasomotor tone and the cardioaccelerator fibers (T1-T4) below the level of injury; (2) cardiac tamponade; (3) tension pneumothorax (a higher risk considering his likely rib fractures and recent central line placement); (4) myocardial ischemia or infarction (secondary to CAD, anemia, hypoxia, hypotension, increased myocardial oxygen demand, coronary stent thrombosis); (5) occult bleeding (i.e. abdomen and/or long bone fracture); (6) pulmonary embolism, secondary to his long bone fracture (fat embolism) or venous thrombosis (loss of lower extremity vasomotor tone can lead to arterial and venous pooling, with subsequent thrombosis); and (7) blunting of his renin-angiotensin-aldosterone system by lisinopril (the sympathetic system of these patients may be compromised, making them even more dependent on the renin- angiotensin-aldosterone system for maintenance of blood pressure). 10) What is spinal shock? Spinal shock is a condition that may occur following acute spinal cord injury, and is characterized by flaccid paralysis, paralytic ileus, and the loss of sensation, spinal reflexes, sympathetic vasomotor tone, and temperature regulation below the level of injury. With higher lesions, the patient may experience the loss of diaphragmatic function (may require mechanical ventilation), intercostal and abdominal muscle function (contributing to respiratory dysfunction, an inability to effectively clear pulmonary secretions, and the risk for pulmonary infection), and the cardioaccelerator fibers (loss of the T1-T4 sympathetic innervation of the heart limits the ability to compensate for hypovolemia and decreased systemic vascular resistance). This condition typically persists for a 1-3 weeks (may last up to 3 months), during which time the patient is at risk for respiratory dysfunction, hemodynamic instability, pulmonary aspiration, deep venous thrombosis / pulmonary embolism, and hypothermia. 11) The patient remains dyspneic and is yet to be intubated. Assuming the airway exam is reassuring despite the facial trauma and blood in the oropharynx, would you perform a rapid sequence induction with succinylcholine? I would not perform a rapid sequence induction due to the risk for significant hypotension in this patient with chronic hypertension who is hemodynamically unstable and taking an ACE inhibitor (ACE inhibitors are associated with an increased risk of intraoperative hypotension). The administration of a large bolus of induction drug, as occurs when performing a rapid sequence induction, would increase the risk of end-organ ischemia in this hemodynamically unstable patient, who has coronary artery disease, spinal cord trauma, and potentially elevated intracranial pressure. However, I recognize that he is at increased risk for aspiration due to his GERD, a potentially full stomach (i.e. possible recent food ingestion; decreased gastric emptying secondary to pain and/or obesity), paralytic ileus, and impaired airway reflexes. Therefore, I would: (1) continue to administer fluids and vasoactive agents as necessary; (2) ensure the presence of difficult airway equipment and a surgeon capable of obtaining a surgical airway; (3) provide aspiration prophylaxis; (4) pre oxygenate the patient with 100% oxygen (patients with high spinal injury are more susceptible to significant bradycardia during tracheal suctioning, and hypoxemia accentuates this response); (5) direct two people to correctly initiate manual in-line stabilization; (6) remove the front of the c-collar to facilitate mouth opening (given the patient's cervical spine injury, I would not apply cricoid pressure, despite the risk of aspiration); (7) carefully titrate intravenous narcotics and etomidate to achieve an adequate plane of anesthesia to prevent bronchospasm or a sympathetic surge while, at the same time, avoiding worsening hemodynamic instability; (8) administer succinylcholine to facilitate intubation and prevent patient movement; and (9) perform careful laryngoscopy for ETT placement. 12) Is the administration of succinylcholine acceptable with acute spinal cord injury? The administration of succinylcholine would be acceptable for the first 24-48 hours following acute spinal cord injury. However, the proliferation of cholinergic receptors that subsequently occurs places the patient at risk for a potentially lethal succinylcholine-induced hyperkalemic response. This exaggerated hyperkalemic response to succinylcholine does not occur until 24-48 hours after the spinal cord injury, is most pronounced 4 weeks to 5 months following the injury, may persist for an extended period of time (while the risk decreases significantly 6 months following the injury, it is best to avoid succinylcholine administration in these patients, if possible), and can lead to ventricular tachycardia, fibrillation, and cardiac arrest. 13) You attempt to intubate the patient and are unable to visualize the cords under direct laryngoscopy. Would you remove the c-collar in order to improve the view? I would not remove the c-collar due to the potential risk for further neurological damage with mobilization of his neck. However, I recognize that delayed intubation places this patient at risk for aspiration and hypoxia. Therefore, I would attempt to optimize visualization by changing laryngoscopy blades and/or patient positioning (i.e. reverse-trendelenburg, if tolerated). If these measures were unsuccessful, I would maintain in-line stabilization, provide gentle positive pressure ventilation with 100% oxygen (< 20 mmHg to prevent gastric distention), and ask the surgeon to perform a tracheostomy.

1) Five hours after pulling the epidural catheter, the patient is febrile and experiencing back pain and bilateral leg weakness. What do you think? 2) The patient subsequently develops urinary incontinence. What will you do? 3) You determine that her urinary incontinence and leg weakness are the result of relapsing multiple sclerosis. The obstetrician asks you when she can restart her preoperative dose of enoxaparin. What would you tell him? 4) Prior to restarting the enoxaparin, the patient becomes dyspneic. What do you think may be the cause? 5) How is pulmonary thromboembolism diagnosed? 6) Assuming this were pulmonary embolism, how would you treat her? 7) Two days later she is extubated, stable, and complaining of a headache. What do you think? 8) Assuming it is a post-dural puncture headache, how would you treat her?

While her symptoms may be secondary to a combination of less serious conditions such as bacteremia from chorioamnionitis (fever), tissue damage with needle instrumentation (back pain), and residual epidural blockade (ongoing bilateral leg weakness), I would be concerned that they were representative of something more significant, such as relapsing multiple sclerosis or epidural/spinal hematoma. While the exacerbations associated with multiple sclerosis are usually reduced during pregnancy, this patient's fever (secondary to chorioamnionitis) places her at increased risk for relapse. Likewise, the peripartum administration of anticoagulants combined with traumatic epidural catheter placement may increase her risk for epidural or spinal hematoma, especially if the plan of care deviated from ASRA recommendations. Therefore, in order to rule out a more serious condition, I would evaluate the patient for signs and symptoms consistent with epidural/spinal hematoma, such as back pain or pressure that is severe and unrelenting, tenderness over the spinous or paraspinous area, bowel or bladder dysfunction, progressive weakness (rather than recessive weakness), paresthesias, sensory deficits, and unexplained fever (hers would be explained by her chorioamnionitis). Likewise, I would look for signs and symptoms consistent with relapsing multiple sclerosis, such as paresthesias, weakness, sensory deficits, urinary incontinence, bowel retention, visual and gait disturbances, emotional lability, and autonomic dysfunction. While this patient's symptoms would also be consistent with epidural/spinal abscess or meningitis, these conditions are less likely to occur so soon after neuraxial placement. 2) The patient subsequently develops urinary incontinence. What will you do? Urinary incontinence combined with leg weakness is consistent with are lapse of her multiple sclerosis. However, given the peripartum administration of anticoagulants, the traumatic epidural placement, and her other symptoms consistent with spinal cord compression secondary to epidural or spinal hematoma (i.e. leg weakness, back pain, and unexplained fever), I would aggressively attempt to rule out this potentially catastrophic complication. To this end, I would: (1) examine the patient for tenderness over the spinous or paraspinous area; (2) attempt to determine if her weakness was progressive, rather than recessive (recessive weakness would be more consistent with slowly resolving neuraxial blockade); (3) obtain an MRI of the spine to identify spinal cord compression; and, if this finding was confirmed, (4) consult a neurosurgeon for possible spinal cord decompression (which should occur within 6-12 hours in order to avoid irreversible spinal cord injury). 3) You determine that her urinary incontinence and leg weakness are the result of relapsing multiple sclerosis. The obstetrician asks you when she can restart her preoperative dose of enoxaparin. What would you tell him? Since this patient is receiving twice-daily dosing of enoxaparin, I would recommend that her first post-operative dose occur at least 24 hours following surgery. Moreover, I would ensure that the epidural catheter was removed at least 2 hours prior to restarting this anticoagulant, since an indwelling catheter is contraindicated in conjunction with twice-daily dosing of enoxaparin. I would also make it clear that the decision as to when to restart thromboprophylaxis following surgery and neuraxial instrumentation involves weighing the risks of thromboembolism against the risks of surgical bleeding and epidural/spinal hematoma. 4) Prior to restarting the enoxaparin, the patient becomes dyspneic. What do you think may be the cause? Given this pregnant patient's deep vein thrombosis, severe aortic stenosis, asthma, relapsing multiple sclerosis, preeclampsia, increased risk for aspiration, and recent central line placement, my differential for her dyspnea would include: (1) pulmonary thromboembolism (deep vein thrombosis, temporary discontinuation of enoxaparin, pregnancy, preeclampsia, and cesarean section all increase the risk); (2) heart failure (aortic stenosis, myocardial ischemia, loss of "atrial kick" with dysrhythmia); (3) myocardial infarction (aortic stenosis places her at increased risk); (4) bronchospasm (asthma); (5) tension pneumothorax (recent central line placement); (6) pulmonary edema (preeclampsia, aortic stenosis); (7) aspiration (pregnancy); and (8) multiple sclerotic involvement of the brain stem (can lead to hypoxemia and respiratory failure). 5) How is pulmonary thromboembolism diagnosed? Many of the manifestations of pulmonary embolism are nonspecific, making a clinical diagnosis very difficult. Therefore, diagnostic testing is often required in addition to clinical assessment to establish a diagnosis and initiate treatment. Clinical signs and symptoms of pulmonary thromboembolism include: (1) dyspnea, (2) tachypnea, (3) cough, (4) hemoptysis, (5) tachycardia, (6) fever, (7) accentuated or split second heart sound, (8) pleuritic pain, (9) localized rales, (10) hypoxemia, (11) hypocapnea, (12) thrombophlebitis (unilateral extremity swelling, erythema, and tenderness), (13) jugular venous distention (pulmonary hypertension can lead to right heart failure and jugular venous distention), (14) hemodynamic instability, (15) palpitations, (16) central venous and pulmonary artery catheter changes, such as normal to low pulmonary artery occlusion pressure (<15 mmHg), increased pulmonary artery pressure, and increased central venous pressure, and (17) EKG changes, such as new right bundle branch block, ST-T wave changes, peaked P waves, right-axis deviation, T-wave inversion, and supraventricular arrhythmias. Diagnostic testing options include: (1) lower extremity venous ultrasonography (for DVT), (2) Spiral (helical) computed tomography, (3) ventilation-perfusion scan, (4) pulmonary angiography (gold standard), (5) CXR (atelectasis, pleural effusion, elevated hemidiaphragm, segmental infiltrate), (6) D-dimer assay, (7) transthoracic echocardiography (right ventricular overload), and (8) transesophageal echocardiography (acute right atrial and/or ventricular dilation, pulmonary artery hypertension, thrombus in pulmonary arteries). 6) Assuming this were pulmonary embolism, how would you treat her? In treating her pulmonary thromboembolism, I would: (1) apply 100% oxygen; (2) administer inotropes and fluids using the CVP and PAC to guide therapy; (3) provide analgesics as hemodynamically tolerated; (4) continue monitoring with the arterial line, central venous pressure catheter, and pulmonary artery catheter; (5) consider administering a pulmonary vasodilator, if she developed pulmonary hypertension (the phosphodiesterase inhibitors, amrinone and milrinone, cause pulmonary arterial vasodilation and improve myocardial contractility); (6) intubate and initiate mechanical ventilation, if necessary; and (7) transport the patient to the ICU when stable enough for transport. While anticoagulation with unfractionated heparin or LMWH would be desirable in order to prevent the formation of new clots and limit the extension of existing clots, it may be advisable to avoid it in this case due to her recent surgery and the subsequent increased risk of bleeding. Rather, consideration should be given to the placement of a vena-cava filter to prevent the passage of lower extremity emboli into the heart and lungs. If she were experiencing massive pulmonary embolism with severe hemodynamic depression, severe hypoxemia, and cardiac arrest, or was unresponsive to resuscitative measures, I would employ transesophageal echocardiography and consider pulmonary embolectomy. Given the high risk of bleeding having undergone recent surgery, I would only administer thrombolytics as a treatment of last resort. 7) Two days later she is extubated, stable, and complaining of a headache. What do you think? The most common type of postpartum headache is a tension headache. However, given her history of migraines and recent accidental dural puncture with an epidural needle, I would place migraine headache, lactation headache (patients with a history of migraine headaches sometimes experience an intense headache during breast feeding), post-dural puncture headache (PDPH), and pneumocephalus (often sudden in onset following dural puncture using air for loss of resistance; may persist for up to a week) very high on my differential. However, I would consider the possibility that her headache was related to her preeclampsia (headache is a premonitory sign of eclampsia) or, given her recent chorioamnionitis with fever (suggestive of bacteremia), that it was associated with meningitis. Moreover, pregnancy may increase the risk of subarachnoid hemorrhage (possibly due to increased blood volume combined with hormonally-induced changes affecting arterial integrity), while dural puncture increases the risk of subdural hematoma (leakage of CSF decreased ICP -> increased stress on cerebral bridging vessels) and cortical vein thrombosis (leakage of CSF cerebral vasodilation and decreased ICP predisposition to cerebral thrombosis). Finally, I would consider other potential causes of postpartum headache, such as caffeine withdrawal, sinusitis, cerebral ischemia/infarction, and intracranial tumor. 8) Assuming it is a post-dural puncture headache, how would you treat her? I would explain that epidural blood patch is the most affective treatment for PDPH and involves the injection of 15-20 mL of her blood into the epidural space at the level of the dural puncture. However, assuming she was receiving anticoagulants (heparin, LMWH, or warfarin) as part of her treatment for pulmonary embolism, an epidural blood patch would be inappropriate due to the increased risk for epidural or spinal hematoma. Therefore, I would initiate more conservative treatment options such as hydration (no evidence of therapeutic benefit), caffeine, the placement of an abdominal binder (increases abdominal pressure, possibly leading to an increase in CSF pressure), and pain control. Moreover, I would reassure her that PDPHs are self-limited and almost always resolve within a week.

1) After multiple attempts at placement, you unintentionally enter the intrathecal space with the epidural needle. Would you place an intrathecal catheter for neuraxial anesthesia? 2) What are the signs and symptoms of post-dural puncture headache? 3) You place an epidural catheter, which is working well for labor, but the baby's heart tones are non-reassuring and the obstetrician wants to perform a cesarean section. Would you provide any preoperative medications? 4) Would you provide perioperative steroid supplementation? 5) The baby's heart tones go down into the 50's for 4 minutes and the obstetrician makes the cesarean section emergent. Would you use her neuraxial catheter for the case? 6) Is using a higher concentration local anesthetic, as would be required for cesarean section, acceptable in a patient with multiple sclerosis? 7) You aspirate blood through the catheter and decide to provide a general anesthetic. You are concerned about her airway, but she refuses an awake intubation. How will you induce her? 8) How will you maintain anesthesia? 9) You notice ST depression on the EKG. Would you give nitroglycerine? 10) The ST depression resolves with treatment and the baby is delivered. Following delivery, the uterus is "boggy" and the patient has lost 1400 mL of blood. What would you do? 11) Would you use any other uterotonic agents?

While the placement of an intrathecal catheter would avoid further instrumentation and potentially reduce the risk of post-dural puncture headache (leaving the catheter in place for 24 hours following delivery may reduce the incidence of PDPH by reducing the loss of CSF into the epidural space and by inducing an inflammatory response that promotes closure of the puncture site), I would not place and utilize an intrathecal catheter for analgesia because of the increased risk of exacerbating her multiple sclerosis and due to my desire to avoid a rapid sympathectomy in a patient with preeclampsia and severe aortic stenosis. The fact that many preeclamptic patients are hypovolemic, despite increased systemic vascular resistance, further increases my concern that a rapid sympathectomy could lead to significant hypotension with a subsequent reduction in preload (which would not be well tolerated by someone with severe aortic stenosis). Instead, I would place an epidural catheter and make the patient aware of the risks, signs and symptoms, and therapeutic options associated with post-dural puncture headache. 2) What are the signs and symptoms of post-dural puncture headache? The lost CSF, decreased buoyant support for the brain, and cerebral vasodilation (increased cerebral blood flow to compensate for decreased CSF) that accompanies significant dural puncture, can lead to a PDPH, the signs and symptoms of which include a frontal-occipital headache, decreased pain with recumbent positioning, nausea, vomiting, neck stiffness, back pain, photophobia, diplopia, difficulty in accommodation, tinnitus, and hearing loss. Rarely, PDPH is associated with seizures (most likely secondary to cerebral vasospasm), abdominal pain, and diarrhea. Loss of CSF can also lead to cranial nerve stretching with subsequent palsy. Stretching of the sixth cranial nerve (the cranial nerve that is the most susceptible to traction) impairs eye abduction, and may result in diplopia. Hearing loss may occur due to reduced CSF pressure and an alteration of hair cell position in the inner ear. 3) You place an epidural catheter, which is working well for labor, but the baby's heart tones are non-reassuring and the obstetrician wants to perform a cesarean section. Would you provide any preoperative medications? As time permitted, I would give her: (1) albuterol to optimize her asthmatic condition; (2) metoclopramide, an H2-blocker, and a nonparticulate antacid to reduce the risk of aspiration pneumonitis; (3) corticosteroid supplementation (her steroid treatment may have resulted in adrenal suppression), and (4) consider administering a beta-blocker to prevent tachycardia in this patient with severe aortic stenosis (beta-blockers must be used cautiously in asthmatic patients). While the administration of a benzodiazepine would be helpful in preventing anxiety and the resultant tachycardia, it would potentially lead to neonatal hypotonia secondary to placental transfer. 4) Would you provide perioperative steroid supplementation? I would provide her with steroid supplementation, since her prior treatment with exogenous steroids may have resulted in suppression of her hypothalamic-pituitary-adrenal (HPA) axis and an inability to produce adequate cortisol under the physiologic stresses experienced during the perioperative period. Insufficient cortisol production could result in an Addisonian crisis, a life-threatening condition during which patients may experience fever, abdominal pain, dehydration, nausea and vomiting, hypoglycemia (cortisol promotes gluconeogenesis, has anti insulin affects, and inhibits peripheral utilization of glucose), acidosis, hyperkalemia, hyponatremia, circulatory collapse (this would be even more concerning in the setting of severe aortic stenosis), and depressed mentation. Perioperative corticosteroid supplementation is controversial because of the questionable necessity of supplementation beyond the patient's usual steroid dose, and due to the potential deleterious effects of supraphysiologic doses of steroids, such as infection, poor wound healing, fluid retention, electrolyte imbalances, immunosuppression, hypertension, and hyperglycemia. However, many of these side effects are unproven and/or not clinically significant; and the risk/benefit ratio of administering supraphysiologic doses of steroids is usually considered to be small. Therefore, assuming this patient was receiving at least the equivalent of 5 mg of prednisone per day (long term suppression of the hypothalamic-pituitary-adrenal axis is unlikely with smaller doses), I would administer 75 mg of intravenous hydrocortisone preoperatively, with a rapid taper to her usual dose in 1 -2 days. Clinical Note: Common Steroid Replacement Regimens 1) 100 mg of intravenous hydrocortisone preoperatively, followed by 100 mg every 8 hours on the day of surgery. 2) 25 mg of intravenous hydrocortisone at induction, followed by 100 mg over the next 24 hours. 3) Minor Surgery: 25 mg of intravenous hydrocortisone preoperatively, on the day of surgery. No additional tapered dosing is required. Moderate Surgery: 50-75 mg of intravenous hydrocortisone preoperatively, on the day of surgery. Tapered dosing: 50 mg intraoperatively, followed by 20 mg every 8 hours on the first day, and returning to the patient's usual dose on day 2. Severe Surgery: 100-150 mg of intravenous hydrocortisone preoperatively, on the day of surgery. Tapered dosing: 50 mg intraoperatively, followed by 25-50 mg every 8 hours for 2 days, and returning to the patient's usual dose on day 3. 5) The baby's heart tones go down into the 50's for 4 minutes and the obstetrician makes the cesarean section emergent. Would you use her neuraxial catheter for the case? Given my concerns about difficult airway management in this obese and pregnant patient with a Mallampati score of 3, my preference would be to utilize her epidural catheter to provide anesthesia for the cesarean section. However, given the patient's preeclampsia (likely hypovolemia) and the need to avoid a precipitous drop in systemic vascular resistance in the presence of severe aortic stenosis, the establishment of adequate levels of analgesia for cesarean section would likely require significantly more time than would inducing general anesthesia, placing this potentially distressed baby at increased risk. Therefore, I would optimize the mother's hemodynamics, ensure adequate left uterine displacement, and provide 100% oxygen, hoping to improve the baby's condition. If the heart tones remained down, I would perform a more thorough airway evaluation, weigh the risks of difficult airway management against the risks of delaying delivery of the baby, and make a decision to proceed with either regional or general anesthesia, always keeping in mind that the mother is my primary patient. Test Taking Note: The "UBP Answer" to this question demonstrates how you can demonstrate a great deal of knowledge and still fail to answer the question being asked. Making this mistake during your exam is very likely to frustrate your examiner because they need you to make a decision and answer the question. 6) Is using a higher concentration local anesthetic, as would be required for cesarean section, acceptable in a patient with multiple sclerosis? While there is evidence that the use of higher concentrations of local anesthetic for epidural analgesia may increase the risk of exacerbating the symptoms of multiple sclerosis, this risk may be minimized by the usually short duration of cesarean section, which limits the progressive increase in CSF concentration of local anesthetic. So, while the use of regional anesthesia for cesarean section in patients with multiple sclerosis is controversial, some practitioners feel that the benefits of utilizing regional anesthesia (avoidance of airway instrumentation, reduced risk of aspiration, superior post-operative analgesia, etc.) outweigh the potential risks. 7) You aspirate blood through the catheter and decide to provide a general anesthetic. You are concerned about her airway, but she refuses an awake intubation. How will you induce her? Given her potentially difficult airway and various comorbidities, I would: (1) verify the presence of the appropriate lines and monitors; (2) make sure all premedications had been given, including esmolol to prevent tachycardia during laryngoscopy (esmolol is short acting and B1-selective for asthmatic patient); (3) ensure the presence of difficult airway equipment, external cardioversion pads (loss of the "atrial kick" is not well tolerated by patients with severe aortic stenosis), and phenylephrine (a better choice to treat reduced SVR than ephedrine); (4) place the patient in the sniff position with adequate left uterine displacement (while reverse- trendelenburg position would also facilitate rapid intubation, it may not be tolerated due to decreased cardiac filling); (5) apply cricoid pressure; (6) administer a combination of etomidate and narcotics (etomidate is less likely to result in cardiovascular depression and the narcotics would help to avoid tachycardia and/or bronchospasm in this patient with severe aortic stenosis and asthma); (7) rapidly secure the airway with an endotracheal tube, and (8) let the neonatal resuscitation team know about the recent administration of narcotics. My goals during induction would be to maintain spontaneous ventilation (difficult airway); avoid tachycardia, bradycardia, and/or myocardial depression (aortic stenosis); achieve an adequate depth of anesthesia to avoid bronchospasm (asthma) while maintaining SVR (aortic stenosis); and reduce the risk of aspiration (obesity and pregnancy). Clinical Note: Succinylcholine would not be appropriate in this case due to the potential for Difficult airway management and the risk of an exaggerated hyperkalemic response (secondary to upregulation of ACh receptors at the neuromuscular junction with chronic skeletal muscle weakness). 8) How will you maintain anesthesia? Prior to delivery of the baby, I would maintain anesthesia primarily with a volatile agent, administering narcotics only if necessary to prevent tachycardia in this patient with severe aortic stenosis (if possible, narcotics and benzodiazepines should be avoided until the baby is delivered). Following delivery, I would maintain anesthesia and ensure amnesia by providing a low concentration of volatile agent, a small dose of benzodiazepine, and intravenous narcotics. If I were concerned about impaired left ventricular function, I would consider avoiding volatile agents and administering higher doses of narcotics (nitrous oxide could also be utilized). Given her severe aortic stenosis, my goal would be to maintain adequate anesthesia while avoiding myocardial depression, decreased systemic vascular resistance, tachycardia, and dysrhythmias. 9) You notice ST depression on the EKG. Would you give nitroglycerine? Nitroglycerine would not be my first choice. While it could potentially improve coronary blood flow to more ischemic areas of the subendocardium and relieve any coronary artery spasm, it could also lead to significantly reduced preload (nitroglycerine produces significant venodilation in addition to some arterial dilation). Although a reduction in afterload is often beneficial for patients experiencing myocardial ischemia and/or left ventricular dysfunction, the potential drop in preload secondary to venodilation may not be well tolerated by a patient with severe aortic stenosis. Moreover, the benefits usually obtained through a reduction in afterload are limited due to the relatively fixed afterload imposed by the stenotic aortic valve. Therefore, in the presence of systemic hypotension, I may consider the administration of a vasoconstrictor, recognizing that decreases in systemic vascular resistance are more likely to decrease coronary perfusion during diastole than to reduce myocardial oxygen demand. In a situation where I believed a reduction in afterload would be beneficial (e.g. left ventricular dysfunction), I would consider administering nicardipine, carefully titrated to reduce afterload without significantly affecting ventricular preload (some consider nicardipine to be preferable to SNP or nitroglycerine since it results in less venodilation). 10) The ST depression resolves with treatment and the baby is delivered. Following delivery, the uterus is "boggy" and the patient has lost 1400 mL of blood. What would you do? First, I would verify that the patient was receiving an infusion of Pitocin, keeping in mind that Pitocin can cause peripheral vasodilation with subsequent hypotension and tachycardia, and being prepared to treat these cardiovascular affects aggressively with phenylephrine (tachycardia and hypotension significantly increase the risk of myocardial ischemia in the setting of severe aortic stenosis). In addition, I would: (1) apply 100% oxygen and check my monitors to ensure the patient was stable (especially the EKG and blood pressure, given this patient's severe aortic stenosis); (2) encourage the obstetrician to continue bimanual compression and uterine massage; (3) verify adequate IV access and obtain a type and cross-match; (4) order baseline coagulation studies and a complete blood count; (5) notify the blood bank of the possible need for blood transfusion; and (6) observe the surgical field (including the uterus, if externalized) looking for an obvious cause of continued bleeding. If it appeared that uterine atony was the cause of continued bleeding, I would consider reducing my inhalational agent and increasing the dose of oxytocin. If these interventions were unsuccessful or inadequate, I would consider administering a rectal dose of misoprostol (Cytotec; a prostaglandin E1 analogue; 800-1000 micrograms per rectum). Finally, if all of these measures were unsuccessful, the surgeon may have to employ an intrauterine balloon (works via tamponade; success rate up to 80%); place uterine compression sutures; ligate the internal iliac, uterine, and ovarian arteries; and/or perform an emergency hysterectomy. 11) Would you use any other uterotonic agents? Only as a last resort. While methylergonovine (methergine), 15- methyl-prostaglandin F2-alpha (hemabate), and dinoprostone (prostaglandin E2) are often utilized in the treatment of uterine atony, I would be reluctant to administer these drugs to this patient with preeclampsia, asthma, chorioamnionitis, and severe aortic stenosis for the following reasons: (1) methergine should be avoided, if possible, in preeclamptic patients due to the potential for exacerbated hypertension; (2) methergine can also cause coronary artery vasoconstriction, which would not be well tolerated by this patient whose myocardial oxygen supply is already compromised, and who has already experienced significant blood loss (resulting in reduced oxygen carrying capacity); (3) hemabate may cause bronchospasm and is relatively contraindicated in asthmatic patients; and (4) the decreased systemic vascular resistance and tachycardia often associated with dinoprostone administration would not be well tolerated in the setting of severe aortic stenosis. Clinical Note: Prostaglandins (i.e. hemabate and dinoprostone) tend to be less effective in patients with chorioamnionitis.

explain typical closure of ductus arteriosus? why didn't it close in this instance?

With the initiation of ventilation, arterial oxygen levels are increased and pulmonary vascular resistance is reduced. The decrease in pulmonary vascular resistance leads to a reversal of flow through the ductus arteriosus, thereby exposing the ductus to systemic blood with a relatively higher oxygen concentration. This exposure to increased oxygen levels in combination with the rapid decrease in circulating prostaglandins (primarily prostaglandin E2) that occurs following placental separation results in the functional closure of the ductus within 2-4 days of birth. Permanent closure (ductal fibrosis) occurs over several weeks, leaving a residual band of tissue called the ligamentum arteriosum. 2) What is the most likely reason it didn't close in this case? In this case, this neonate's ductus arteriosus has most likely remained patent due to a combination of hypoxia (baby was in respiratory distress) and the thinner, less responsive muscular layer in the ductus of premature neonates. In the setting of hypoxia, the normal increase in arterial oxygen concentration that occurs following delivery is diminished or absent, thereby blunting one of the primary stimulants of ductal closure. This is even more of a problem for premature neonates, because their ductus arteriosus is comprised of a poorly contractile muscular layer, making it less responsive to increasing oxygen levels and other endogenous mediators (e.g. bradykinin). Predisposing factors to PDA include prematurity, respiratory distress syndrome (RDS), hypoxia, acidosis, and excessive fluid therapy.

A 5-day-old male infant weighing 980 grams and born at 28 weeks gestation is scheduled for ligation of a patent ductus arteriosus (PDA). After delivery, the infant was intubated while still in the operating room secondary to respiratory distress. On day four, his respiratory condition deteriorated further with worsening hypoxia and increased CO2 retention. Auscultation of the chest revealed a systolic murmur, and a PDA was confirmed with echocardiography. Medical management failed to resolve the condition and the decision was made to surgically ligate the PDA. He is currently receiving indomethacin and being mechanically ventilated with a FiO2 of 50%. 1) Tell me about the normal closure of the ductus arteriosus 2) What is the most likely reason it didn't close in this case? 3) What is respiratory distress syndrome of the newborn (RDS)? 4) Why is this infant taking indomethacin? 5) An echocardiogram shows left atrial enlargement. Why would a PDA result in this finding? 6) What would be included in your pre-operative evaluation of this infant? 7) Would you administer any premedication? 8) The urinalysis reveals 1+ glucosuria. Does this concern you? 9) What are the potential complications that you are anticipating during this procedure?

With the initiation of ventilation, arterial oxygen levels are increased and pulmonary vascular resistance is reduced. The decrease in pulmonary vascular resistance leads to a reversal of flow through the ductus arteriosus, thereby exposing the ductus to systemic blood with a relatively higher oxygen concentration. This exposure to increased oxygen levels in combination with the rapid decrease in circulating prostaglandins (primarily prostaglandin E2) that occurs following placental separation results in the functional closure of the ductus within 2-4 days of birth. Permanent closure (ductal fibrosis) occurs over several weeks, leaving a residual band of tissue called the ligamentum arteriosum. 2) What is the most likely reason it didn't close in this case? In this case, this neonate's ductus arteriosus has most likely remained patent due to a combination of hypoxia (baby was in respiratory distress) and the thinner, less responsive muscular layer in the ductus of premature neonates. In the setting of hypoxia, the normal increase in arterial oxygen concentration that occurs following delivery is diminished or absent, thereby blunting one of the primary stimulants of ductal closure. This is even more of a problem for premature neonates, because their ductus arteriosus is comprised of a poorly contractile muscular layer, making it less responsive to increasing oxygen levels and other endogenous mediators (e.g. bradykinin). Predisposing factors to PDA include prematurity, respiratory distress syndrome (RDS), hypoxia, acidosis, and excessive fluid therapy. 3) What is respiratory distress syndrome of the newborn (RDS)? Neonatal respiratory distress syndrome is a condition that commonly occurs shortly after delivery in premature infants secondary to insufficient surfactant production (production is usually inadequate prior to 35 weeks gestation). Insufficient surfactant results in widespread atelectasis following delivery, which in turn leads to intrapulmonary shunting with subsequent hypoxemia and metabolic acidosis. The infant with RDS typically exhibits tachypnea, tachycardia, nasal flaring, intercostal and subcostal retractions, bilateral rales (that fail to clear with suctioning), and cyanosis. Radiographic examination reveals diffuse, "ground glass" bilateral infiltrates and reduced lung volumes (secondary to atelectasis), while an ABG shows hypoxemia, metabolic acidosis, and respiratory alkalosis. Fortunately, survival in infants who develop RDS has been greatly increased with maternal steroid administration (to increase surfactant production in utero) and neonatal exogenous surfactant administration. Clinical Notes: · ACOG recommends a single course of antenatal corticosteroids be administered to any woman after 24 weeks gestation who is at risk for delivery before 34 weeks. Antenatal steroids are, therefore, administered between 24 and 34 weeks. · Risk factors for RDS include: (1) low gestational age, (2) low birth weight, and (3) surgical delivery without labor. · The long-term consequence of RDS is the potential development of bronchopulmonary dysplasia. 4) Why is this infant taking indomethacin? Locally produced and circulating prostaglandins (primarily PGE1 and PGE2) play a major role in the patency of the ductus arteriosus through smooth muscle relaxation. Therefore, prostaglandin synthetase inhibitors, like indomethacin and ibuprofen, are often used in the medical management of PDA to encourage closure. Side effects associated with indomethacin include thrombocytopenia, hyponatremia, and reduced renal (renal failure), mesenteric (intestinal perforation), and cerebral blood flow. Ibuprofen is often utilized in low birth weight premature neonates, because its efficacy is equivalent to that of indomethacin, and it has less detrimental effects on end-organ perfusion. 5) An echocardiogram shows left atrial enlargement. Why would a PDA result in this finding? Neonates with a PDA may develop left atrial enlargement because the PDA allows shunting of blood from the systemic to the pulmonary circulation. When the shunt is large, the blood flow to the lungs can be 3-4 times normal, resulting in volume overload of the left heart and pulmonary vasculature (potentially leading to left heart and respiratory failure). Volume overloading may then lead to left atrial dilation and left ventricular hypertrophy. If the condition is left untreated, pulmonary vascular overload leads to irreversible pulmonary hypertension, right heart failure, and a reversal of the left-to-right shunt to a right-to-left shunt (Eisenmenger's syndrome). The fact that this child is experiencing left atrial enlargement would suggest that blood is shunting from left-to-right despite his initial respiratory distress (which often leads to increased pulmonary vascular resistance). Echocardiography with Doppler studies may prove helpful in confirming the direction of flow through the PDA. Clinical Note: · The direction of flow through the PDA is determined by the difference in pressure between the pulmonary and systemic vasculature, and by the length and diameter of the ductus arteriosus. 6) What would be included in your pre-operative evaluation of this infant? I would begin by reviewing the chart and having a discussion with the neonatologist to determine the maternal drug history, the infant's treatment course, and the infant's current respiratory, cardiovascular, and fluid status (e.g. blood pressures, ABG results, ventilator settings, and oxygen requirements). Next, I would perform a careful physical exam focusing on the airway, fluid status, and the cardiopulmonary system. Finally, I would type and cross the patient (blood loss is usually minimal, but could be massive if a major blood vessel, such as the aorta, pulmonary artery, or ductus, is torn) and review or order a chest x-ray (to identify any pulmonary edema or cardiac dilation), ABG (to identify any metabolic or respiratory acidosis), urinalysis (indomethacin can effect renal function), H/H (potential for major blood loss), and serum electrolyte levels (indomethacin can cause hyponatremia, the neonate's small intravascular volume places them at increased risk for electrolyte disturbances). 7) Would you administer any premedication? I would probably not premedicate this sick neonate with narcotics or a benzodiazepine, as sedation is unnecessary in this age group. Atropine (0.01-0.02 mg/kg) is sometimes administered to (1) decrease secretions, (2) minimize vagal reflexes resulting from laryngoscopy and/or surgical manipulation (e.g. strabismus surgery), and (3) prevent bradycardia secondary to succinylcholine and/or volatile agent administration (the latter results secondary to myocardial depression). However, since modern volatile agents are not associated with significant bradycardia and because timing the anticholinergic effects with induction is often difficult, I would not administer it as a premedication in this case. I would, instead, have this drug available at induction and administer it if necessary. 8) The urinalysis reveals 1+ glucosuria. Does this concern you? I am concerned, as glucose homeostasis is somewhat precarious in premature infants secondary to reduced glycogen stores and immature renal function. However, glucosuria is not uncommon in preterm infants less than 34 weeks gestation due to reduced renal tubular reabsorption of glucose. If the infant were > 34 weeks gestation, I would be more concerned that this finding represented hyperglycemia. 9) What are the potential complications that you are anticipating during this procedure? Potential surgical complications associated with this procedure include: (1) recurrent laryngeal nerve injury (hoarseness), (2) left phrenic nerve injury (left hemidiaphragm paralysis), (3) thoracic duct injury (chylothorax), (4) massive blood loss (secondary to inadvertent ligation or laceration of the ductus arteriosus, aorta or pulmonary artery), (5) hypertension (often occurs post-operatively following the ligation of the ductus arteriosus), and (6) reopening of the ductus (a rare event that may occur when the ductus is ligated without division). The baby's prematurity and extremely low birth weight, place him at increased risk for (7) hypothermia, (8) retinopathy of prematurity, (9) intraventricular hemorrhage, (10) postoperative apnea, and (11) hypoglycemia. Given the neonate's cardiopulmonary status and the nature of the surgery, I would also be concerned about (12) hypoxemia (secondary to retraction on the lung or pulmonary hypertension leading to intrapulmonary and/or extrapulmonary right-to-left shunting), (13) heart failure (due to cardiac ischemia, increased after load, and/or increased left-to-right shunting), and (14) hypotension (secondary to massive blood loss, anesthetic overdose, vagal reflex in response to surgical manipulation of the vagus nerve or lung tissue, heart failure, and/or compression of the heart or great vessels).

consider placing a pulmonary artery catheter for these conditions:

heart failure, significant coronary artery disease, valvular heart disease, left ventricular dysfunction, or severe chronic obstructive pulmonary disease


Set pelajaran terkait

ch6 installing a physical network

View Set

5-2 Properties of Quadratic Functions in Standard Form

View Set

13; The British Invasion Continues

View Set